SóProvas



Questões de Quantificadores


ID
203488
Banca
FEPESE
Órgão
SEFAZ-SC
Ano
2010
Provas
Disciplina
Raciocínio Lógico
Assuntos

Em relação à afirmação


Se x = 16 e y ≥ 7 então x∙y ≥ 112



pode-se concluir que:

Alternativas
Comentários
  • Na hora da prova você se desespera  com uma dessas. Após tentar converter as alternativas em variáveis A, B e C, vi que perderia muito tempo, assim procurei outra solução.

    Como a premissa era uma condicional e todas as alternativas também, pensei na hora que só podia ser questão de equivalência.

    A letra A é o uso da equivalência abaixo:

    A -> B = ~B -> ~A
  • Tinha que arrumar formatação dessa questão. Só fui entender olhando direto na prova. 
  • Será que alguem pode me explicar a letra c?
    Grata e bons estudos
  • A letra C está igual aos dados do enunciado..
  • Natalia na letra "C" não há equivalência lógica, senão vejamos:
    c) A ---> B, B---->A 'nenhuma dessas duas formas se equivale a proposição do enunciado.

    Para tal seria necessário duas formas:
    1. ~B----->~A,  ~A ou B.
    OBS.: a correta equivale-se à primeira forma da 1.
  • É suficiente usar a equivalência p ^ q -> r = ~r -> ~p v ~q, tomando:

    p: x = 16. -----> ~p: x ~= 16 (o símbolo "~=" quer dizer "diferente")

    q: y >= 7. -----> ~q: y < 7

    r: x.y >= 112. -----> ~r: x.y < 112.

    Temos que p ^ q -> r é igual a "se x = 16 e y >= 7, então x.y => 112", que é igual a ~r -> ~p v ~q, ou seja, "se x.y < 112, então x ~= 16 ou y < 7."

    Resposta: a.

    Opus Pi.

  • Fiz da seguinte maneira:


    termo A) x =16

    termo B) Y ≥ 7

    termo C) X .Y  112


    Sendo assim, o enunciado ficaria:

    (A ^ B) → C


    Tabela verdade do enunciado:

    A     B     C     (A ^ B)     (A ^ B)→C

    V     V     V         V                V

    V     V     F         V                F

    V     F     V         F                V

    V     F     F         F                V

    F     V     V         F                V

    F     V     F         F                V

    F     F     V         F                V

    F     F     F         F                V


    Observando bem, percebe-se que a alternativa A é exatamente:

    ~C → (~A v ~B)


    A última coluna da tabela verdade para alternativa A é exatamente igual a do enunciado.


    Espero ter ajudado,


    Bons estudos a todos!

  • Questão possui uma condicional p e q - > r 

    p = x = 16

    q = y >_ 7

    r = x.y >_ 112

     

    a equivalência de ( p e q ) -> r é:

    ~r -> ~ (p e q)

     

    Logo, ela pode ser rescrita assim:

    ~r -> (~p ou ~q) 

     

    ficando

    ~p = x diferente 16 --------------- troquei o sinal = por diferente

    ~q = y > 7

    ~r = x.y > 112

    escrevendo ~r -> (~p ou ~q) 

     

    fica igual alternativa A -----> x.y > 112, então x diferente 16 ou y < 7

     

    questão dificil, ja adicionei pra minha revisão.

     

  • Ela quer uma equivalência, e para acelerar na prova, faça direto: 

    Equivalência 1 de P->Q: (negar, negar e inverter) ~Q->~P

    ou

    Equivalência 2 de P->Q: (negar a primeira OU mantém a segunda) ~P OU Q

    Como a questão trabalha com símbolos matemáticos, troque por símbolos direfentes: (> e = por <)  (= por símbolo de diferente)

    Achando os símbolos diferentes, fica fácil de ver que tem o modo de equivalência 1 na assertiva A.

  • Aparentemente difícil, mas muito fácil. Não precisa disso tudo aí.

    Basta calcular: 16x7 = 112

    Vê-se que x.y é maior ou igual a 112. Então essa proposição é verdadeira.

    Assim:

    Para ser menor que 112, X vai ser diferente de 16 OU Y menor que 7.  ALTERNATIVA "A".

    As outras alternativas nem dão margem para olharmos, pois os sinais de maior que, menor que e igual estão totalmente desproporcionais a um resultado viável.

  • essa questão dava pra matar só olhando o conectivo "e", já que para negar o "e" precisa ser o "ou" e a única alternativa que tem o "ou" é a A

  • GABARITO A

    Calcular? Basta usar a CONTRAPOSITIVA !!!!

  • A banca botou as inequações só para assustar o candidato. :)

  • Nesse tipo de questão tenha em mente duas coisas

    1- ta pedindo uma equivalência

    2- a negação de  ≥ é < e a negação de = é  ≠

  • Se x∙y < 112 então x ≠ 16 ou y < 7.


ID
279451
Banca
CONSULPLAN
Órgão
Prefeitura de Itabaiana - SE
Ano
2010
Provas
Disciplina
Raciocínio Lógico
Assuntos

Sejam os conjuntos: A = { x ∈ N / 1 ≤ x < 5 }; B = { x ≤ Z /-3 < x ≤ 4 } e C = { x ∈ N / x ≤ 3 }  Pode-se concluir que:

Alternativas
Comentários
  • Na questão A= {xeN/ 1<= x <5} ou seja  = x pertence a numeros naturais tal que 1 menor igual a x que é menor que 5, assim 1,2,3,4;
    Na questão B= {xeZ/ -3 x <=4} ou seja  = x pertence a numeros inteiros tal que menos 3 x menor igual a 4, assim -2,-1,0,1,2,3-4;
    Na questão C= {xeN/ x<= 3} ou seja  = x pertence a numeros naturais tal x  menor igual a 3,  assim 0,1,2,3,;

    Analisando as questões temos a letra C: (A intersecção com B) União (B intersecção C) = A União com B

    A intersecção com B = 1,2,3,4
    B Intersecção com C = 0,1,2,3
    A União dos dois resulta em 0,1,2,3,4
    que é igual a
    A U C = 0,1,2,3,4

    Abraços


  • Fazendo a montagem dos conjuntos:
    A={0,1,2,3,4}
    B={-2,-1,0,1,2,3,4}
    C={0,1,2,3}

    Fazendo primeiramente a união:
    AUB=>B ---- A={0,1,2,3,4} U B= {-2,-1,0,1,2,3,4} => B={-2,-1,0,1,2,3,4}
    AUC=>A ---- A={0,1,2,3,4} U C={0,1,2,3} => A={0,1,2,3,4}
    BUC=>B --- B={-2,-1,0,1,2,3,4} U C={0,1,2,3} => B={-2,-1,0,1,2,3,4}

    A interseção:

    A e B=> A   A={0,1,2,3,4} e  B={-2,-1,0,1,2,3,4} =>   A={0,1,2,3,4}
    A e C => C  A={0,1,2,3,4} e C={0,1,2,3} =>  C={0,1,2,3}
    B e C => C B={-2,-1,0,1,2,3,4} e C={0,1,2,3} =>  C={0,1,2,3}

    * desculpem-me o uso do "e"

    Fazendo as resoluções das questôes:

    a) (A U B) e (B U C)= B e B, então A e C está errado
    b) (A e C) U ( B e C) = C U C, então C U B está errado
    c) (A e B) U ( B e C) = A U C, então a alternativa está correta
    d) (A U C) e (B U C) = A e B, então B e C está errado
    e) (A e B) e ( B e C) = A e C, então A e B está errado

    Logo, a alternativa correta é a letra c)
  • Naturais se restringe aos positivos!!

  • Shyrleide Martins, seu raciocínio está correto, mas atente para o fato de que o conjunto A= { 1, 2, 3, 4}, visto que no enunciado da questão é informado que 1 é menor ou igual a x, então o conjunto A não contém 0.

  • Questão fácil, porém trabalhosa. 

  • Nossa Senhora, que sono essa professora comentando a questao kkkk Desisto

  • questao completamente desgraçada. Pra que o infeliz do avaliador faz uma coisa assim? É um infeliz!

  • {XEN/1<=X<5}= Se X é maior ou igual a 01, o Zero entra no conjunto?

  • Trabalhosa, mas não impossível! UFFA!

  • Deu trabalho, mas valeu a pena.

    GABARITO "C"

  •  

    Cuidado. Não pode ter 0 no conjunto A, tal como informado pela Shyrleide Martins.


ID
370618
Banca
FCC
Órgão
TCE-GO
Ano
2009
Provas
Disciplina
Raciocínio Lógico
Assuntos

São dadas as afirmações:

– Toda cobra é um réptil.

– Existem répteis venenosos.

Se as duas afirmações são verdadeiras, então, com certeza, também é verdade que

Alternativas
Comentários
  • Gab: E

    Segundo a questão, é verdadeiro que toda cobra é um réptil. Logo, sendo venenosa ou não, ela será mesmo um réptil

  • DICA: não resolvam questões assim de acordo com os conhecimentos de biologia, pois o que se quer de fato é o raciocínio lógico! E nem sempre coincidem :)

  • Alguém pode explicar?

  • É DE DENTRO PARA FORA, OU SEJA, É MAIS VÁLIDO DIZER QUE EXISTE UMA COBRA VENENOSA E QUE É RÉPTIL DO QUE DIZER QUE SE UM ÉPTIL É VENENOSO ENTÃO ELE É UMA COBRA.


ID
590716
Banca
FDRH
Órgão
IGP-RS
Ano
2008
Provas
Disciplina
Raciocínio Lógico
Assuntos

Considere os argumentos abaixo:
I – Todos os gatos são pretos.
Alguns animais pretos mordem.
Logo, alguns gatos mordem.

II – Se 11 é um número primo, então, 8 não é um número par.
Ora 8 é um número par, portanto, 11 não é um número primo.

III – Todos os X são Y.
Todos os Z são Y.
Alguns X estão quebrados.
Logo, alguns Y estão quebrados.
Quais são válidos?

Alternativas
Comentários
  • I - INVÁLIDO - O GRUPO DE GATOS (que são todos pretos) PODE PERTENCER SOMENTE AO GRUPO DE ANIMAIS QUE MORDEM, OU PERTENCER SOMENTE AO GRUPO DE ANIMAIS QUE NÃO MORDEM, OOOU AINDA DIVIDIDOS EM METADE E MATADE, ISTO É, NÃO SE PODE CONCLUIR QUE ALGUNS GATOS MODEM, POIS PODE SER QUE NENHUM DELES MORDEM, OU ATÉ ENTÃO TOOODOS MORDEM.


    II - VÁLIDO - 11Pr --> ~8Pa, ORAS SE 8 É PAR, LOGO A SEGUNDA PARTE DE PROPOSIÇÃO SERÁ FALSA. E PARA QUE A PREMISSA SEJA VERDADEIRA OBRIGATORIAMENTE A PRIMEIRA PARTE TEM QUE SER FALSA, ISTO É, 11 NÃO É UM NÚMERO PRIMO. 


    III - VÁLIDO - SE ''X'' É QUEBRADO DENTRO DO GRUPO DE ''Y'', É ÓBVIO E ULULANTE QUE ELE SERÁ QUEBRADO COM O GRUPO DE ''Z''. NÃO EXISTE OUTRO GRUPO COM QUE ELE PODERÁ SER QUEBRADO, ISTO É, ALGUNS ''Y'' ESTÃO QUEBRADOS.



    GABARITO ''D''


ID
595237
Banca
FCC
Órgão
Prefeitura de São Paulo - SP
Ano
2007
Provas
Disciplina
Raciocínio Lógico
Assuntos

Considerando os Auditores-Fiscais que, certo mês, estiveram envolvidos no planejamento das atividades de fiscalização de contribuintes, arrecadação e cobrança de impostos, observou-se que:

- todos os que planejaram a arrecadação de impostos também planejaram a fiscalização de contribuintes;

- alguns, que planejaram a cobrança de impostos, também planejaram a fiscalização de contribuintes.

Com base nas observações feitas, é correto afirmar que, com certeza,

Alternativas
Comentários
  • Gab: B se algum Auditor-fiscal esteve envolvido nos planejamentos da arrecadação e da cobrança de impostos, então ele também planejou a fiscalização de contribuintes.

  • Podemos definir 3 grupos de Auditores-fiscais: Arrecadação, Fiscalização e Cobrança. Com o auxílio destes conjuntos, vamos interpretar as informações dadas:

    − todos os que planejaram a arrecadação de impostos também planejaram a fiscalização de contribuintes;

    Esta informação nos diz que todos os membros do conjunto Arrecadação também são membros do conjunto Fiscalização, isto é, Arrecadação está contido em Fiscalização:

    − alguns, que planejaram a cobrança de impostos, também planejaram a fiscalização de contribuintes.

    Aqui vemos que existem elementos na intersecção entre o conjunto Cobrança e o conjunto Fiscalização:

    Atenção para um detalhe: temos certeza que existem elementos nas regiões 1 ou 2 acima (pois há fiscais que planejaram cobrança e fiscalização). Mas não temos certeza se estes elementos estão apenas na região 1, apenas em 2 ou em 1 e 2. Nada foi dito sobre a intersecção entre Arrecadação e Cobrança.

    Com este diagrama em mãos, vamos analisar as alternativas:

    (A) todo Auditor-fiscal que planejou a fiscalização de contribuintes esteve envolvido no planejamento da arrecadação de impostos.

    Falso. Arrecadação está contido em Fiscalização, e não o contrário.

    (B) se algum Auditor-fiscal esteve envolvido nos planejamentos da arrecadação e da cobrança de impostos, então ele também planejou a fiscalização de contribuintes.

    Verdadeiro. Este Auditor-fiscal estaria na região 2 do gráfico acima (intersecção entre Arrecadação e Cobrança), e consequentemente estaria dentro do conjunto Fiscalização.

    (C) existe um Auditor-fiscal que esteve envolvido tanto no planejamento da arrecadação de impostos como no da cobrança dos mesmos.

    Falso. Não temos elementos para afirmar que existem elementos na região 2 (Arrecadação e Cobrança), como vimos acima.

    (D) existem Auditores-fiscais que estiveram envolvidos no planejamento da arrecadação de impostos e não no da fiscalização de contribuintes.

    Falso. Arrecadação está contido em Fiscalização.

    (E) pelo menos um Auditor-fiscal que esteve envolvido no planejamento da cobrança de impostos também planejou a arrecadação dos mesmos.

    Falso. Pode ser que a intersecção entre Cobrança e Fiscalização se encontre toda na região 1, não havendo elementos na região 2 (que seria a intersecção com Arrecadação).

    Resposta: B

  • Única possibilidade é a letra B!

  • Desenhando os conjuntos fica bem fácil.


ID
795280
Banca
FCC
Órgão
TST
Ano
2012
Provas
Disciplina
Raciocínio Lógico
Assuntos

A declaração abaixo foi feita pelo gerente de recursos humanos da empresa X durante uma feira de recrutamento em uma faculdade:


“Todo funcionário de nossa empresa possui plano de saúde e ganha mais de R$ 3.000,00 por mês.”
Mais tarde, consultando seus arquivos, o diretor percebeu que havia se enganado em sua declaração. Dessa forma, conclui-se que, necessariamente,

Alternativas
Comentários
  • discordo do gabarito, pois pode haver mais de um funcionário na situação descrita na letra C, invalidando a questão.
  • Concordo, a questão foi mal redigida, o correto seria "pelo menos um (ou algum) funcionário não tem plano de saúde ou ganha até R$ 3.000".
  • A declaração dizia:

    Todo funcionário de nossa empresa possui plano de saúde e ganha mais de R$ 3.000,00 por mês.”

    Porém, o diretor percebeu que havia se enganado, portanto, basta que 1 funcionário não tenha plano de saúde ou ganhe até R$ 3.000,00 para invalidar, negar a declaração, tornando-a desse modo FALSA.

    Logo, necessariamente
    um funcionário da empresa X não tem plano de saúde ou ganha até R$ 3.000,00 por mês.
  • tendo em vista que a negação do todo seria "algum não", eu não entendo essa banca...
  • Pessoal, a questão não disse que é  um funcionário que se encaixa nessa condição, excluindo qualquer outro. Ela disse que há um funcionário, necessariamente, nessa condição... Por isso, acho que o gabarito está correto sim, na falta de algo melhor.
  • ALTERNATIVA C) GABARITO CERTO

    Proposição composta no conectivo "e" "Todo funcionário de nossa empresa possui plano de saúde e ganha mais de R$ 3.000,00 por mês"
    logo< basta que uma das proposições ser falsa para a declaração ser falsa

    1ª Proposição: Todo funcionário de nossa empresa possui plano de saúde
    2ª Proposição: ganha mais de R$ 3.000,00 por mês



    Lembre-se que no enunciado não fala onde foi o erro da declaração do gerente, ou seja, pode ser na primeira proposição e não na segunda ou na segunda e não na primeira ou nas duas que o resultado será falso. 

    Na alternativa C a banca fez a negação da primeira proposição e vez a da segunda e ligou elas no conectivo "OU" 
    POIS no conectivo "OU" TANTO FAZ A PRIMEIRA SER VERDADEIRA OU A SEGUNDA SER VERDADEIRA, DESDE QUE HAJA UM V. PARA O RESULTADO SER V.

    Atenção: A alternativa e) estar igualzinha só muda o conectivo que é o E, que obrigaria que o erro da declaração fosse nas duas.


    Bons estudos!
  • TENHO UMA DÚVIDA!!!

    Qdo saberei que a questão está se referindo à negação de proposição e não à negação de quantificador lógico?

    Abraço!!!
  • Todo               funcionário de nossa empresa possui plano de saúde
      ∀                                           P
     e                   ganha mais de R$ 3.000,00 por mês.
                                                    q

    NEGAÇÃO:

    ¬ (∀) = ? ¬P                   ? = E (INVERTIDO)
        
    ¬ ∧  = E (INVERTIDO)

    = TODO (A), NENHUM, NINGUÉM, CADA, PARA CADA.
     ? = NEM TODO, ALGUM ALGUÉM, AO MENOS UM, PELO MENOS UM, EXISTE UM.   
                                                      
    ∀  P ? q

    NEGAÇÃO : ?¬P    ?    ¬ q
     
    c)    um funcionário da empresa X não tem  PLANO DE SAÚDE  ou    ganha até R$ 3.000,00 por mês.  (VERDADEIRA)
            ?                                                  ¬                       P            ?                  ¬ q
     
    d)   nenhum funcionário da empresa X  TEM PLANO DE SAÚDE      ou       todos         ganham até R$ 3.000,00 por mês.
                  ∀                                                                P                    V          ∀                                       ¬ q
     
  • LETRA C
    A questão pede a negação da afirmação:
    TODO FUNCIONÁRIO DE NOSSA EMPRESA POSSUI PLANO DE SAÚDE E GANHA MAIS DE R$ 3000,00 POR MÊS. Essa fica assim ~(p^q). A negação dela ~pv~q
    ~(p^q) <=> ~pv~q (negação todas e vira ou)
    A 1a proposição tem um TODO  que é quantificador universal, para nega-lo utilizamos um quantificador existencial. Pode ser: um, existe um, pelo menos, existem..
    No caso da qusstão ficou assim:
    UM FUNCIONÁRIO DA EMPRESA NÃO POSSUI PLANO DE SAÚDE OU GANHA ATÉ R$ 300,00 POR MÊS. A negação de ganha mais de 3000 mil por mês, é GANHA ATÉ 3000 MIL.
    Veja o vídeo da resolução no site: http://matematicaparticular.com/questao-18-tecnico-judiciario-tribunal-superior-do-trabalho/
  • De forma resumida:

    Todo funcionário de nossa empresa possui plano de saúde e ganha mais de R$ 3.000,00 por mês.”
    TODO                                                                         E                   Q

    Se o gerente se enganou (A questão está pedindo a NEGAÇÃO!!!!)

    TODO (P e Q)    tem como negação       PELO MENOS UM (~P ou ~Q)



    Ou Seja: LETRA C é realmente o gabarito!!!!!

  • Compartilho minhas anotações, as quais usei para responder a questão:
    Negação do quantificador "TODO"

    --- algum + negação
    --- existe + negação
    --- pelo menos um + negação

    Negação da conjunção  "E" = ~p v ~q        [ nega a primeira, coloca o "OU" e nega a segunda]

    Negação da Proposição:
    TODO funcionário de nossa empresa possui plano de saúde E ganha mais de R$ 3.000,00 por mês.”

    Dessa forma é só negar o TODO, acrescentar o OU e negar a segunda proposição.
    Analisando as opções as únicas que poderiam ser seriam a "a" e a "c", mas a "a" mantém a conjunção "e" o que a torna errada.

    c) um funcionário da empresa(a negação é "pelo menos um", então dizer que um funcionário da empresa não tem plano de saúde já nega a primeira proposição.) X não tem plano de saúde ou ganha até R$ 3.000,00 por mês(colocou o "ou". Ou seja se é verdade dizer que "[...] ganha mais que R$ 3.000,00, por lógica, quando diz que ganha até R$ 3.000,00 estará negando).
  • o professor bruno villar deu como correta a letra a
  • Todo >Negação<Alguém (pelo menos um)>Negação<Nenhum
  • Pessoal, acredito que a Jeane tem razão, mas concordo que o ideal seria "Pelo menos um", mas se vocês olharem as outras alternativas, essa com certeza é a menos absurda. Não tem como reclamar, uma das 5 responde a questão, basta procurar a menos errada.
  • Negação de proposições compostas:

    Negação de conjunção:        Resultado:

    ~(p ^ q)                                     ~p v ~q

    Negação  de "Todo funcionário de nossa empresa possui plano de saúde e ganha mais de R$ 3.000,00 por mês" é:

    "Existe pelo menos um
     funcionário de nossa empresa que não possui plano de saúde ou que ganha no máximo (até) R$ 3.000,00 por mês"

    Basta que um funcionário não tenha plano para negar.
  • PESSOAL!

    A BANCA CESPE ARGUMENTA QUE AS DISJUNÇÕES "OU" e "OU...OU" SÃO IGUAIS. POR ISSO, A GENTE SABE QUE A LETRA C ESTÁ ERRADA, POIS PRECISARIA DA DIJUNÇÃO OU...OU PARA NEGAR A SENTENÇA, MAS FAZER O QUÊ? SE A CESPE ACHA QUE É CERTO ASSIM, ENTÃO VAMOS TER QUE ACEITAR.
  • NEGAÇÃO DA DISJUNÇÃO:
    ¬(PvQ)= ¬P^¬Q

    NEGAÇÃO DA CONJUNÇÃO:
    ¬(P^Q)= ¬Pv¬Q

    NEGAÇÃO DA CONDICIONAL:
    ¬(P→Q)= P^¬Q

    NEGAÇÃO DA BICONDICIONAL:
    ¬(P↔Q)= P↔¬Q= ¬P↔Q
  • Todas as alternativas estão claramente erradas.
    A repostas apontada pelo gabarito é a menos errada. O correto seria dizer que "pelo menos um" ou "alguém" não se enquadra no que foi dito.
    Mas a FCC nunca admite seus erros...
  • Muito simples: Para negar o TODO basta apenas (pelo menos um)

    Para ele descobrir que sua afirmação foi errônea bastou saber de um funcionário que não tinha plano de saúde e ganhava mais de 3.000.
    Não necessariamente ALGUNS ou ALGUM grupo, bastou saber de um e pronto, errou a afirmação!
  • Com certeza Fernando, a falta de um desses termos que  você descreveu ( "pelo menos um" ou "algum") torna invalidada a questão. Concordo que a "C" é o que a banca buscou empregar como gabarito e, para ser sincero, o que eu marcaria se fosse candidato ao cargo no dia da prova, porém, também está errada a afirmativa. Não tem como afirmar(NECESSARIAMENTE) conforme enunciado que UM funcionário da empresa... pois sabemos que pode ser mais de UM. O correto seria PELO MENOS UM... e ponto final, kkkkkkkkk

    Abraço e...

    Bons Estudos

  • Todo funcionário de nossa empresa possui plano de saúde e ganha mais de R$ 3.000,00 por mês

     

    Para negar o TODO há três opções (PEA + não):

    - Pelo menos um funcionário de nossa empresa não possui plano de saúde ou não ganha mais de R$3.000,00 por mês: 1ª opção troca-se o "todo" por "pelo menos um", nega o verbo (possui). Porém a frase é ligada pelo conectivo E e sua negação é negar tudo e trocar pelo OU. O antecedente já foi negado, restando negar o verbo do consequente também (não ganha).

     

    - Existe um funcionário de nossa empresa que não possui plano de saúde ou não ganha mais de R$3.000,00 por mês: 1ª opção troca-se o "todo" por "existe", nega o verbo (possui). Porém a frase é ligada pelo conectivo E e sua negação é negar tudo e trocar pelo OU. O antecedente já foi negado, restando negar o verbo do consequente também (não ganha).

     

    - Algum funcionário de nossa empresa não possui plano de saúde ou não ganha mais de R$3.000,00 por mês: 1ª opção troca-se o "todo" por "algum", nega o verbo (possui). Porém a frase é ligada pelo conectivo E e sua negação é negar tudo e trocar pelo OU. O antecedente já foi negado, restando negar o verbo do consequente também (não ganha).

     

    Nenhuma das alternativas encaixa, logo sem gabarito. 

  • Tipo de questão que você deve escolher a menos errada...

     

    Negação do Todo é o pelo menos um (a alternativa C é a única que contém +/- essa expressão)

    A negação de uma conjunção (e) é a disjunção (ou), com a negação de ambas as proposições.

     

    C - um funcionário não tem plano de saúde e não ganha mais de R$3mil.

     

    A diferença está em:

    "não ganha mais de R$3mil" 

    e

    "ganha até R$3mil" 

    ...que analisando, dá na mesma coisa.

     

    Exemplificando:

    não ganha mais de R$3.000,00 = ganha R$3.000,00, pois mais de R$3.000,00 seria R$3.001,00.

  • Todo A é B e C

     

    Negação de Todo A  é B e C: algum A não é B ou não é C Um [pelo menos um] A não é B ou não é C

     

    Todo A é B: Todo funcionário tem plano de saúde

    Negação: Algum funcionário não tem plano de saúde (ou, no caso, um funcionário não tem plano de saúde)

     

    Negação do eou

     

    Todo A é C: Todo funcionário ganha mais de 3k por mês

    Negação: Algum funcionário não ganha mais de 3k por mês (ou, no caso, um funcionário ganha, no máximo, 3k por mês)

     

  • Tô contigo Larissa, mas me parece que ela fez uma miscelânea incompreensível.

  • Vai fazer uma questão atual da FCC e marcar isso daí de UM pra ver... questão sem resposta!

  • Um pode ser numeral ou artigo indefinido. Se o candidato conseguir enxergar que a banca utilizou o "um" como artigo indefinido, passando a ter o sentido de "algum", e não "1" (numeral), a letra C faz todo sentido. O problema é o candidato ter que literalmente ADIVINHAR isso.


ID
1066396
Banca
FCC
Órgão
SEFAZ-RJ
Ano
2014
Provas
Disciplina
Raciocínio Lógico
Assuntos

Suponha que sejam verdadeiras as seguintes informações:

I. Todos os empregados da empresa Alfa são competentes.

II. Mário não trabalha na empresa Alfa.

III. André é competente.

IV. Alguns empregados da empresa Alfa são estudantes.

Então, é correto afirmar que :

Alternativas
Comentários
  • "(A) todos os estudantes são competentes. 

     ERRADO. Não temos informações para chegar em uma conclusão como esta. 

    (B) existe pelo menos um estudante que é competente. 

     CORRETO. Sabemos que alguns empregados de Alfa são estudantes. Como todos os empregados de Alfa são competentes, então certamente esses estudantes (que são empregados de Alfa) são competentes. Veja o “x” que assinalei no diagrama abaixo. Nesta região estão estudantes que são empregados de Alfa e, por consequência, são competentes: 

    (C) André trabalha na empresa Alfa. 

     ERRADO. Só sabemos que André é competente, de modo que certamente ele está dentro do conjunto vermelho. Mas isso não faz com que ele obrigatoriamente esteja no conjunto verde (empregados de Alfa). 

    (D) Mário não é competente. 

     ERRADO. Como Mário não trabalha na empresa Alfa, ele certamente está fora do conjunto verde. Mas ainda assim pode ser que ele faça parte do conjunto vermelho (dos competentes). 

    (E) existe pelo menos um estudante que não trabalha na empresa Alfa. 

     ERRADO. Não temos elementos para essa afirmação. Só sabemos que alguns empregados de Alfa são estudantes. Pode ser até que TODOS os estudantes sejam empregados de Alfa. "

    Fonte: prof. Arthur Lima

    Resposta: B 


  • Se a letra A fosse: "todos os estudantes DA EMPRESA ALFA são competentes"  Nesse caso estaria certo a letra A?

  • Se a alternativa correta disse "algunS", no plural, não significa que MAIS DE UM é competente? Errei por isso, pois "pelo menos um" seria equivalente a ALGUM. Alguns (no plural) já sugere que pelo menos 2 empregados são estudantes. Concordam? 

     

    Eduardo Frazão, sim, estaria correta nesse caso, pois todos os funcionários, ESTUDANTES ou não, são competentes.

  • Façam o diagrama que facilitará a vida!


ID
1112710
Banca
FCC
Órgão
TRF - 3ª REGIÃO
Ano
2014
Provas
Disciplina
Raciocínio Lógico
Assuntos

Diante, apenas, das premissas “Nenhum piloto é médico”, “Nenhum poeta é médico” e “Todos os astronautas são pilotos”, então é correto afirmar que :

Alternativas
Comentários
  • O melhor é desenhar os conjuntos no caderno, mas vamos lá:

    Nenhum piloto é médico: não há intersecção entre o conjunto dos pilotos e dos médicos;
    Nenhum poeta é médico: não há intersecção entre o conjunto dos poetas e dos médicos;
    Todos os astronautas são pilotos: O conjunto dos astronautas está dentro do dos pilotos.

    a) algum astronauta é médico.
    Falsa, se o conjunto dos astronautas está dentro do dos pilotos e nenhum piloto é médico, podemos afirmar que nenhum astronauta é médico.

    b) todo poeta é astronauta.
    A questão não estabelece relação entre poetas e astronautas e pilotos, esta afirmativa é inconclusiva.

    c) nenhum astronauta é médico.
    Verdadeira, o conjunto dos astronautas está dentro do dos pilotos e nenhum piloto é médico, então nenhum astronauta é médico.

    d) algum poeta não é astronauta.
    Afirmativa inconclusiva, uma vez que nada se fala sobre a relação entre poetas e astronautas e pilotos.

    e) algum poeta é astronauta e algum piloto não é médico.
    A primeira parte da afirmativa é inconclusiva, a explicação é a mesma da alternativa d. A segunda parte é verdadeira, pois o pilotos não são médicos.

    Bons estudos, Elton.

  • Se todo austronauta é piloto, e nenhum piloto é médico...então nenhum astronauta é medico. FAZENDO OS CONJUNTOS DÁ CERTIM.

     

    GABARITO ''C''

  • Explicação :

    https://www.youtube.com/watch?v=_7b-FjvOChk

    8:30


ID
1120075
Banca
FCC
Órgão
TRT - 2ª REGIÃO (SP)
Ano
2014
Provas
Disciplina
Raciocínio Lógico
Assuntos

Um dia antes da reunião anual com os responsáveis por todas as franquias de uma cadeia de lanchonetes, o diretor comercial recebeu um relatório contendo a seguinte informação:

Todas as franquias enviaram o balanço anual e nenhuma delas teve prejuízo neste ano.

Minutos antes da reunião, porém, ele recebeu uma mensagem em seu celular enviada pelo gerente que elaborou o relatório, relatando que a informação não estava correta. Dessa forma, o diretor pôde concluir que, necessariamente,

Alternativas
Comentários
  • Questão simples mais exige atenção do candidato.

    Negação de uma conjunção, com equivalência entre "nenhum" e "todo" 

    Na negação onde tem: TODO toca-se por ALGUM (pelo menos um)

                                          ALGUM (pelo menos um) toca-sr por NENHUM

    agora só fazer a negação da condicional (nega as duas e troca o "E" pelo "OU").

     nem todas (pelo menos uma) as franquias enviaram o balanço anual ou pelo menos uma delas teve prejuízo neste ano.

    Espero que alguém entenda.

  • não temos uma equivalência e da mesma forma não temos uma relação de negação (na verdade o problema tenta passar que seria uma relação de equivalencia para ser transformada numa relação de negação), ora, o que temos são duas proposições P e Q que precisam ser negadas ~ (p ^ q) para ~ p ou ~ q, simples assim, por isso fica: Nem todos ..., OU pelo menos uma teve prejuízo neste ano. veja que somente faz a negação antes dos termos iniciais de cada frase. simples assim, não complique.

    abraços.

     

  • Todas as franquias enviaram o balanço anual       ^      nenhuma delas teve prejuízo neste ano. 



    1º - lógica de primeira ordem:

    ALGUMAS FRANQUIAS NÃO ENVIARAM O BALANÇO ANUAL      ^      ALGUMAS DELAS TIVERAM PREJUÍZOS ESTE ANO.



    2º - Lei  de morgan:

    NEM TODAS AS FRANQUIAS ENVIARAM O BALANÇO ANUAL      v      PELO MENOS UMA DELAS TEVE PREJUÍZO NESTE ANO. 
    ALGUMAS...NÃO                                                                                         ALGUMA / EXISTE UMA           


    GABARITO ''A''
  • http://profmilton.blogspot.com.br/2015_05_01_archive.html

  • Dica

     

    TODO= PEA +  NÃO

    Pelo menos uma  das franquias não enviaram o balanço anual.

    Existe uma franquia que não enviou o balanço anual.

    Alguma franquia não enviou o balanço anual.

    ===============================

    "nem todas as franquias enviaram o balanço anual" ( Nem = pelo menos uma + não)

  • Todas as franquias enviaram o balanço anual e nenhuma delas teve prejuízo neste ano.
    É pra negar a proposição:

    a) nem todas as franquias enviaram o balanço anual ou pelo menos uma delas teve prejuízo neste ano.

    Negação de TODO É = Algum/pelo menos um/existe um que não é
    Negação de NENHUM = Algum/pelo menos um/existe um que é

  • Nei lins, mas exige, exige mais atenção mesmo...

  • TODO não se nega com TODO nem com NENHUM!

    NENHUM não se nega com NENHUM nem com TODO!

    Abraços!

  • Se a conjunção “Todas as franquias enviaram o balanço anual E nenhuma delas teve prejuízo neste ano” é FALSA, podemos concluir que a sua negação é verdadeira. Esta negação é:

    “Nem todas as franquias enviaram o balanço anual OU alguma delas teve prejuízo neste ano”

    Temos uma variação disto na alternativa E.

    Resposta: E


ID
1130068
Banca
FCC
Órgão
PM-BA
Ano
2012
Provas
Disciplina
Raciocínio Lógico
Assuntos

Há um grupo de 13 meninos. Alguém diz: "Todos esses meninos têm 13 anos de idade.” Para negar essa afirmação, o número mínimo de meninos que não pode ter 13 anos de idade é:

Alternativas
Comentários
  • negação de TODO A é B = PELO MENOS 1 A não é B

    Resposta: letra A


  • A negação de que TODO A é B = PELO MENOS 1 A não é B então a resposta é letra=A




  • Resolvo essa questão aqui nesse vídeo

    https://youtu.be/yBwdPQJyqR8

    Ou procure por "Professor em Casa - Felipe Cardoso" no YouTube =D


ID
1137046
Banca
FUNDAÇÃO SOUSÂNDRADE
Órgão
CBM-GO
Ano
2010
Provas
Disciplina
Raciocínio Lógico
Assuntos

Sabe-se que em uma cidade, todos os Filósofos são Professores, mas não existem Professores que sejam Cantores. Sabe-se, também, que pelo menos um Músico é Filósofo. Pode-se concluir que nessa cidade

Alternativas
Comentários
  • e ai galera, me expliquem ai por favor

  • fazendo por diagramas fica mais fácil!!!!

     

  • MÚSICO NÃO QUER DIZER QUE PODE SER CANTOR ... PODE SER BATERISTA, GUITARRISTA

    JÁ CANTOR SE INCLUI NO ESPECÍFICO (MÚSICO) TODO CANTOR É MÚSICO

    ALGUM MÚSICO NÃO É CANTOR!!

  • na minha visão a questão está mal formulada. pois pra mim não é obvio que o grupo cantor está inserido no grupo de músicos. Poderiam afirmar isso.

    assim fiz um grupo separado, acabei errando a questão.


ID
1199950
Banca
COPEVE-UFAL
Órgão
UFAL
Ano
2014
Provas
Disciplina
Raciocínio Lógico
Assuntos

Considere a assertiva: Se um apartamento está acima do quarto andar, então ele tem sacada.

Essa afirmativa pode ser deduzida, do ponto de vista lógico, de qual das alternativas abaixo?

Alternativas
Comentários
  • Fiz um vídeo com a resolução dessa questão, não deixem de ver.


    Link para o vídeo com a resolução dessa questão: https://youtu.be/xyXX2ffrrAM


    Professor Ivan Chagas

     

    Muito mais vídeos com questões resolvidas em www.gurudamatematica.com.br

     

  • São expressões equivalentes da condicional "Se um apartamento está acima do quarto andar, então ele tem sacada":

     

     

    1) SE um apartamento está acima do quarto andar, ENTÃO ele tem sacada. 

     

    2) O apartamento tem sacada, SE estiver acima do quarto andar. 

     

    3) QUANDO um apartamento está acima do quarto andar, ele tem sacada. 

     

    4) Um apartamento estar acima do quarto andar IMPLICA ele ter sacada. 

     

    5) Um apartamento estar acima do quarto andar É CONDIÇÃO SUFICIENTE para que ele tenha sacada. 

     

    6) Ter sacada É CONDIÇÃO NECESSÁRIA para que um apartamento esteja acima do quarto andar.

     

    7) Um apartamento está acima do quarto andar SOMENTE SE ele tiver sacada. 

     

    8) TODO apartamento que está acima do quarto andar tem sacada. (gabarito da questão)

  • A questão só nos garante que os apartamenos ACIMA do quarto andar têm sacada:

    a) Nenhum apartamento no quarto andar tem sacada. = Pode haver, mas o enunciado não diz.

     

    b) Todo apartamento acima do quarto andar tem sacada. Essa é a única conclusão que podemos tirar do enunciado, principalmente porque ele não fala sobre os apartamentos que estão abaixo.

     

    c) Se um apartamento tem sacada, então ele está acima do quarto andar. = Pode haver apartamento abaixo do quarto andar que possuam, mas o enunciado não diz.

     

    d) Todas as sacadas somente são construídas para apartamentos acima do quarto andar. = Não podemos concluir isso, pois pode haver um ou outro apartamento abaixo do quarto andar com sacada, mas o enunciado não diz.

     

    e) Sacadas não são construídas abaixo do quarto andar. = O enunciado não fala sobre os apartamentos que estão abaixo do quarto andar, portanto não temos certeza sobre eles.

  • Eu achei que eu teria que negar a proposição dada na questão!

    ''Se um apartamento está acima do quarto andar, então ele tem sacada.'' Logo, seria: Se um apartamento está acima do quarto andar, então ele NÃO tem sacada.

    Porém, percebi no enunciado que eles pedem uma DEDUÇÃO, invếs de uma negação. :/

     

  • A questão não está pedindo nem equivalência e nem negação!

    OBSERVE:

    Considere a assertiva: Se um apartamento está acima do quarto andar, então ele tem sacada.

    Essa afirmativa pode ser deduzida, do ponto de vista lógico, de qual das alternativas abaixo?

    A questão quer saber se o candidato sabe diferenciar os TIPOS DE ARGUMENTOS!

    Tipos de argumentos:

    Dedução o argumento dedutivo parte de situações GERAIS para situações particulares. Ex:

    P: Todo professor é aluno

    Q: Daniel é professor

    C: Logo, Daniel é aluno.

    Indução o argumento indutivo é o contrário do argumento dedutivo, pois parte de informações PARTICULARES para chegar a uma conclusão geral. Ex:

    P: Cerveja embriaga.

    Q: Uísque embriaga.

    R: Vodca embriaga.

    C: Portanto, toda bebida alcoólica embriaga.

    Analogia as analogias são comparações. Partindo de uma situação já conhecida verificamos outras desconhecidas, mas semelhantes. Ex:

    P: no Piauí faz calor

    Q: no Ceará faz calor

    R: no Paraná faz calor

    C: sendo assim, no Brasil faz calor.

    Tbm temos as Falácias e os Silogismos! Pesquisem

    Logo a única opção que parte de uma situação geral é a B!

    Argumento: Todo apartamento (situação geral) acima do quarto andar tem sacada.

    Conclusão: Se um apartamento (conclusão particular) está acima do quarto andar, então ele tem sacada.


ID
1219252
Banca
IBFC
Órgão
SEDS-MG
Ano
2014
Provas
Disciplina
Raciocínio Lógico
Assuntos

Todo mafagáfo é um guilherdo e todo guilherdo é um rosmedo. Desse modo, é correto afrmar que:

Alternativas
Comentários
  • Gabarito D


    Fazendo diagrama fica fácil visualizar (não consegui inserir figura).

    3 círculos, um dentro do outro. O maior (externo) está rosmedo, no intermediário guilherdo e no menor o mafagafo. Assim, fica fácil encontrar a resposta

  • Pode ser que não dê certo todas as vezes, mas eu substituí os seres, tipo. 

    Todo cão é um canino e todo canino é um animal. 

    a) Há cão que não é animal - errado

    b) Todo canino é cão - errado

    c) Nenhum animal é cão - errado

    d) Alguns caninos podem ser cães - certo


  • Pra quem além de RL for estudar tb português: lembre-se "mafagáfo" não é acentuada.

  • GABARITO. D.

    BEM INTERESSANTE J.  VLW

  •                                    
    (  ((mafagafoguilerdo )  rosmedo)
                  |                  |                  \-> contêm todos os mafagafos e todos o guilerdos
                  |                   \-> contêm todos os mafagafos
                   \-> contido no guilherdo

    Com o gráfico fica mais fácil de resolver

  • GABARITO: d

    {  R  [   G   (   M  )    ]    }


    ;  ]

  • O ideal  seria  utilizar os Diagramas de Ven Euller, uma pena não podermos inserir figuras aqui!

    São três conjuntos conforme a ordem, mafagáfo está contido em guilherdo, os dois contidos em rosmedo.

  • mas gente se todo mafagafo é um guilherdo e todo guilherdo é um rosmedo, existe guilherdo que é marfagafo, então existe rosmedo marfagafo tbm!

  • o próprio gráfico mostra isso ! se estão os primeiros inclusos no maior, então objetos dos primeiros são também o maior, isso é obvio. Esse gabarito ta errado!


  • Para resolver esta questão, basta fazer o diagrama abaixo:


    Logo percebemos que não há Mafagafo que não é Rosmedo, que nem todo Guilherdo é Mafagafo, que existem Rosmedos que são Mafagafos e finalmente constatamos que alguns Guilherdos podem ser Mafagafos.


    RESPOSTA: (D)

  • Para a lógica, o "algum" pode ser o "todo", que foi o caso dessa questão.

  • coloque tudo no diagrama que vc encontra a resposta

  • Acertei dessa forma:


    Todo A é B

    Todo B é C

    Logo alguns B podem ser A.

  • NEGAÇÃO DO TODO É ALGUM e a unica alternativa que tem o P E A + não é a "d"

  • (         (        (M)     G)       R)

    Todo mafagáfo é um guilherdo e todo guilherdo é um rosmedo. Desse modo, é correto afrmar que:

    Alguns guilherdos podem ser mafagáfos.

  • olhe a resposta do video: https://www.youtube.com/watch?v=iadMldn7GsI

  • Negação:

    TODO=ALGUM

    NENHUM=PELO MENOS UM

  • Olá pessoal,

     

    Vejam o vídeo com a resolução dessa questão no link abaixo

    https://youtu.be/kLNMe0sXuZE

     

    Professor Ivan Chagas

    www.gurudamatematica.com.br

  • Substituindo os nomes por adjetivos pátrios, fica mais fácil raciocinar logicamente:

    Mafagafo - Manauara

    Guilherdo - Amazonense

    Rosmedo - Brasileiro

    Há mafagáfo que não é rosmedo. - Há manauara que não é Amazonense. - ERRADO!

    Todo guilherdo é mafagáfo. - Todo amazonense é manauara. - ERRADO!

    Nenhum rosmedo é mafagáfo. - Nenhum brasileiro é manauara. - ERRADO!

    Alguns guilherdos podem ser mafagáfos. - Alguns amazonenses podem ser manauaras. - CERTO!

  • {R [G (M )] }

    Todo M é G

    Algum G é M

    Todo G é R

    Algum R é G

    Algum R é M

    Gabarito "D"


ID
1242580
Banca
FGV
Órgão
TJ-AM
Ano
2013
Provas
Disciplina
Raciocínio Lógico
Assuntos

Em uma reunião de senhoras em um clube, Maria disse: —“Todas as pessoas casadas são felizes

Em seguida, Lucia retrucou: —“Isso não é verdade”. Considerando que Lucia tem razão, conclui-se logicamente que

Alternativas
Comentários
  • Contrariando o todas = pelo menos uma

    Logo, se pelo menos uma pessoa casada não for feliz a afirmação já está incorreta. 

    Gabarito: A.

  • Regra do Cruzamento:

    Todas são ------------ Nenhuma é

    X

    Alguma é ------------ Alguma não é

    Então fica:

    Todas são x Alguma não é

    Nenhuma é x Alguma é

    GABARITO LETRA A

  • NEGAÇÃO

     

                     TODO  ------------------->  ALGUM ... NÃO / PELO MENOS UM ... NÃO ... / EXISTE ... NÃO...

               NENHUM / NÃO EXISTE      ----------------------->  ALGUM 

                     ALGUM                ------------------------->   NENHUM / NÃO EXISTE

    ALGUM ... NÃO / PELO MENOS UM ... NÃO ... / EXISTE ... NÃO... --------->  TODO

  • GAB. A)

    pelo menos uma pessoa casada não é feliz.

  • Negação dos Quantificadores:

    • TODO troca por ==> ALGUM / PELO MENOS UM/ EXISTE + NEGA o verbo

    Ex:  “Todo aluno é inteligente" ALGUM aluno NÃO é inteligente / PELO MENOS UM aluno NÃO é inteligente/

    EXISTE aluno que NÃO é inteligente.


ID
1300585
Banca
FGV
Órgão
TJ-AM
Ano
2013
Provas
Disciplina
Raciocínio Lógico
Assuntos

Considere como verdadeiras as sentenças a seguir.

I. Alguns matemáticos são professores.
II. Nenhum físico é matemático.

Então, é necessariamente verdade que

Alternativas
Comentários

  • Letra d.

     

  • Por eliminação,chega-se a conclusão que:as letras(A) e (C) querem dizer a mesma coisa.assim como as letras (B) e (E).então só sobra a letra (D)

  • Pelo diagrama de Venn.


    Letra D. 

  • d) Algum professor nao é físico porque algum deles será matemático ( conforme diz na sentença I) 

  • Letra D. Tentei pela negativa das proposições, mas fica complicado. O diagrama de Venn resolve, mas o que é diagrama de Venn? Nada mais do que atribuir às frases a figura gráfica de conjuntos de elementos e suas respectivas interseções/contém/está contido. Assim, se temos três conjuntos, de professores, de matemáticos e de físicos, sabemos pelas duas afirmativas que há interseção entre professores e matemáticos (mas podem haver professores não matemáticos e matemáticos não professores) e não há interseção de físicos que também são matemáticos. Não podemos afirmar, pelos elementos dados na questão, nenhuma das respostas do gabarito, à exceção de que, ao menos um professor não é físico, porque será matemático. Fazendo graficamente os três conjuntos, fica fácil. O difícil é desenhar aqui! Rs...
  • Se nenhum físico é matemático (II), então os professores que são matemáticos (I) não são físicos.

    Letra D.

  • I.  Alguns matemáticos são professores.  (Pelo menos um professor é matemático)
    II.  Nenhum físico é matemático.    

    resposta:

     d) algum professor não é físico. ( Porque pelo menos um é matemático)

            

  • Resolvi pelo macete do Alberto Cobra

  • Comutativas (Algum A é B <==> Algum B é A), eliminadas alternativas A, C.

    Comutativas (Nenhum A é B <==> Nenhum B é A) eliminadas alternativas B, E.

    Gabarito Letra (D) --> Algum NÃO É B ==> NUNCA pode ser comutativa.

  • Se nenhum físico é matemático, o conjunto dos Físicos não tem intersecção com o dos Matemáticos. Se alguns matemáticos são professores, então há intersecção entre o conjunto dos Matemáticos e o conjunto dos Professores. Pode haver também intersecção entre os Professores e os Físicos, embora não tenhamos certeza disso com as informações dadas.

    Analisando as alternativas:

    (A) algum professor é físico. --> não podemos afirmar que há intersecção entre os Professores e os Físicos.

    (B) nenhum professor é físico. --> também não podemos afirmar que NÃO HÁ intersecção entre os Professores e os Físicos.

    (C) algum físico é professor. --> idem ao raciocínio do item A.

    (D) algum professor não é físico. --> os professores que são matemáticos certamente NÃO são físicos, pois nenhum físico é matemático. Assim, alguns professores (os matemáticos) não são físicos. Esse é o gabarito.

    (E) nenhum físico é professor. --> idem ao raciocínio do item B.

    Resposta: D 

  • GAB. D)

    algum professor não é físico.


ID
1300588
Banca
FGV
Órgão
TJ-AM
Ano
2013
Provas
Disciplina
Raciocínio Lógico
Assuntos

Se não é verdade que “Todos assistentes judiciários de determinado fórum são formados em advocacia”, então é necessariamente verdade que

Alternativas
Comentários
  • Como a frase apresentada é falsa, formando conjuntos, aparecem duas possibilidades:

    1) os conjuntos de "assistente judiciário" e o conjunto dos "formados em advocacia" serem totalmente independentes ou excludentes;
    2) haver intercecção entre eles;
    As opções A e B são iguais (devem ser eliminadas)
    A opção C não pode ser confirmada com as informações disponíveis
    As opções D e E podem ocorrer ao mesmo tempo (comprovamos com a possibilidade 2, é só desenhar os conjuntos)

  • ~(Todo A é B) = Algum A ñ é B. gabarito letra E.


  • Negação de (TODO) é (ALGUM, EXISTE UM)!!

  • A negação de Todo A é B  =  Algum A não é B.

     

  • A negação de TODO - ALGUM, EXISTE, PELOS MENOS UM + NEGAÇÃO DA FRASE.

  • negação do TODO: P E A + NÃO

    P E A : PELOS MENOS UM ; EXISTE; ALGUM,

    + NEGAÇÃO DA FRASE.

     

  • me deu uma agonia essa coisa de formado em advocacia kkkkk que curso é esse??

  • Para provarmos que quem disse a frase “Todos assistentes judiciários de determinado fórum são formados em advocacia” mentiu, basta encontrarmos algum assistente judiciário que NÃO é formado em advocacia. Assim, podemos dizer que:

    “Algum assistente judiciário NÃO é formado em advocacia”

    Resposta: E

  • Negação dos Quantificadores:

    • TODO troca por ==> ALGUM / PELO MENOS UM/ EXISTE + NEGA o verbo

    Ex:  “Todo aluno é inteligente" ALGUM aluno NÃO é inteligente / PELO MENOS UM aluno NÃO é inteligente/

    EXISTE aluno que NÃO é inteligente.


ID
1308310
Banca
CESPE / CEBRASPE
Órgão
SESA-ES
Ano
2013
Provas
Disciplina
Raciocínio Lógico
Assuntos

      Em alguns casos, o medicamento A tem os efeitos colaterais B. Sempre que são observados os efeitos colaterais B, o paciente apresenta os sintomas C.

Considerando verdadeiras as proposições apresentadas acima, assinale a opção correta.

Alternativas
Comentários
  • MEDICAMENTO A interseção EFEITO COLATERAL B (alguns que tomam os medicamentos a  sentem os efeitos B)

    Os SINTOMAS C podem aparecer, portanto, em três situações:

    1º) Na interseção 

    2º) Somente relacionado ao efeito colaral B 

    3º) Fora de todos os círculos

     

    Usando isso, podemos perceber que, por eliminação, a alternativa correta é a A.

  • Compreendo o porquê do item a está correto, mas não compreendi o porquê do item C está errado, alguém poderia ajudar?

  • sempre que se usa o medicamento B tem efeitos colaterais do medicamento C, mas qdo se usa o medicamento C nao tem os efeitos do medicamento B, por isso a opçao letra c esta errada

  • El Elyon, quem toma B com certeza vai apresentar os sintomas C, mas não necessariamente os sintomas C estão ligados somente ao B, podem ser do medicamente A ou de outro medicamento também.

  • A alternativa “B” está errada, pois a própria questão diz que somente em alguns casos em que o paciente tomar o remédio “A” terá os efeitos colaterais “B” e assim nem todos terão os sintomas “C”.
     

    A alternativa “C” está errada, pois não foram em todos os casos em que o medicamento “A” apresentou os efeitos colaterais “B” então “A” esta parcialmente em “B” e todos que tiveram o efeito colateral “B” apresentam os sintomas “C” então “B” esta  integramente em “C” e “C” não é necessariamente composto somente por “B”  podendo também ser composto pelos casos em que o medicamento “A” não gerou o efeito colateral “B”
     

    A alternativa “D” e “E” estão erradas, pois a questão ficou aberta em relação aos que tomaram ou não tomaram o remédio “A” podendo considerar que existem pacientes que tomaram e que não tomaram...

     

    Como ja respondido alternativa "A"

  • Resolvi assim: 2 círculos A interseção B, e um 3o terceiro círculo "abraçando" todo o B e passando em uma parte de A.

  • Resolução desenhada:

    http://sketchtoy.com/68969103

  • Uso o seguinte método:

    TODOS = DENTRO (circula da premisa 1, dentro da premisa 2.

    ALGUM (mesma coisa que EXISTE ou PELO MENOS UM) = CRUZA. ( Os circulos da premisa 1 e 2 se cruzam, formando uma intersecção.

    NENHUM = FORA. (Os circulos das premisas são "desenhados" de forma separada).

  • LETRA A

  • Vamos tentar ajudar os guerreiros com esse tipo de questão que trata dos quantificadores lógicos:

    "Em alguns casos, o medicamento A tem os efeitos colaterais B. Sempre que são observados os efeitos colaterais B, o paciente apresenta os sintomas C.".

    Resolução:

    Em alguns casos= alguns. O próprio conceito da questão já está restringindo a assertiva. Só com essa informação já é possível inferir um resultado.

    P: Em alguns casos, o medicamento A tem os efeitos colaterais B = V

    Q: Sempre que são observados os efeitos colaterais B, o paciente apresenta os sintomas C = V

    Questão: Considerando verdadeiras as proposições apresentadas acima, assinale a opção correta.

    A) Alguns pacientes que tomaram o medicamento A apresentam os sintomas C

    Comentário: por eliminação é a uma conclusão lógico possível, já que com base na soma das proposições é possível afirmar isso, já que se essa assertiva é uma soma dos conceitos das proposições simples acima com valor verdadeiro. (Um sujeito, um verbo e um predicado). O princípio do O princípio do Terceiro Excluído pode ser usado para explicar a questão.

    P: Alguns pacientes que tomaram o medicamento A = V (a questão diz que é verdadeiro)

    Q: apresentam os sintomas C = V (a questão diz que é verdadeiro)

    P ^ Q = V (Tudo V dá V) = verdadeiro.(A junção das proposições simples com valor lógico verdadeiro apresentam essa conclusão lógico como possível e verdadeira. É uma inferência e dedução.

    B) Se um paciente tomar o medicamento A, então ele apresentará os sintomas C

    Apenas um exemplo:

    P: Se um paciente tomar o medicamento A = V ou F

    Q: então ele apresentará os sintomas C = V ou F

    Comentário: Nesse caso, pode ser verdadeiro ou falso. Porém, não é possível definir com base na assertiva um valor exato. Com base no princípio da "não contradição", a questão está errada.

    C) Se um paciente apresentar os sintomas C, então ele sofreu os efeitos colaterais B.

    Comentário: Nesse caso, assim como no de cima, pode ser verdadeiro ou falso. Porém, não é possível definir com base na assertiva um valor exato. Com base no princípio da "não contradição", a questão está errada.

    D) Os pacientes que têm os sintomas C tomaram o medicamento A.

    Comentário: Nesse caso, assim como no de cima, pode ser verdadeiro ou falso. Porém, não é possível definir com base na assertiva um valor exato. Com base no princípio da "não contradição", a questão está errada.

    E Se um paciente sofrer os efeitos colaterais B e apresentar os sintomas C, ele tomou o medicamento A.

    Comentário: Nesse caso, assim como no de cima, pode ser verdadeiro ou falso. Porém, não é possível definir com base na assertiva um valor exato. Com base no princípio da "não contradição", a questão está errada.

    Espero ter contribuído para o raciocínio dos guerreiros. Há outras formas de resolver a questão, apenas citei uma que pode ser usada.


ID
1311172
Banca
CESGRANRIO
Órgão
IBGE
Ano
2013
Provas
Disciplina
Raciocínio Lógico
Assuntos

Considere verdadeiras as seguintes premissas:

Todas as pessoas que andam de trem moram longe do centro.
Todas as pessoas que andam de carro não andam de ônibus.
Algumas pessoas andam de ônibus e de trem.

Portanto,

Alternativas
Comentários
  • https://www.aprovaconcursos.com.br/noticias/2013/12/03/gabarito-comentado-concurso-ibge-tecnico-raciocinio-logico/

  • Trem - Longe do Centro

    Carro -  ~ Onibus

    Onibus e Trem


    Portanto, Se Onibus e Trem são verdadeiros:

    Carro -  ~ Onibus = Onibus - ~Carro

    Trem - Longe do Centro


    Com essas informações, só nos resta a resposta (B) algumas pessoas que moram longe do centro não andam de carro.


  • Montando o diagrama de Venn para solucionarmos esta questão, temos:

    Assim, observando cada alternativa e verificando de acordo com o nosso diagrama acima, vemos que a única correta é a alternativa B.


    Resposta: Alternativa B

  • Questão dúbia ao meu ver.


    Porque não é a letra d correta também, uma vez que ela tem valor lógico igual a D? me corrijam se eu estiver errado.

    vejam a minha lógica; Não é equivalente dizer

    algumas roupas são azuis e não são de marca

    algumas roupas são azuis e são de marca

    Dá no mesmo.

    Pois é justamente isso que a letra b e a letra d fazem.

    Só existem duas possibilidades, perto ou longe. Não existe um meio termo. 

    resultado de questão forjado. Eu entraria com recurso. Eles tentam criar uma questão muito absurda, pq até mesmo o diagrama de venn nesse caso se torna quase impossível de fazer.

  • Que questão polêmica! Nem uma pessoa do primário montaria uma questão assim.

  • Ninguém conseguiu me explicar de maneira clara porque a D está errada. Para mim o gabarito tem duas alterativas corretas. 

  • Baseando-se nas premissas  s não há como termos certeza se "algumas pessoas que andam de carro moram longe do centro" pois tambem não podemos afirmar que "das pessoas que andam de trem algumas ande de carro".

    Logo a afirmação que podemos fazer (a certeza que temos) é que 

     b)algumas pessoas que moram longe do centro não andam de carro. 

    Fiz circulos para cada tipo de pessoas; tipo, o circulo das pessoas q andam de trem tem uma intersecção com as que andam de onibus, como podemos afirmar pela premissa  Algumas pessoas andam de ônibus e de trem. Sabemos que Todas as pessoas que andam de trem moram longe do centro logo as que andam de (trem) e de onibus moram( longe do centro), e estas ja que andam de (onibus) não andam de (carro), afirmando assim que b)algumas pessoas que moram longe do centro não andam de carro. 

  • algumas roupas são azuis e não são de marca

    algumas roupas são azuis e são de marca

    Dá no mesmo. ? Claro que nao 

    E pra mim a D nao possui verdade pois a palavra antecedente a afirmativa é que faz diferenciaçao à logica - todas e algumas.

  • A alternativa D não pode ser a correta pois não podemos afirmar com toda certeza que quem anda de carro mora longe do centro. Existem várias possibilidades para quem anda de carro, para responder questões assim é preciso pensar em todas. 

  • Gab: B

    O conjuntos das pessoas que andam de trem está contido no das pessoas que moram longe (todos que andam de trem moram longe). Sabemos que quem anda de ônibus não anda de carro, ou seja, não há uma intersecção entre esses conjuntos e que há pessoas que andam de trem e de ônibus simultaneamente. Logo, as pessoas que andam de trem e de ônibus não andam de carro e moram longe.

     

  • GAB B

     

    Eu também não estava entendendo o porquê da D não estar certa e no comentário do nosso colega Robson que indicou a questão comentada do Aprova concursos eu consegui ser convencida do gabarito rs.

     

    Os conjuntos de “carro” acima são as possibilidades de localização desse grupo, sendo que todos ficam afastados do conjunto ônibus e não podemos garantir quaisquer destes com certeza.  (vale a pena dar uma olhada no diagrama do site, que, pra mim, é o mesmo diagrama do comentário do prof. do QC.)

     

    O conjunto de “carro” só não pode se misturar com ônibus e, como há pessoas que andam de trem e ônibus, então podemos concluir que há pessoas que andam de trem que não andam de carro. Quais? Aquelas que andam de ônibus, pois se anda de ônibus não anda de carro.

     

    Como todos os que andam de trem moram longe, então também podemos concluir que algumas pessoas que moram longe não andam de carro. Quais? Aquelas que andam de trem e ônibus, pois todos os que andam de ônibus não andam de carro.

     

    https://www.aprovaconcursos.com.br/noticias/2013/12/03/gabarito-comentado-concurso-ibge-tecnico-raciocinio-logico/

  • Os que moram longe do centro e com certeza não andam de carro são justamente aqueles que andam de trem e de ônibus, porque se anda de ônibus, NÃO ANDA DE CARRO, e se anda de trem, MORA LONGE DO CENTRO.


ID
1313224
Banca
FGV
Órgão
INEA-RJ
Ano
2013
Provas
Disciplina
Raciocínio Lógico
Assuntos

O time titular de futebol de um clube possui 11 jogadores sendo que 4 são nordestinos, 3 são gaúchos, 2 são paranaenses e 2 são mineiros. Há 5 canhotos e, entre eles, sabe-se que um deles é pernambucano. Então, sobre os jogadores do time é correto afirmar que

Alternativas
Comentários
  • Olá, pessoal!

    Essa questão não foi alterada pela Banca. Alternativa correta Letra C.

    Bons estudos!
    Equipe Qconcursos.com

  • se por acaso os quatro nordestinos ( contando com o pernambucano ) forem canhotos, sobrará uma mãozinha para o resto. e todos estes, de certa forma, estão no sul tche. resposta letra D.

  • Lembre de que há cinco jogadores do Sul (3 gaúchos e 2 paranaenses), logo, sendo que um dos canhotos é nordestino, sobram quatro canhotos, então pelo menos um sulista é destro.

  • Tão fácil que custei a acreditar que era FGV KKK

  • pra mim essa questão tem duas respostas C e D

  • Mineiros não são sulistas. Marcio Galvino, a "mãozinha" restante pode ser de um mineiro.

    GABARITO: C

  • Se 1 nordestino é canhoto, logo 3 não serão.

    Sobram 4 canhotos, para 3 gaúchos, 2 paranaenses e 2 mineiros

    Se 2 mineiros forem canhotos, ainda sobram 2 canhotos, o quais serão obrigatoriamente do sul (PR ou RS)

    Alternativa correta = C

    Bons estudos!!!

  • e eu lá sabia q o parana é no sul

  • Região nordeste: 4 (inclui o pernambucano)

    Região sul: 5 (3 gaúchos e 2 paranaenses)

    Região sudeste: 2 mineiros

  • Se a equipe tem cinco canhotos e pelo menos um é de Pernambuco, restam quatro.

    Somando todos os jogadores da região sul (três gaúchos + dois paranaenses), resulta em cinco.

    Ou seja, temos a certeza que pelo menos um jogador da região sul é destro.

    Gabarito: Letra C

  • Boa questão e fácil a questão. Errei por causa da falta de conhecimento geografia! Vida que segue!

ID
1313236
Banca
FGV
Órgão
INEA-RJ
Ano
2013
Provas
Disciplina
Raciocínio Lógico
Assuntos

Sejam X, Y e Z propriedades diferentes relativas a pessoas. Nesse caso,

• pelo menos uma pessoa que tem X também tem Y;
• todas as pessoas que têm Y também têm Z.

A esse respeito, assinale a afirmativa correta

Alternativas
Comentários
  • Pela primeira sentença que diz que pelo menos uma pessoa que tem X, também tem Y e que todos que tem Y têm Z, dá pra que imaginar que essa uma pessoa que tem X, também tem Y, e por consequência tem Z.

  • NÃO TEM COMO O ''X'' (que é pelo menos um) TER O ''Y'' SEM TER O ''Z'', POIS O ''Y'' ESTÁ DENTRO DO CONJUNTO ''Z''



    GABARITO ''D''
  • ...a que tem Y!

  • Negação dos Quantificadores:

    • TODO troca por ==> ALGUM / PELO MENOS UM/ EXISTE + NEGA o verbo

    Ex:  “Todo aluno é inteligente" ALGUM aluno NÃO é inteligente / PELO MENOS UM aluno NÃO é inteligente/

    EXISTE aluno que NÃO é inteligente.

  • é muita lombra viu!!!

ID
1313593
Banca
CETRO
Órgão
Prefeitura de São Paulo - SP
Ano
2014
Provas
Disciplina
Raciocínio Lógico
Assuntos

Preocupados em reestruturar as atividades oferecidas pelo Centro Esportivo da cidade, os dirigentes fizeram uma pesquisa sobre a preferência dos usuários aos esportes oferecidos. Notou-se que todos os praticantes de caminhada também faziam yoga, mas nenhum dos alunos de yoga praticava natação. Todos os alunos de spinning eram também praticantes de pilates e alguns dos que praticavam pilates faziam natação. Como nenhum dos alunos de pilates praticava yoga e nenhum dos que faziam spinningpraticavam natação, conclui-se que

Alternativas
Comentários
  • C està contido em Y;

    S està contido em P;

    P intersec N >0;

    Como nenhum dos alunos de pilates praticava yoga, logo nenhum de spinning faz caminhada

  • Vejam a resolução dessa questão no vídeo que gravei: https://youtu.be/hps2iJ_zb1o

  • DA UM NO NAS IDEIAS.


ID
1317184
Banca
FGV
Órgão
SUDENE-PE
Ano
2013
Provas
Disciplina
Raciocínio Lógico
Assuntos

Supondo que a afirmativa “Todos os estados d do Nordeste sofrem com m  a  seca  ou  com  o  excesso de  chuvas”  seja  falsa,  analise as afirmativas a seguir. 

I.  “Nenhum estado do  Nordeste sofre  com  a  seca  ou  com o excesso de chuvas”. 
II.  “Algum estado do Nordeste não sofre com a seca”. 
III.  “Algum estado do Nordeste sofre com o excesso de chuvas”. 

Assinale:

Alternativas
Comentários
  • Aqui também quero ajuda. 

  • "Todos os estados do Nordeste sofrem com a  seca  ou  com  o  excesso de  chuvas" se essa é falsa então ele tá pedindo oque é verdade.

    A negativa de "todo" é "algum não é" então:

    "Algum estado brasileiro não sofre com a seca e nem (também não) com o excesso de chuvas".

    Gabarito B, pois representa a primeira parte da assertiva acima.

  • I.  “Nenhum estado do  Nordeste sofre  com  a  seca  ou  com o excesso de chuvas”. 

    Errado - a negativa de todos é algum, pelo menos um e etc. + a negação da frase
    II.  “Algum estado do Nordeste não sofre com a seca”. 

    Certa 
    III.  “Algum estado do Nordeste sofre com o excesso de chuvas”.

    Aqui faltou apenas a negação da frase

     

  •  “Todos os estados do Nordeste sofrem com  a  seca  ou  com  o  excesso de  chuvas” 

    A frase é uma disjunção. Para que ela seja falsa (como a questão pede) as duas proposiçoes devem ser falsas (FF), pois uma disjunção só é falsa quando as duas proposições são F.

    Portanto: 

    1ª proposição: Todos os estados do Nordeste sofrem com  a  seca (F) = Algum estado do Nordeste não sofre com a seca

    2ª proposição: Todos os estados do Nordeste sofrem como  excesso de  chuvas (F) = Algum estado do Nordeste não sofre com o excesso de chuvas.

    Só temos a 1ª como opção de resposta, então apenas o item II está correto.

     

  • Negação dos Quantificadores:

    • TODO troca por ==> ALGUM / PELO MENOS UM/ EXISTE + NEGA o verbo

    Ex:  “Todo aluno é inteligente" ALGUM aluno NÃO é inteligente / PELO MENOS UM aluno NÃO é inteligente/

    EXISTE aluno que NÃO é inteligente.


ID
1330066
Banca
FMP Concursos
Órgão
PROCEMPA
Ano
2012
Provas
Disciplina
Raciocínio Lógico
Assuntos

Todos jogadores do São José são gaúchos. Alguns jogadores gaúchos são canhotos. Logo,

Alternativas
Comentários
  • Gabarito C

    Todos jogadores do São José são gaúchos.

    Alguns jogadores gaúchos são canhotos.

    Logo,

    alguns jogadores canhotos são gaúchos.

    A) alguns jogadores do São José são canhotos. (Não dá pra saber)

    B) alguns jogadores do São José não são canhotos. (Não dá pra saber)

    C) alguns jogadores canhotos são gaúchos. (Correta)

    D) todos os jogadores gaúchos são canhotos. (Não dá pra saber)

    E) todos os jogadores do São José são canhotos. (Não dá pra saber)


ID
1332643
Banca
FGV
Órgão
AL-MT
Ano
2013
Provas
Disciplina
Raciocínio Lógico
Assuntos

Certo deputado fez a seguinte afirmação sobre os trabalhos da semana: 


“Todos os itens da pauta serão votados". 

Na semana seguinte, verificou-se que essa afirmação foi falsa.
É correto concluir que

Alternativas
Comentários
  • Resposta letra C

    Expressões: Todo A é B

    Negação: Algum A não é B, Existe A que não é B, Pelo menos um A não é B, Nem todo A é B.

  • Quando fala TODO para ser Verdade,  se pelo menos 1 não foi, quer dizer que foi mentira.

    Ex: "Eu acerto todas"

    - MENTIROSO! VOCÊ ERROU PELO MENOS UMA!

  • NEGAÇÃO

     

                     TODO  ------------------->  ALGUM ... NÃO / PELO MENOS UM ... NÃO ... / EXISTE ... NÃO...

               NENHUM / NÃO EXISTE      ----------------------->  ALGUM 

                     ALGUM                ------------------------->   NENHUM / NÃO EXISTE

    ALGUM ... NÃO / PELO MENOS UM ... NÃO ... / EXISTE ... NÃO... --------->  TODO

  • GAB. C)

    Pelo menos um item da pauta não foi votado.


ID
1332841
Banca
CETRO
Órgão
AEB
Ano
2014
Provas
Disciplina
Raciocínio Lógico
Assuntos

Em uma empresa, sabe-se que existe pelo menos um advogado do sexo feminino. Sabe-se, ainda, que, nessa empresa, todos os funcionários do sexo feminino usam brincos. Dessa forma, é correto concluir que

Alternativas
Comentários
  • algum advogado usa brinco 

    resposta; c

  • se pelo menos um advogado é do sexo feminino e todos os funcionários do sexo feminino usam brinco, obviamente algum advogado certamente usa brinco (a do sexo feminino)

  • Fiz pela lógica do diagrama...assim acertei...

  • Vejam a resolução dessa questão no vídeo que gravei: https://youtu.be/hm5Y0u6gNg4

  • kkkkkk


ID
1360396
Banca
CESGRANRIO
Órgão
Petrobras
Ano
2014
Provas
Disciplina
Raciocínio Lógico
Assuntos

Determinado técnico de atletismo considera seus atletas como bons ou maus, em função de serem fumantes ou não. Analise as proposições que se seguem no contexto da lógica dos predicados.

I - Nenhum fumante é bom atleta.

II - Todos os fumantes são maus atletas.

III - Pelo menos um fumante é mau atleta.

IV - Todos os fumantes são bons atletas.

As proposições que formam um par tal que uma é a negação da outra são:

Alternativas
Comentários
  • Redobrar a atenção na diferença entre negação e antítese.

  • Não entendi

  • Para negar o TODO basta pelo menos um ou algum.

    Ex: Todo peixe vive no mar.(Negação) Algum peixe não vive no mar ou Pelo menos um peixe não vive no mar.

  • Algum A é B = pelo menos um A que seja B

    Negação (algum A é B) = Nenhum A é B

    ------------------------------------------------------------------------------------------------------------------------------

    Nenhum A é B = todo A não é B = Não existe um A que seja B

    Negação (Nenhum A é B) = algum A é B = pelo menos um A que seja B

    ------------------------------------------------------------------------------------------------------------------------------

    Todo A é B = nenhum A não é B = Não há um A que não seja B

    Negação (Todo A é B) = pelo menos um A não é B

  •  

    Olá pessoal,
    Vejam o vídeo com a resolução dessa questão no link abaixo

     

    http://youtu.be/A7uESCZebyM

     

     

    Professor Ivan Chagas
    www.gurudamatematica.com.br

  • Levando-se em conta que a negação do quantificador "Todos" é "Pelo menos um" (vice - versa) e que ao negarmos qualquer proposição significa trocar seu sentido, temos que:

    III - Pelo menos um fumante é mau atleta. 

    IV - Todos os fumantes são bons atletas. 

    Formam um par tal que uma é a negação da outra.

    Resposta: Alternativa E.
  • Esse gabarito está errado. Vejamos:

    A negação de TODO = ALGUM. 

    A negação de ALGUM = NENHUM

    A negação de NENHUM = ALGUM



    Logo a as alternativas que seriam negações uma da outra são I e III. Vejamos:

    I - Nenhum fumante é bom atleta.               (negação)  III - Pelo menos um(algum) fumante é mau atleta. 

    III - Pelo menos um(algum) fumante é mau atleta.  (negação)  I - Nenhum fumante é bom atleta.



    Agora vejamos a alternativa E:


    III - Pelo menos um(algum) fumante é mau atleta.   (negação) I - Nenhum fumante é bom atleta. (Notem que a negação de III = I)

    IV - Todos os fumantes são bons atletas.      (negação) III - Pelo menos um fumante é mau atleta.


  • Obrigado prof Chagas

  • QC contrate o professor Prof. Chagas para comentar as questões.

     

  • Questão boba que eu errei, mas era só lembrar do bizu do professor Renato aqui do Qconcursos....

     

    Negação do TODO  = PEA+NÃO

  • considerando:

    F(x): fumante.

    A(x): bom atleta.

    a negação de qualquer que seja é existe pelo menos um, então...

    ~(qualquer que seja x (F(x) -> A(x)) ) é (lembrando a regra  que F(x) -> A(x) ~F(x) v A(x)) (existe pelo menos um x(~~F(x) ^ ~A(x)) (existe pelo menos um x (F(x) ^ ~A(x)) que significa: existe pelo menos um x, x é fumante e  x é mau atleta (ou, x não é bom atleta)

  • LETRA E


ID
1382758
Banca
FUNRIO
Órgão
IF-BA
Ano
2014
Provas
Disciplina
Raciocínio Lógico
Assuntos

Sabe-se que todo B é A e que algum C é A. Segue-se necessariamente que

Alternativas
Comentários
  • gab. D pelo menos um (todo) A é B


  • Negação do TODO é ALGUM ou PELO MENOS UM
    p: Todo B é A

    ~p: pelo menos um A é B     ou
    ~p: algum A é B.
  • Não entendi o raciocínio nem da banca nem dos colegas abaixo.

    Pela teoria dos conjuntos e em pesquisas na net não achei conclusão para essa questão. Em nenhum momento foi solicitado a negação do TODO.

    Alguém explica?

  • Fiz pelo conjuntos e errei. Essa questão você tem que resolver pelo metodo Análise dos Quantificadores “Todo”, “Algum” e “Nenhum”. Ela te induz ao erro.

  • Ex: A= {1,2,3}  B= {1} C={3,6}

    Gabarito: D
  • Todo B é A e algum C é A.
    a)todo A é B (FALSO: pois considerando que B está contido em A, nem todo A vai ser B também)
    b)algum C é B (FALSO: pois não é possível afirmar isso através da sentença)

    c)todo A é C(FALSO: já que se algum C é A não é possível que todo A seja C)

    d)pelo menos um A é B (VERDADEIRO:pois já que B está contido em A deve existir ao menos um elemento de A em B, senão o conjunto B não existiria)

    e)nenhum B é C(FALSO: essa conclusão não é possível afirmar através da sentença, pois existe a possibilidade de ser)

  • Analisando as opções, tem-se:
    A) todo A é B.
    Falso, pois não se pode assegurar que A = B;

    B) algum C é B.
    Falso, pois não se pode afirmar.

    C) todo A é C.
    Falso, pois algum C é A.

    D) pelo menos um A é B.
    Verdadeiro, pois todo B é A.

    E) nenhum B é C.
    Falso, pois não se pode afirmar.

    Resposta D)

ID
1397968
Banca
VUNESP
Órgão
PRODEST-ES
Ano
2014
Provas
Disciplina
Raciocínio Lógico
Assuntos

Se todos os plastorius são plosturias, se algumas plosturias são plasturios, e se nenhuma plosturia é plustoria, então, é possível afirmar que

Alternativas
Comentários
  • Todo ''oriu'' é ''uria''.

    Nenhuma ''uria'' é ''oria''.

     

    Não há nenhum ''oriu'' que seja ''oria'' também. 

     

    nenhum plastoriu é plustoria. (letra c)

  •  c)nenhum plastoriu é plustoria.

    todos os plastorius são plosturias && nenhuma plosturia é plustoria. Os plastorius estao todos contidos em plosturias. Se nenhum plosturia é plustoria, tambem inclui plastorius

  • Véi...

  • aEUIJAEJIUEAJ... isso num final de prova faltando 10m dá até morte.

  • questão elaborada pelo encardido kkkkk

     

  • Zulivre!

  • Galera, renomeiam os itens e façam o diagrama que fica mais fácil:

     

    Se todos os plastorius são plosturias, se algumas plosturias são plasturios, e se nenhuma plosturia é plustoria, então, é possível afirmar que:

     

    plastoriu: X

     

    plosturia: Y

     

    plasturio: W

     

    plustoria: Z

     

    https://uploaddeimagens.com.br/imagens/sem_titulo-jpg-ae8f2048-8ea9-41e1-b611-c7ddaf64fcc7

     

    GAB:C

  • Qual o erro da D?

  • Seria engraçado Thiago Nunes explicando essa kkkkkkkk

  • Consegui por diagramas , mas só por Deus !

    Gabarito C

  • Essa questão é filha da plustoria.....kkkkkkkkkkkkk

  • Eu consegui visualizar assim: http://sketchtoy.com/69356700

  • Essa questão me fez lembrar do Jaco Pastorius, um dos maiores baixistas de todos os tempos. Nenhum plustorio ou plustoria será como ele!! :)

  • só li plapleuplarularl e não entendi mais nada KAKAKAKA

  • São necessários 6 diagramas para você visualizar bem cada caso com intercessão e sem intercessão.

  • não sabe se ri ou chora hahahha

  • O segredo da questão está na PLUSTORIA rs

    Brincadeiras a parte, tente fazer diagramas colocando esses nomes confuso acima de cada diagrama...ou ainda, como o colega disse renomeie, fica mais fácil a visualização.

  • questão ridícula, confunde quem estudou. Perdi uns 15 minutos desenhando essa m..., mas acertei....no calor da prova eu não teria esse tempo e provavelmente erraria. A banca nivelou quem estuda com quem chuta a p... toda...parabéns banca!!!

  • questão ridícula, confunde quem estudou. Perdi uns 15 minutos desenhando essa m..., mas acertei....no calor da prova eu não teria esse tempo e provavelmente erraria. A banca nivelou quem estuda com quem chuta a p... toda...parabéns banca!!!

  • Esses nomes parecidos é só pra fazer o candidato, ainda que saiba, chutar...deu pra acertar, mas na prova sei se teria essa paciência não

  • Nenhum A é D Renomeie para A,B,C e D e façam o diagrama !
  • PARECENDO QUE BEBI... PQP

  • Todos os plastorius são plosturias

    Algumas plosturias são plasturios

    Nenhuma plosturia é plustoria

    Logo, nenhum plastoriu é plustoria

    Nem tentei fazer diagrama, foi assim o meu raciocínio.

    GAB: C

  • Errando pela 6ª vez...

  • Quando a banca quer desclassificar os disléxicos...


ID
1400614
Banca
FGV
Órgão
AL-MT
Ano
2013
Provas
Disciplina
Raciocínio Lógico
Assuntos

Certo deputado fez a seguinte afirmação sobre os trabalhos da semana:

         “Todos os itens da pauta serão votados”.
Na semana seguinte, verificou-se que essa afirmação foi falsa.

É correto concluir que

Alternativas
Comentários
  • A negação de Todos os equivale a dizer que algum ficou de fora. Se pelo menos um não foi votado já nego o TODOS. Letra C

  • Quantificadores UNIVERSAIS (Todo, Nenhum) SÃO NEGADOS com Quantificadores EXISTENCIAIS (Algum, Existe, Pelo menos um) e VICE-VERSA. Na questão: Todo A é B nega-se Algum não é B.

  • NEGAÇÃO

     

                     TODO  ------------------->  ALGUM ... NÃO / PELO MENOS UM ... NÃO ... / EXISTE ... NÃO...

               NENHUM / NÃO EXISTE      ----------------------->  ALGUM 

                     ALGUM                ------------------------->   NENHUM / NÃO EXISTE

    ALGUM ... NÃO / PELO MENOS UM ... NÃO ... / EXISTE ... NÃO... --------->  TODO

  • GAB. B)

    Pelo menos um item da pauta não foi votado.

  • Para negar TODO, utiliza-se: pelo menos um/existe um/ algum + NÃO.

    Ex: Todos os itens da pauta serão votados

    Negação > Pelo menos um item da pauta não foi votado.

    Gabarito: Letra C

  • Negação dos Quantificadores:

    • TODO troca por ==> ALGUM / PELO MENOS UM/ EXISTE + NEGA o verbo

    Ex:  “Todo aluno é inteligente" ALGUM aluno NÃO é inteligente / PELO MENOS UM aluno NÃO é inteligente/

    EXISTE aluno que NÃO é inteligente.


ID
1425895
Banca
BIO-RIO
Órgão
CRMV-RJ
Ano
2014
Provas
Disciplina
Raciocínio Lógico
Assuntos

Numa selva, todos os felinos são pardos e nenhum animal pardo sabe nadar. Assim, nessa selva:

Alternativas
Comentários
  • A letra A tb está correta, da mesma forma que afirmo que "nenhum felino sabe nadar", tb está correto dizer que "Algum felino não sabe nadar".

    Questão deveria ser anulada, pois tem duas respostas corretas. 

  • www.gurudamatematica.com.br  

     


ID
1437664
Banca
CETRO
Órgão
Prefeitura de Manaus - AM
Ano
2012
Provas
Disciplina
Raciocínio Lógico
Assuntos

Quem não pergunta fica com dúvidas. Nenhum adolescente pergunta. Logo,

Alternativas
Comentários
  • Gab: A

    http://sketchtoy.com/69372816


ID
1437688
Banca
CETRO
Órgão
Prefeitura de Manaus - AM
Ano
2012
Provas
Disciplina
Raciocínio Lógico
Assuntos

Todo gato é egoísta. Algum gato é branco. Logo,

Alternativas
Comentários
  • Letra A

    a) Certo - Todo gato é egoísta, algum gato é branco, portanto, algum branco é egoísta. (o conjunto dos gatos está integralmente dentro do conjunto dos egoístas e o conjunto dos brancos está parcialmente dentro do conjunto dos gatos e consequentemente parte dos egoístas também.

    b) Errado - Como o conjunto dos Brancos está parcialmente dentro do conjunto dos egoístas, apenas alguns brancos são egoístas.

    c) Errado - Como citado, o conjunto dos Brancos está parcialmente dentro do conjunto dos gatos, portanto, algum branco é gato.

    d) Errado - Como o conjunto dos Gatos está dentro do conjunto dos egoístas, presume-se que o conjunto dos Egoístas seja maior, portanto, parte dos egoístas não é gato (coloque uma moeda de 10 centavos sobre a de 1 real, cobre parcialmente, as bordas que sobram são os egoístas que não são gatos)

    e) Errado - O conjunto dos brancos está parcialmente dentro do conjunto dos Egoístas, lembre da informação "Algum gato é branco", como Gato está dentro de Egoísta, para que o branco alcance o conjunto dos gatos, se faz necessário cobrir parte dos egoístas também.

    Espero ter ajudado, tentei ser o mais detalhista possível.

    Deus é fiel.






  • Lógica com Diagrama - 2as Leis de DE MORGAN. (Quantificadores Universais X Quantificadores Existenciais)


ID
1451530
Banca
CETRO
Órgão
AEB
Ano
2014
Provas
Disciplina
Raciocínio Lógico
Assuntos

Sabe-se que existem pessoas dedicadas e que existem pessoas felizes. Admitindo-se verdadeira a frase: “Todas as pessoas que são felizes, são dedicadas”, é correto concluir que

Alternativas
Comentários
  • O conjunto de pessoas felizes está contido no conjunto de pessoas dedicadas, logo elimina-se a letra A, já que podem existir pessoas infelizes e não dedicadas. Elimina-se também a letra B, pois todas as pessoas felizes são dedicadas. Como o conjunto de pessoas dedicadas contém o de pessoas felizes, podemos concluir que nem todas as pessoas dedicadas são felizes, ou seja, podem havem pessoas dedicadas e infelizes e, assim eliminamos a letra E e também a letra C.

    Resta a letra D: Gabarito.

  • http://sketchtoy.com/69295188


ID
1452691
Banca
FGV
Órgão
DPE-MT
Ano
2015
Provas
Disciplina
Raciocínio Lógico
Assuntos

Considere verdadeiras as afirmações a seguir.

• Existem advogados que são poetas.
• Todos os poetas escrevem bem.

Com base nas afirmações, é correto concluir que

Alternativas
Comentários
  • Acredito que não há resposta.

    A questão considera letra "a" como resposta, mas acredito que não se pode dizer que se o advogado não é poeta ele não escreva bem. O que se poderia dizer é se o advogado for poeta, então necessariamente escreva bem.

  • Achei a questão polêmica. Ele me dá uma proposição categórica e pede um princípio. Ô, fgv

  • A questão diz: TODO poeta escreve bem.

    Logo, se alguém não escrever bem, automaticamente não é poeta.

  • Vejo várias bancas adotando esse sistema de questão, porém acho muito errado, é o sistema dá "mais certa" ou "menos errada".

    Todos os POETAS escrevem bem, como há advogados que são poetas, não tem como saber com exatidão se advogados está dentro de escreve bem ou se faz apenas uma pequena parte do escreve bem. Por eliminação a mais coerente é a alternativa (A).

    FGV né, só aceita!

  • Lembrem da tabela verdade pessoal...

  • Tem gente confundindo. A alternativa "A" realmente está correta. Vejamos:

    "Existem advogados que são poetas. 
    Todos os poetas escrevem bem. 

    a) se um advogado não escreve bem então não é poeta."

    Ora, a questão diz que "todos os poetas escrevem bem", logo "se um advogado nao escreve bem, então ele nao é um poeta"

    Os colegas estão em seus comentarios invertendo a afirmativa da questão dizendo:  "se o advogado não é poeta ele não escreva bem." 

    Ora a afirmativa nao diz que "se ele nao é poeta ele nao escreve bem", diz o contrario "se ele nao escreve bem ele nao é poeta".

    Isso é obvio, pois dizendo que "Todos os poetas escrevem bem" eu nao estou dizendo que ninguem mais escreve bem, pode ser que tenham pessoas que escrevam bem, mas um poeta necessariamente escreve bem, por isso alguem para ser poeta deve escrever bem.

    Espero ter conseguido transpassar em palavras pros colegas o que na minha mente está claro.

    Não pode inverter a afirmativa da questão, senão fica errado. 

    Bons Estudos.

  • É possível responder por conjuntos. Há 3 conjuntos:

    Conjunto "Advogados" (A); Conjunto "Poetas" (P); Conjunto "Escrever bem" (E).
    Devemos, primeiramente, desenhar o conjunto dos universais (Todos). Portanto, se "Todos os poetas escrevem bem", logo o conjunto "Poetas" está contido no conjunto "Escrever bem" (P ⊂ E). E, se "Existem advogados que são poetas", logo o conjunto "Advogados" possui uma interseção com o conjunto "Poetas" (A ∩ P) e, consequentemente, com o conjunto no qual este está contido (A ∩ E). 
    Conclusão: A parte do conjunto "Advogados" que não possui uma interseção com o conjunto "Escrever bem" também não pode pertencer ao conjunto "Poetas", já que "'TODOS' os poetas escrevem bem", logo, "se um advogado não escreve bem então não é poeta". Resposta: Alternativa "A".    


  • Veja o vídeo que gravei com a resolução dessa questão:

    https://youtu.be/UwTxS5rs2vA

    Professor Ivan Chagas

  • GAB- A

  • Com as informações fornecidas no enunciado podemos montar o diagrama lógico abaixo:

    Repare que as pessoas na interseção entre os conjuntos dos advogados e dos poetas estão também dentro do conjunto das pessoas que escrevem bem. Assim, os advogados que são poetas necessariamente escreve bem. Caso um advogado não escreva bem, ele certamente não pode ser um poeta.

    Resposta: A

  • Pode haver advogados que escrevem bem e não são poetas.
  • Se todo poeta escreve bem, e o advogado não escreve bem, então o advogado não é poeta.

    Repare que a questão diz que EXISTEM advogados poetas, porém, os que não escrevem bem não são poetas.

    RESPOSTA letra A.


ID
1459330
Banca
FUNRIO
Órgão
IF-BA
Ano
2014
Provas
Disciplina
Raciocínio Lógico
Assuntos

Todo professor é inteligente e algum professor é paciente. Paulo é inteligente, Sérgio não é professor, Carlos não é inteligente e Antônio é paciente. Logo, conclui-se necessariamente que

Alternativas
Comentários
  • Conclusões que podemos inferir:


    - Todo professor é inteligente, mas nem todo inteligente é professor.  Assim, podemos dizer que Paulo pode ou não ser professor 

    - Sérgio pode ou não ser inteligente

    - Se Carlos não é inteligente, então necessariamente ele não é professor 

    - Antônio pode ou não ser professor 


    Letra E 


  • Podemos resolver facilmente a questão montando o diagrama de Venn abaixo:


    Assim, com base no diagrama: Se Paulo for inteligente, não necessariamente ele será professor, se Sérgio não é professor, nada impede que ele seja inteligente, se Carlos não é inteligente, ele automaticamente não pode ser professor, apenas paciente. E se Antônio é paciente, ele pode ser também professor, ou inteligente e não ser professor, ou apenas paciente.
    Assim, concluímos que Carlos não é professor.


    Resposta: Alternativa E.

  • Obrigado Wagner Candido, tinha chegado na letra E, no entanto, o gabarito diz que é D. 

  • Afinal qual a resposta correta por favor.

  • Negando todo mundo!

    Pelo menos um professor é inteligente e nenhum professor é paciente:

    - Paulo não é inteligente ----> não é professor

    -Sergio é professor----->inteligente

    -Carlos é inteligente----> é professor

    -Antonio não é paciente------> não é professor

    Analisando as alternativas com a Negação feita a resposta certa é a letra: "E"


ID
1459918
Banca
CESGRANRIO
Órgão
Petrobras
Ano
2012
Provas
Disciplina
Raciocínio Lógico
Assuntos

Considere as seguintes assertivas: todo gerente é um funcionário; cada funcionário é gerenciado por um gerente.
A análise da relação lógica entre essas afirmativas permite concluir que

Alternativas
Comentários
  • Letra (e)

     

  • e-

    parafrase do enunciado. todo gerente é funcionario; o oposto nao é verdade. cada funcionario tem 1 gerente, o que significa que pode haver varios para 1 gerente.


ID
1464049
Banca
FCC
Órgão
TRT - 18ª Região (GO)
Ano
2013
Provas
Disciplina
Raciocínio Lógico
Assuntos

Rafael participou de um bolão com seus colegas de trabalho em que precisava tentar adivinhar o vencedor das três partidas que o time de sua cidade realizaria em um torneio. Os palpites de Rafael foram os seguintes:

                        Partida                          Palpite de Rafael
          Kanela Fina × Arranka Toco       Vitória do Kanela Fina
          Kanela Fina × Xinelinho                         Empate
          Kanela Fina × Espanta Sapo        Vitória do Kanela Fina

Neste torneio, em cada partida, o vencedor ganhava 3 pontos e o perdedor não ganhava ponto. Em caso de empate, cada equipe marcava 1 ponto. Ao final dos três jogos, a equipe do Kanela Fina marcou 4 pontos e Rafael errou todos os seus palpites. Apenas com essas informações, pode-se concluir que, necessariamente, a equipe Kanela Fina

Alternativas
Comentários
  • Gabarito: Letra C

    A equipe Kanela Fina não ganhou do Arranca Toko e nem do Espanta Sapo pois o Rafael errou todos os seus palpites, mas ela precisa ter ganhado algum jogo para conseguir marcar os 4 pontos. Logo, ela venceu o Xinelinho.

  • A única soma possível seria uma vitória(3 pt) + um empate(1 pt)


    Como ele errou, não há como afirmar que o Kanela Fina venceu o 1º ou  3º jogo, 


    dessa forma, só sobra a vitória para o 2º jogo.



    Já que o Kanela Fina venceu o 2º jogo, e empatou outro, temos que poderá ser tanto o 1º ou  3º jogo, 


    no entanto não há como afirmar qual deles é o do empate e qual o da derrota, mas com certeza podemos

    afirmar que ele venceu o o 2º jogo.



                    Partida                                      Palpite de Rafael 
     Kanela Fina × Arranka Toco                   Vitória do Kanela Fina ..........(ERROU) - então pode ser empate ou derrota
     Kanela Fina × Xinelinho                                    Empate          ..........(ERROU) - só pode ser vitória
     Kanela Fina × Espanta Sapo                  Vitória do Kanela Fina  ..........(ERROU) - então pode ser empate ou derrota

  • Não encontrei a resposta ...

    vitoria : perde ou empata

    empata: perde ou ganha

    vitoria: ganha ou empata 

  • Das 3 apostas do Rafael 2 eram em vitórias, sabendo que o time kanela fina fez apenas 4 pontos e que Rafael errou todos os chutes chegamos a conclusão que o time que ele não apostou na vitória foi o único que venceu, pois algum desses times fez 3 pontos . Os outros dois não há como saber o resultado.

  • Errei com força

  • Muito fácil, é só pensar um pouco. Já que ele errou todos os palpites e que segundo a assertiva entendemos que Kanela fina teve uma vitória e um empate, chegamos a conclusão que o empate no palpite é falso e houve uma vitória, logo a vitória foi do kanela fina.

  • responde a questõa e saberá se está certa ou errada caraliow

  • A maioria das questões mais elaboradas não têm comentário de professor no Q concursos.

  • CORRETA.Pessoas que não sao assinantes.


ID
1472788
Banca
CESGRANRIO
Órgão
IBGE
Ano
2014
Provas
Disciplina
Raciocínio Lógico
Assuntos

A respeito de um pequeno grupo indígena, um repórter afirmou: “todos os indivíduos do grupo têm pelo menos 18 anos de idade”. Logo depois, descobriu-se que a afirmação a respeito da idade dos indivíduos desse grupo não era verdadeira.
Isso significa que

Alternativas
Comentários
  • Para eu negar essa afirmação basta PELO MENOS um indivíduo ter menos de 18 anos de idade. 

  • A afirmação Falsa: “todos os indivíduos do grupo têm pelo menos 18 anos de idade”.

    Isso implica dizer que todos os indivíduos têm idade maior ou igual a 18 anos.

    A afirmação verdadeira: Alternativa "E" : "pelo menos um indivíduo do grupo tem menos de 18 anos de idade".

    Logo, pelo menos um é menor de 18 anos.

    João Thiago

  • Não entendi a questão. E por que não poderia ter mais de 18 anos, como informado na alternativa D?

  • A negação não seria dizer que eles têm mais de 18 porque o enunciado diz que eles têm 18, no mínimo. "No mínimo" significa dizer que eles podem ter 18 ou mais, logo a sua negação seria dizer que eles têm menos de 18 anos.

  • GABARITO: E

     

     

    "Todo A  é B" ---negação--- "Algum A não é B" e vice-versa.

    "Algum A é B" ---negação--- "Nenhum A é B" e vice-versa.

     

    * Prestar atenção na ideia da questão.

  • Eu entendi da seguinte forma:

     “todos os indivíduos do grupo têm pelo menos 18 anos de idade” = F

    Para ela ser verdadeira: “Algum indivíduo do grupo não têm menos 18 anos de idade” = V, ou ainda, “pelo menos um indivíduo do grupo têm mais de 18 anos de idade” = V

    Para que seja falsa novamente: “pelo menos um indivíduo do grupo não têm mais de 18 anos de idade” = F, ou seja, "tem menos de 18 anos"

    Questão confusa!

     

  • Negações simples:

    AFIRMAÇÃO  - NEGAÇÃO

    É - NÃO É

    TODOS - ALGUM/PELO MENOS UM/EXISTE + NÃO É

    NENHUM - ALGUM/PELO MENOS UM/EXISTE + É

     

  • gabarito E pra vc q ta no grátis

    Luana Silva

    entendi o erro da .D por sua causa,

    obrigada

  • LETRA E

  • Substitua o "pelo menos" por "no minimo" e seja feliz kkkk

  • GAB LETRA: E

    Pessoal então, Vamos resolver no português.

    A afirmação fala que: “todos os indivíduos do grupo têm pelo menos 18 anos de idade”. Logo depois, descobriu-se que a afirmação a respeito da idade dos indivíduos desse grupo não era verdadeira.

    Ou seja é falsa Então sabemos que já não e verdadeira. Logo Raciocinando Logicamente ''Alguém deve ter ''Ate'' ou "Pelo menos'' 18 anos de idade, e não tem como ele ter outras idades Superiores, ou seja tem que ser inferior a 18 anos. com isso eliminamos a letra: A,B,C,

    ''Pelo menos'' é igual a ''ate'' é uma proporção sem extrapolar. Portanto Eliminamos a alternativa: D

    GAB LOGICAMENTE: ''pelo menos um indivíduo do grupo tem menos de 18 anos de idade''.

  • Link da resolução do exercício:

    https://youtu.be/bK4EcYUGyUQ


ID
1501318
Banca
COPEVE-UFAL
Órgão
UNCISAL
Ano
2015
Provas
Disciplina
Raciocínio Lógico
Assuntos

Em uma clínica existem as funções de médico, técnico de laboratório, recepcionista e expedidor de exames. Suponhamos que: alguns médicos assumem também a função de técnico de laboratório; os recepcionistas não são médicos, mas alguns também são técnicos de laboratório; todos os expedidores de exames são também recepcionistas. Desse contexto, infere-se que

Alternativas
Comentários
  • Gabarito A. Me corrijam se eu estiver errada, mas aqui seria um argumento categórico contraditório?  http://minerva.ufpel.edu.br/~camila.costa/Arquivo/2013_01/logica/cap_3_pag_110_134.pdf

  • Fiz um vídeo com a resolução dessa questão, não deixem de ver.


    Link para o vídeo com a resolução dessa questão:  https://youtu.be/wzEJ6tcMA7U


    Professor Ivan Chagas

     

    Muito mais vídeos com questões resolvidas em www.gurudamatematica.com.br

     

  • Partimos da seguinte premissa: alguns médicos assumem também a função de técnico de laboratório; os recepcionistas não são médicos, mas alguns também são técnicos de laboratório; todos os expedidores de exames são também recepcionistas

     

     a) CORRETO. Alguns médicos são técnicos e alguns recepcionistas também são técnicos. Isso não quer dizer que os técnicos que são médicos e recepcionistas representam todos os técnicos. Existe a possibilidade de haver técnicos que não são médicos nem recepcionistas.

     

     b) A premissa afirma que (todos) os expedidores de exames são recepcionistas, mas não necessariamente o contrário. Pode haver recepcionistas que não são expedidores de exames.

     

     c) Não é possível afirmar que existem técnicos que sejam expedidores de exames. Os recepcionistas englobam todos os expedidores de exames e alguns técnicos, mas não há nenhuma menção quanto à técnicos que sejam expedidores de exames.

     

     d) Os expedidores de exames podem, sim, ser técnicos de laboratórios. A questão nada afirma sobre isso, mas existe a possibilidade, pois os expedidores de exame se encontram no mesmo grupo de alguns técnicos, grupo este chamado recepcionistas.

     

     e) Dentro do grupo dos médicos e recepcionistas existem técnicos de laboratório. Mas isso não quer dizer que todos os técnicos estejam nesses grupos. São alguns técnicos que compõem ambos os grupos.

     

     

  • Muito obrigada Prof. Ivan Chagas.

     

  • http://sketchtoy.com/69295178

    Fiz o diagrama, vejam como ficou mais fácil.


ID
1505995
Banca
UFMG
Órgão
UFMG
Ano
2010
Provas
Disciplina
Raciocínio Lógico
Assuntos

Considerando que alguns advogados são políticos e que não é verdade que algum escritor é político, conclui-se que

Alternativas
Comentários
  • Enunciado diz que:
    -Alguns advogados são políticos

    -Não é verdade que algum escritor é político. Leia-se: nenhum escritor é político.

     

    Assertivas

     

    A) Errado. Pode existir um advogado escritor que não seja político desde que haja ao menos um outro advogado que seja político.

    B) Certa. Se todos os advogados fossem escritores, então nenhum advogado seria político, visto que nenhum escritor é político. Porém o enunciado afirma que alguns advogados são políticos, logo esses advogados não são escritores.

    C) Errada. Ser escritor não é condição necessário para não ser político. E mesmo que fosse, nada impede que todos os advogados fossem políticos. Todos os advogados serem políticos não faz com que a frase "alguns advogados são políticos" seja falsa.

    D) Errada. Pode existir um advogado escritor que não seja político desde que haja ao menos um outro advogado que seja político.

  • A alternativa C está incorreta pois o enunciado afirma que alguns advogados são políticos e que não é verdade que algum escritor é político.

  • Olhando apenas as alternativas já é possível eliminar 2 alternativas, a "A" e a "D" pois elas são iguais, apenas inverteram. Pois vejam que dizer:  "nenhum A é B" é a mesma coisa que: "nenhum B é A"

     

    A) nenhum advogado é escritor

    E) nenhum escritor é advogado

     

    Bons Estudos!!!


ID
1508887
Banca
FGV
Órgão
SSP-AM
Ano
2015
Provas
Disciplina
Raciocínio Lógico
Assuntos

Considere a afirmação: “Todo animal de 4 patas é mamífero”.

A negação dessa afirmação é:

Alternativas
Comentários
  • esta pergunta trás uma pegadinha, pq a negação de " todo" é algum, que na resposta ficou implícito, ou seja, a resposta no seu teor seria " Existe algum animal de 4 patas que não é mamífero" atenção quanto aos termos implícitos

  • Galera vamos lá macetinho da tia!

    negação do todo pense em P E A + NÃO

    onde; 

    1ª parte                                              segunda parte NEGA (verbo)

    P (pelo menos 1)

    E (existe 1)

    A (Algum)                                                         GABARITO: E, de Estagio Probatório, rsrsrsrs 

  • Negação dos quantificadores

    Afirmação             Negação

    Todos são...            Algum não é...

    Nenhum é...            Algum é...

    Algum não é...        Todos são...

    Algum é...               Nenhum é...


    Afirmação: Todo animal de 4 patas é mamífero.

    Negação: Algum animal de 4 patas não é mamífero.

    Resposta: Existe animal de 4 patas que não é mamífero. ==> Expressa a mesma ideia da negação anterior.

  • Gabarito Letra E

     

    Para negação de TODO, ALGUM, NENHUM, ALGUM NÃO É, eu utilizo esta tabela e faço a inversão, igual regra de três.

     

    TODO -----------NENHUM

    ALGUM É -----ALGUM NÃO É


    Agora, inverto "regra de três"

    TODO=ALGUM NÃO É; PELO MENOS UM NÃO É

    NENHUM=ALGUM É; PELO MENOS UM

     

    Funciona!!!

     

    RESPOSTA: Existe animal de 4 patas que NÃO é mamífero (OBSERVE QUE TEM A PALAVRA "NÃO" NA RESPOSTA)

     

     

     

  • N Y M

  • A negação da frase “todo animal de 4 patas é mamífero” é “algum animal de 4 patas não é mamífero”.
     

     

    Gabarito: E

  • GAB E

    MÉTODO TELLES:  

     

     https://www.youtube.com/watch?v=Mc5lEDRiotI

     

    NEGAÇÃO DO “TODO”, é quebrar  a IDEIA de generalização do “todo”, com ALGUM seguido de  “NÃO”.        Mantém o REFERENCIAL  (dia, noite, claro, escuro)

     

    "Todo agente penitenciário é do sexo masculino"

    ENTÃO:

    "ALGUM (pelo menos um) agente penitenciário é do sexo feminino"

     

  • Negação dos Quantificadores:

    • TODO troca por ==> ALGUM / PELO MENOS UM/ EXISTE + NEGA o verbo

    Ex:  “Todo aluno é inteligente" ALGUM aluno NÃO é inteligente / PELO MENOS UM aluno NÃO é inteligente/

    EXISTE aluno que NÃO é inteligente.

  • A afirmação possui o quantificador universal "todo". Para negar a assertiva, so substituí-lo por algum

    quantificador existencial: "pelo menos um", "algum" ou "existe". nega-se também o predicado da

    sentença. já eliminamos os itens A, B e C pois apresentam quantificadores universais.

    p: Todo animal de 4 patas é mamífero.

    ~p: Existe animal de 4 patas que não é mamífero.

    Gab : E

  • Resolução em vídeo no link: https://www.youtube.com/watch?v=yUIrgYqo20s&t=73s

    Não erre mais essas questões

  •  “Todo animal de 4 patas é mamífero”.

    Utilizando o macete PEA+NÃO, onde P = Pelo menos um, E = Existe um e A = Algum + NÃO = Negação, ficará:

    R= Existe animal de 4 patas que não é mamífero.


ID
1536322
Banca
FGV
Órgão
FIOCRUZ
Ano
2010
Provas
Disciplina
Raciocínio Lógico
Assuntos

Sobre um conjunto de vinte estetoscópios sabe-se que:

I. pelo menos dois deles estão contaminados;
II. dados três quaisquer desses estetoscópios, pelo menos um deles não está contaminado.

Sobre esse conjunto de vinte estetoscópios tem-se que:

Alternativas
Comentários
  • Do I - Se pelos menos dois estão contaminados, isso quer dizer que pode ser dois OU mais de dois!

    DO II - Se eu pego três quaisquer DESSES (dos 20 estetoscópios), pelo menos um não estar contaminado, isso quer dizer que tem no máximo dois contaminados.

    Concluí-se que exatamente 18 não estão contamindos! Letra C.

  • II. dados três quaisquer desses estetoscópios, pelo menos um deles não está contaminado.

     

    Significa que só há 2 contaminados. Porque se tivessem 3 (ou mais), a pessoa poderia ter o ''azar'' de pegar justamente três contaminados... o que desmentiria a frase acima. 

     

    Há 2 contaminados e 18 não contaminados (letra c)

     

  • Não entendi. A questão fala pelo menos 2. Logo pode ser 2 ou mais, por exemplo 3, 4... Então como afirmar "exatamente" 18 ? Se pode variar, ser 17, 16...

  • O comentário do Robert procede. Se pelo menos 2 estão contaminados, significa que mais podem estar, mesmo que não se pegue nenhum em uma amostra aleatória de 3 unidades. Pode-se por exemplo ter 15 unidades contaminadas, e ainda assim coletar três amostras distintas de três unidades sem que nenhuma esteja contaminada.

  • I. pelo menos dois deles estão contaminados;   

    (DO TOTAL DE 20 NO MÍNIMO 2 ESTÃO CONTAMINADOS)

    PODE SER 2 , 3, 4 ...20, SÓ NÃO INFERIOR A 2

    II. dados três quaisquer desses estetoscópios, pelo menos um deles não está contaminado.

    ( SE PEGAR QUALQUER CONJUNTO DE 3 ESTECOSCÓPIOS NO MÍNIMO 1 NÃO VAI ESTAR CONTAMINADO)

    SE CONSIDERAR 2 CONTAMINADOS, SATISFAZ AS DUAS PREMISSAS, LOGO PODEMOS CONSIDERAR 2 CONTAMINADOS E 18 NÃO CONTAMINADOS.

  • Não deveria ser PELO MENOS 18 não estão contaminados? é EXATAMENTE 18 mesmo?

  • não acreditei no potencial da questão está tão fácil e inventei moda, resultado, errei kkkkk

  • fiquei até tonta!


ID
1547374
Banca
FGV
Órgão
DPE-RO
Ano
2015
Provas
Disciplina
Raciocínio Lógico
Assuntos

Considere a afirmação: “Nenhum pintor é cego”.

A negação dessa afirmação é:

Alternativas
Comentários
  • É sempre bom lembrar que negar uma proposição é torná-la falsa. Para que a afirmação "Nenhum pintor é cego" seja falsa, pelo menos um deles o é. 

    Quando negamos proposições com todo/nenhum usamos pelo menos um/existe/algum e vice-versa.
  • Letra (a)


    Para provar que essa afirmação é falsa, basta encontrarmos um pintor que seja cego. Por isso, a negação dessa afirmação pode ser escrita assim:


    “Algum pintor é cego”

    “Existe pintor que é cego”

    “Pelo menos um pintor é cego”



    Excelente comentário Larissa.


    Bons estudos.

  • Considere a afirmação: “Nenhum pintor é cego”. 

    NENHUM --> ALGUM NÃO É        OU          HÁ PELO MENOS 1 QUE NÃO É                OU EXISTE UM QUE NÃO É

    A negação dessa afirmação é:


  • Letra A

    Exemplo da Negação com a palavra todo


    A palavra Todos e Nenhum são universal

    Alguma e alguns são palavras particulares, ou seja, ela particulariza a palavra

    Toda negação universal não pode ser outra afirmação universal, ou seja, tem que ser palavras particulares

    Ex: Todo homens são honesto → negação, algum homem não é honesto

    Nenhum homem é bonito → algum homem é bonito

    Todo A é B = Algum A não é B

    Ex: Todo policial é honesto = algum policial não é honesto.

    Algum A é B = Nenhum A é B

    Ex: Alguma ave é mamífera = Nenhuma ave é mamífera


  • bom comentario Larissa

  • Questão de argumentação lógica.

    A Fabiana já disse tudo!!

  • Negação do "NENHUM" é "ALGUM", ficando:

    Algum pintor é cego. Já que não há essa alternativa pra marcar, podemos trocar o "ALGUM" por sinônimos, como:

    "há pelo menos um", "existe um", "há um"...

    Logo: "HÁ PELO MENOS UM PINTOR CEGO" letra "A"

    Bons estudos!


  • Negação do NENHUM

    Macete: PEA

    Pelo menos um

    Existe um

    Algum


    Nenhum pintor é cego

    negação: Há pelo menos um pintor cego ou existe um pintor cego ou algum pintor é cego.

    Letra A

  • --------------------negação(inverte o TANA, ou seja, ANAT
    Todo---------------Algum não - pelo menos 1
    Algum--------------Nenhum
    Nenhum------------Algum - pelo menos 1
    Aalgum não---------Todo

  • ALGUM, EXISTE OU PELO MENOS UM pintor  é cego.

    gabarito A
  • Entendi assim:

    Nenhum pintor é cego. Logo, todos os pintores enxergam.

    Negando: Há (existe) pelo menos um pintor cego!

  • Nenhum => negação será "algum" ou "pelo menos um".

  • Negação de "nenhum" => há um; pelo menos um.

  • NEGAÇÕES:

    Algum A é B  || Nenhum A é B

    Nenhum A é B  || Algum A é B

    Todo A é B || Algum A não é B

    Algum A não é B || Todo A é B

    Algum A é B || Todo A não é B

    Todo A não é B || Algum A é B 

    No caso em tela, nenhum Pintor é Cego, logo, Nenhum A é B, a única alternativa que se encaixa  como sendo a negação de Nenhum A é B  é   Algum A é B(Há pelo menos um pintor cego),  letra "A"

  • Negação de nenhum:

    Só trocar o nenhum por:

    P - pelo menos um

    E - existe um

    A - algum

  • Gabarito Letra A.

    Para negação de TODO, ALGUM, NENHUM, ALGUM NÃO É, eu utilizo esta tabela e faço a inversão, igual regra de três.

     

    TODO -----------NENHUM

    ALGUM É -----ALGUM NÃO É

    Agora, inverto "regra de três"

     

    TODO=ALGUM NÃO É; PELO MENOS UM NÃO É

     

    NENHUM=ALGUM É; PELO MENOS UM

     

    Funciona!!!

  • Depois de olhar e olhar essa questão, percebi que ele faz menção a nenhum (nem ao menos UM) que caraceteriza a negação fazendo referência ao Gab. A, ou seja, Há pelo menos UM. 

  • Um bizu muito bom que eu aprendi com o Prof. Renato Oliveira, daqui do QConcursos(muito fera esse cara!)

    TODO(S)= P.E.A. + NÃO, sendo P(pelo menos um) / E(Existe um) / A(algum)

    Ex. Em uma questão afirmando que "Todos os gatos são pardos", é só procurar um resposta entre as alternativas que se encaixe com o P.E.A e negar

    Ex: Algum gato não é pardo / ou / Pelos menos um gato não é pardo / ou / Existe um gato que não é pardo

    No caso do NENHUM , exemplificado pela questão, é só usar o P.E.A sem negar

     

  • Muito boa questão!

     

  • gostei da resposta de Alexandre.

  • GAB. A)


ID
1554067
Banca
FGV
Órgão
DPE-RO
Ano
2015
Provas
Disciplina
Raciocínio Lógico
Assuntos

Considere a afirmação: “Nenhum pintor é cego”.

A negação dessa afirmação é:

Alternativas
Comentários
  • Negação de nenhum é PEA

    P = pelo menos um

    E = existe um

    A = algum

  • Negação de Nenhum

    * Macete

    Use o PEA

    - Pelo menos um...

    - Existe um...

    - Algum...

    Enquanto os campeões treinam, as pessoas comuns dormem!

  • Somente na Negação de "Todos" que usa o PEA + não?

  • William Leão, 

     

    Macete dos Quantificadores

    TODO = PEA + NÃO na segunda parte;

    ALGUM= NETONÃO ( NEnhum ou TODO + NÃO na segunda parte);

    NENHUM= PEA

     

     

     

  • Negação: algum pintor é cego.
     

    Logo: há pelo menos um = algum.
     

     

    Gabarito: A

  • NEGAÇÃO

     

                     TODO  ------------------->  ALGUM ... NÃO / PELO MENOS UM ... NÃO ... / EXISTE ... NÃO...

               NENHUM / NÃO EXISTE      ----------------------->  ALGUM 

                     ALGUM                ------------------------->   NENHUM / NÃO EXISTE

    ALGUM ... NÃO / PELO MENOS UM ... NÃO ... / EXISTE ... NÃO... --------->  TODO

  • GAB. A)

    Há pelo menos um pintor cego.

  • alguém explica a b?
  • A negação de nenhum A é B e algum A e B, logo para "nenhum pintor é cego" sua negação será "algum pintor é cego". Gabarito letra a

    Para mais informações ligue 31 9 89025-520


ID
1759723
Banca
FCC
Órgão
TCE-SP
Ano
2015
Provas
Disciplina
Raciocínio Lógico
Assuntos

É verdade que nenhum professor é rico. É verdade que algum advogado é rico. A partir dessas afirmações, é verdadeiro concluir, corretamente, que 

Alternativas
Comentários
  • GABARITO C 


    Fazendo o diagrama você perceberá que a única relação que a questão não passou foi entre advogado e professor, portanto as únicas possíveis conclusões (com o diagrama) seriam: 




    1°) diagrama: Professor, Advogado e Rico nessa ordem, onde o círculo contendo os professores pegaria somente uma parte dos advogados. E o diagrama dos advogados por sua vez pegaria alguns ricos. 



    2°) diagrama: Professor, Advogado e Rico nessa ordem só que dessa vez não haveria intersecção entre professores e advogados, somente haveria intersecção entre advogado e rico relação essa que a própria questão já trás! 

  • O próprio enunciado diz que são verdades que nenhum professor é rico (professor pode ser pobre, ou classe média), e que algum advogado é rico (pelo menos um advogado é rico).

    É possível confirmar que:


    a) todo advogado é professor. (errada)

    Se pelo menos um advogado é rico, logo este único rico não vai ser professor, então está errada.


    b) nenhum advogado é professor. (errada)

    Não posso afirmar isso, pois pode ter advogado que não é rico e que seja professor.


    c) algum advogado não é professor. (certa)

    Mesmo princípio da segunda afirmação, pelo menos um advogado é rico, e se ele for rico ele não é professor.


    d) todo advogado não é professor. (errada)

    Não posso afirmar isso também, só posso afirmar a anterior que pelo menos um não é professor, pois é rico, se for generalizar para todos, logo, todos os advogados serão ricos, o que não é verdade segundo o enunciado.


    e) algum advogado é professor.

    Não posso afirmar com certeza se pelo menos um advogado é professor, pois ele pode não ser rico e ser professor, assim como ele pode não ser rico e não ser professor.

  • É verdade que nenhum professor é rico

    É verdade que algum advogado é rico.

    c) algum advogado não é professor. CERTO, VEJA BEM;

    SE ALGUM( PELO MENOS UM ) ADV É RICO, LOGO EU POSSO DIZER QUE ESSE MESMO 1 ADV  NAO É PROFESSOR, PQ TODO PROFESSOR NAO É RICO!!!!

  • Resolução desta questão no Canal Matemadicas! Segue o link do vídeo explicativo:

    https://www.youtube.com/watch?v=6wPWTw_015Y&list=PLBJoykwJ-tr3f9GnxV2I-8EqfbNSKsHgk&index=15

  • Não pode concluir nada do que o enunciado não disser, o que o enunciado NÂO diz : pode ou não ser verdadeiro;

    Nesse tipo de questão tem que ir na alternativa que é a ùnica que não cabe exceções, dúvidas, a que Perfeitamente se enquadra no enunciado sem sombra de dúvidas.

    Só resolvo desenhando, e vendo as outras possibilidades que o enunciado não disse.

    Geralmente, falou em TODO, a resposta vai ter a palavra ALGUM.

    02:23

  • Gabarito: Letra C

    -----

    Algum advogado não é professor. Justamente o que não é rico porque todo professor não é rico.

  • -Transformar a proposição em condicional e colocá-la no singular ajuda na clareza.

    -Nessa questão o "Nenhum professor é rico" é o mesmo que "Todo professor é pobre".

    Se Algum advogado é rico E Nenhum professor é rico, Algum advogado não é professor.

    Se o advogado é rico e o professor não é rico, o advogado não é professor.

    Se Algum Advogado é rico E Todo professor é pobre, Algum Advogado não é professor.

    Se o advogado é rico e o professor é pobre, o advogado não é professor.

  • SIM! PORQUE ALGUNS SÃO RICOS, E NENHUM RICO É PROFESSOR.

    BEM-VINDO AO RLM!!

  • o advogado que é rico não pode ser professor

    nesse tipo de questões eu sempre crio conjuntos


ID
1766932
Banca
Prefeitura do Rio de Janeiro - RJ
Órgão
CGM - RJ
Ano
2015
Provas
Disciplina
Raciocínio Lógico
Assuntos

Considere-se a afirmação “todo músico é excêntrico". Se esta afirmação é falsa, então é necessariamente verdadeiro que:

Alternativas
Comentários
  • A negação do TODO é ALGUM NÃO

    Todo músico é excêntrico

    Negação: algum músico não é excêntrico.




  • "Todo músico é excêntrico" é Falsa. Ou seja:

    Basta apenas que UM músico NÃO seja excêntrico.

    Assim, é Verdadeiro que pelo menos um músico não seja excêntrico.

    "Algum músico não é excêntrico."

    Gabarito: C


  • A negação de TODO =  "PEA + NÃO" , QUE QUER DIZER, Pelo menos um

                                                                                                        Existe um

                                                                                                        Algum

    Gab. C      Algum músico NÂO é excêntrico

  • Questão utilizando regras de negação.

    ~(Universal positiva) => Particular negativa

     

    Sabendo essa regra, a alternativa salta aos olhos.

  • GABARITO C

    Negação de todo= algum, pelos menos um, existe um...

  • Porque não poderia ser a letra A?


ID
1786150
Banca
IBFC
Órgão
EMBASA
Ano
2015
Provas
Disciplina
Raciocínio Lógico
Assuntos

Sabendo que todo A é B e nenhum C é A, segue necessariamente que: 

Alternativas
Comentários
  • Questão que pra fazer deve-se colocar um diagrama A dentro do diagrama B, isto está de acordo com o enunciado da questão, ...todo A é B certo? ok, agora perceba que nenhum C é A, o que podemos entender? que existe um diagrama C que não tem elementos de A. Faz um círculo pegando parte de B e ficando uma parte de fora já que não dá pra saber se tem elemento de B em C. Dessa forma conseguimos achar a resposta. Fica visível num diagrama que existe uma parte de B que não é C necessariamente. Espero ter ajudado. GABARITO:  D

  • Fiz o mesmo que você, Márcio. No entanto, também verifiquei que é possível concluir que algum C não é B, tendo em vista que o círculo de C pode estar fora do universo de B. 

  • Para o Bruno: o problema é o "necessariamente". Não NECESSARIAMENTE "Algum C não é B" - O C PODE estar todo contido em B!

  • Fiz a questão pensndo em gincanas. Grupo A, B e C.

    E também me restringindo apenas ao que foi DITO pelo enunciado.

    Quem é do A está no B. Questão não fala nada sobre quem ser do B estar no A. Ou seja, se A tiver 30 e B 60 pessoas, por exemplo, as demais podem não estar no A, uma vez que não houve restrições.

    Quem é do C não está no A. Como nem todo B é A, pela dedução acima, pode ter alguém do C dentro do B, desde que não esteja "misturado" com o A.

    Logo,

    A letra A, B, C seriam excluídas.

     

  • Letra C: algum C não é B se somente se Todo C é B ... mas vale, logo não é a resposta da questão.

    AGora, necessariamente algum B não é C

  • Gabarito D

    Todo = algum não

    Nenhum = algum

     

    * cortamos os iguais, ou seja, o A e substituímos.

    Premissa:

    Todo A é B e nenhum C é A.

    Algum B não é C.

     

     

     

     

  • O vídeo do Professor Chagas explica direitinho

  • Muito bom o vídeo do professor Chagas.

  • Como eu resolvi a questão:

    Questão: Sabendo que todo A é B e nenhum C é A, segue necessariamente que: 

    Dei nomes aos elementos:

    A= Pernambucano

    B = Brasileiro

    C = Alagoano


    Todo        (a) Pernambucano  é    (b) Brasileiro
    Nenhum  (c) Alagoano           é    (a) Pernambucano

     

    a) Todo B é A.

    Todo  (b) Brasileiro) é  (a) Pernambucano    = Errado   

    b) Nenhum C é B.

    Nenhum  (c) Alagoano  é  (b) Brasileiro    = Errado

    c)  Algum C não é B.

    Algum  (c) Alagoano   não é  (b) Brasileiro    = Errado


    d)  Algum B não é C

    Algum (b) Brasileiro  não é    (c) Alagoano     = Correto   (sou brasileira, mas não sou alagona)

  • Bom eu resolvi esta questão levando-se em consideração três possibilidades que poderiam ocorrer com C. Logo a única alternativa que podemos afirmar é a letra D.

     

    Nenhum C é B. Errado, não podemos afirmar isso devido as possibilidades de C também ser B na 2º e 3º ocasião.

     

    http://sketchtoy.com/68187465

  • Pelo menos o B que também for A não pode ser C, então é certo que algum B não é C

  • Resolvo essa e outras questões similares aqui nesse vídeo

    https://youtu.be/yyuo20aPJ_U

    Ou procure por "Professor em Casa - Felipe Cardoso" no YouTube =D

  • Questão de compreensão e interpretação de texto. Cuidar para não extrapolar o enunciado.

  • LETRA D

    SEGUE O RACIOCÍNIO:

    SE A é B e nenhum C... (Para aqui) Observa que continua com um A de novo

    ENTÃO A é B e nenhum C é A, algum B não é C

    LOGO TEMOS: ABC, ABC. São sequências!


ID
1810585
Banca
Prefeitura do Rio de Janeiro - RJ
Órgão
Prefeitura de Rio de Janeiro - RJ
Ano
2016
Provas
Disciplina
Raciocínio Lógico
Assuntos

Em uma conversa entre amigos sobre futebol, concluiu-se que:

· alguns jogadores do time A já jogaram pelo time B;

· nenhum jogador do time C já jogou pelo time B.

Considerando-se tais conclusões como verdadeiras, é necessariamente verdadeiro que: 

Alternativas
Comentários
  • Alguém sabe como se resolve não entendi nada.


  • Essa questão se resolve pelo Diagrama de Venn, ou, nem precisa usá-lo. Temos que alguns jogadores do time "A" jogaram pelo time "B", nesse caso, temos uma intersecção entre esses dois conjuntos A e B. A questão ainda menciona que nenhum jogador do time "C" já jogou pelo time "B", temos então um conjunto vazio entre "B" e "C". Como alguns jogadores do time "A" já jogaram pelo time "B" e nenhum jogador do time "C" jogou pelo time "B" temos que "algum jogador do time 'A' nunca jogou pelo time 'C' " , esses jogadores se encontram na intersecção entre "A" e "B", em que "C" não pode fazer parte.

  • Por que não a C?

  • Porque existe a possibilidade de algum jogador do time A jogar no time C, colega Vanessa IPD.

     

    Pensei assim:

     

    Três bolinhas.

     

    A do time A e a do time B unidas (porque a questão fala que alguns jogadores do time A já jogaram pelo time B)
    A do time C somente unida com a do time A (porque a questão fala que nenhum jogador do time C já jogou pelo time B mas existe a possibilidade, ainda que não se tenha certeza, de que o time C tenha jogado no time A)

     

    Analisando as alternativas:

     

     a) algum jogador do time C já jogou pelo time A: é a margem de incerteza, pode ser que sim e pode ser que não. 

     b) algum jogador do time A já jogou pelo time C: é a margem de incerteza, pode ser que sim e pode ser que não.

     c) nenhum jogador do time A já jogou pelo time C: não podemos afirmar isso porque pode ser que sim e poe ser que não.

     d) algum jogador do time A nunca jogou pelo time C: isso podemos afirmar porque mesmo que 99% dos jogadores tenham jogado no time C (admitindo a possibilidade) pelo menos 1% tem que ter jogado no time A porque, do contrário, não existiria time. 

     

    Desculpa o jeito idiota de explicar. Espero ter ajudado.

     

     

  • perfeito Serena!

     

  • Travei aqui! D?

  • sinceramente não consegui chegar a nehuma das alternativas!!!!!

  • Se alguns de A jogaram em B, significa que há alguns jogadores em comum entre A & B.

    "nenhum jogador do time C já jogou pelo time B. "

    B & C não têm nada em comum. No diagrama de Venn, conjunto C nao enconsta no B, nao necessariamente tendo nenhuma relação com A. Logo, a única opção possível é d

     

    "algum jogador do time A nunca jogou pelo time C"

    Se d fosse falso, a afirmação seria que algum jogador do time A sempre jogou pelo time C, o que implicaria que A = C, o que não é possível com a 1° afirmação

  • Questão mal formulada, o gabarito (e a menos errada) é a letra D, mas, na verdade, não se pode afirmar/negar nenhuma das alternativas.

  • Depois que vc olha p "nunca" (letra D) entende pq as outras estão errada. Errei por falta de atenção.  


ID
1814971
Banca
IDECAN
Órgão
PRODEB
Ano
2015
Provas
Disciplina
Raciocínio Lógico
Assuntos

João comprou um notebook numa loja de informática. Considere que:

- João comprou um notebook branco; 
- nenhum notebook em promoção é importado; e, 
- todos os notebooks brancos a venda nessa loja são importados. 

Qual das afirmações é necessariamente verdadeira?

Alternativas
Comentários
  • - João comprou um notebook branco importado fora da promoção, já que todos os notebooks brancos a venda nessa loja são importados e nenhum em promoção é importado , então

     

    A e C) Se nenhum em promoção é importado e todos os brancos são importados, então os notebooks brancos não estão em promoção;  essa sentença é uma negação do afirmado (ALGUM É)

    B) Pode haver importados de outras cores. Todos os brancos é que são importados

    D)CORRETA. Conforme a dedução explicitada (João comprou um notebook branco importado fora da promoção, já que todos os notebooks brancos a venda nessa loja são importados e nenhum em promoção é importado)

  • Fiz pela IMPLICAÇÃO LÓGICA fazendo os conjuntos!!!!


    - O conjunto dos Notebooks Branco está contido dentro dos Notebooks importados

    -O conjunto dos Notebooks em promoção não está contido dentro do conjunto dos Importados.


    Logo, o Notebook em promoção não está dentro do conjunto dos importados que é onde ons brancos estão contidos!!!


    GABARITO LETRA D


    EX NUNC!!

  • Eu segui o mesmo raciocínio, Ramalho Junior 

    Bola pra frente !!!! Bom estudo a todos

  • Fiz uma tabela e coloquei os dados do nootbook que joão comprou:

    1°) João comprou um notebook BRANCO;
    2°) Todos os notebooks brancos a venda nessa loja são IMPORTADOS
    3°) Nenhum notebook em PROMOÇÃO é importado
    LOGO: O NOOTBOOK QUE JOÃO COMPROU É BRANCO, É IMPORTADO E NÃO ESTÁ NA PROMOÇÃO

    IMPORTADO: ok
    PROMOÇÃO: não
    COR BRANCA ok

    GABARITO D
  • Fiz pelo Diagrama de Venn.

     

    Três bolinhas:

    1 de notebooks em promoção

    1 de notebooks importados

    1 de notebooks brancos à venda

     

    A bolinha de notebooks em promoção fica separada porque a questão diz que "nenhum notebook em promoção é importado"

    A bolinha de notebooks brancos fica dentro da bolinha de notebooks importados porque a questão diz que "todos os notebooks brancos a venda nessa loja são importados"

     

    Analisando as alternativas:

     

    a) Algum notebook em promoção é branco.  Não porque todos os notebooks brancos são importados e nenhum notebook em promoção é importado.

    b) Todos os notebooks importados são brancos. Todo os brancos são importados mas o contrário não ocorre.

    c) Alguns notebooks brancos estão em promoção. Não porque todos os notebooks brancos são importados e nenhum notebook em promoção é importado.

    d) João não comprou um notebook em promoção. Verdade porque ele comprou um branco e todo branco é importado e nenhum importado estava em promoção.

     

    Espero ter ajudado.

  • joao nao tem dó de dinheiro,deve ser concursado!


ID
1828189
Banca
Quadrix
Órgão
SERPRO
Ano
2014
Provas
Disciplina
Raciocínio Lógico
Assuntos

Determinado game on line de combate pontua seus jogadores da seguinte forma: cada combate ganho vale 3 pontos; cada empate vale 1 ponto; e as derrotas não contabilizam ponto.

Durante a criação desse game, o programador ficou na dúvida sobre o que fazer quando houvesse empate, entre dois jogadores, na contagem de ponto, após uma série de combates. "Ficaria na frente o jogador com mais vitórias ou o jogador com mais derrotas?" 

Após analisar a situação dada, podemos concluir que: 


Alternativas
Comentários
  • Discordo do gabarito. Como assim vai beneficiar o mesmo jogador, em caso de empate de pontos? 


    Na situação apresentada o número de VITÓRIAS é igual ao de DERROTAS para os 2 jogadores. Como que a dita escolha pode beneficiar sempre o mesmo jogador? Na verdade não beneficia a ninguém, já que vai continuar empatado do mesmo jeito.


    Na minha opinião o gabarito correto deveria ser C.

  • Também discordo do gabarito e, para mim, a alternativa certa deveria de ser a letra C.

  • Se os dois jogadores tiverem jogado o mesmo número de partidas, então a letra D deveria ser a correta.

    Um caso hipotético:
    Amanda tem 4 vitórias, 0 empate e 9 derrotas. Totalizando 12 pontos em 13 partidas.

    Lucas tem 0 vitória, 12 empates e 1 derrota. Totalizando 12 pontos em 13 partidas.

     

    Estão empatados, mas perceba que Amanda tem mais vitórias e mais derrotas que Lucas. Logo, os dois critérios beneficiam a mesma jogadora.

     

    Essa resposta só funciona para o mesmo número de jogos disputados, caso contrário é possível que o critério faça diferença, como por exemplo:

    Amanda tem 4 vitórias, 0 empate e 9 derrotas. Totalizando 12 pontos.

    Lucas tem 0 vitória, 12 empates e 10.000 derrotas. Totalizando 12 pontos.

     

    Nesse caso hipotético Amanda venceria o desempate por mais vitórias, mas Lucas venceria por mais derrotas.

     

    A letra E é absurda, pois se o enunciado admitiu que os jogadores estão empatados, então o critério de desempate não pode ser "o que tiver mais pontos".

  • Comentário:

    Durante a criação desse game, o programador ficou na dúvida sobre o que fazer quando houvesse empate, entre dois jogadores, na contagem de ponto, após uma série de combates. "Ficaria na frente o jogador com mais vitórias ou o jogador com mais derrotas?"

    Entendi agora, com o comentário do Ed.

    " o programador ficou na dúvida sobre o que fazer quando houvesse empate, entre dois jogadores, na contagem de ponto, após uma série de combates."

    Então a questão é no caso de empate entre dois jogadores na contagem, o que fazer?

    A) o correto só pode ser: ficar na frente o jogador com o maior número de vitórias. (não é esse o correto, pode ser tanto o com maior número de vitórias, quanto o com maior número de derrotas, já que a pontuação de ambos é igual, vai depender do critério escolhido pelo jogo)..

    B) o correto só pode ser: ficar na frente o jogador com o maior número de derrotas. (errado, pode ficar na frente o jogador com maior número de derrotas ou maior número de vitórias, já que o número de pontos de ambos é igual, vai depender do critério escolhido pelo jogo).

    C) a escolha entre esses critérios é indiferente, pois nenhuma geraria um desempate. (É diferente sim, se ambos têm o mesmo número de pontos, um pode ter adquirido os pontos por vitórias e o outro por empates, então o número de vitórias de um e de derrotas de outro seriam diferentes, já que uma vitória vale 3 pontos e um empate vale 1 ponto e as derrotas não valem ponto).

    D) a escolha entre esses critérios é indiferente, pois ambas beneficiariam o mesmo jogador. (se os jogadores tiverem jogado o mesmo número de partidas, o critério de desempate para o jogador com mais vitórias ou o jogador com mais derrotas, beneficiaria apenas um jogador).

    Vejamos:

    Como o enunciado só afirmou que seria um critério de desempate, ou seja estão empatados na pontuação.

    ex:

    A - 4Vitórias = 12 pontos + 3Empates = 3 pontos + 10Derrotas = 0 pontos = total de 15 pontos.

    B - 3Vitórias = 9 pontos + 6Empates = 6 pontos + 9Derrotas = 0pontos = total de 15 pontos

    E) o correto, nesse caso, é ficar na frente o jogador com o maior número de pontos. (Como o critério é de desempate, quer dizer que eles já estão empatados na pontuação, e agora querem escolher outro critério de desempate).

    Qualquer erro me avisa no pv.

    Bons estudos

  • Existe um erro na redação da pergunta. O correto seria: “o jogador com mais vitórias ou o jogador com menos derrotas?” Sendo assim, é indiferente, pois com certeza o jogador com mais vitórias teria menos derrotas. E pra vcs que estão dizendo que a alternativa seria a C e que não haveria desempate, nunca assistiram ao campeonato brasileiro? Kkkkk Tmj pessoal!

ID
1830898
Banca
FGV
Órgão
Prefeitura de Cuiabá - MT
Ano
2015
Provas
Disciplina
Raciocínio Lógico
Assuntos

Em certa comunidade são verdadeiras as seguintes afirmações:
• Todo motorista é homem.
• Nenhum homem sabe cozinhar.
É correto afirmar que

Alternativas
Comentários
  • Alguemmm????

     

  • Melhor forma de se fazer é por conjunto!

    Dentro do conjunto dos "homens" está o subconjunto dos "motoristas"

    Como nenhum homem sabe cozinhar, então o conjunto "cozinhar" não pode ser subconjunto do "homens", ficando totalmente separado deste.

    Logo, nenhum motorista sabe cozinhar.

    (desenhando fica mais fácil a compreensão.)

  • Desenhamos três bolinhas:

    1 representando os motoristas

    1 representando os homens 

    1 representando as pessoas que sabem cozinhar

     

    A bolinha do motorista fica dentro da bolinha dos homens porque o enunciado diz que "Todo motorista é homem"

    A bolinha das pessoas que sabem cozinhar fica separada das outras duas bolinhas porque o enunciado diz que "Nenhum homem sabe cozinhar".

     

    Analisando as alternativas:

     

     a) algum motorista é mulher: o enunciado não faz nenhuma relação com as mulheres.  Não extrapole o enunciado só porque faz sentido pensar em homem e mulher!

     b) algum motorista sabe cozinhar: todo motorista é homem e nenhum homem sabe cozinhas. Isso quer dizer que não há motoristas que sabem cozinhar.

     c) toda mulher sabe cozinhar: o enunciado não faz nenhuma relação com as mulheres. Não extrapole o enunciado só porque faz sentido pensar em homem e mulher!

     d) todo homem é motorista: sabemos que todo motorista é homem mas o inverso não acontece.

     e)nenhum motorista sabe cozinhar: correto porque todo motorista é homem e nenhum homem sabe cozinhar. Isso quer dizer que nenhum motorista sabe cozinhar. 

     

    Espero ter ajudado.

  • Todo/ Nenhum = Nenhum ( retira as partes em comum )

     

  • (   (TODO MOTORISTA)   É HOMEN  )     ( SABE COZINHAR )

     

  • Gabarito E

    • Todo motorista é homem.

    • Nenhum homem sabe cozinhar.

    Como todo motorista é homem, então é certeza dizer que ele não sabe cozinhar.


ID
1850392
Banca
FATEC
Órgão
FATEC
Ano
2011
Provas
Disciplina
Raciocínio Lógico
Assuntos

Aristóteles (384 – 322 a.C.), filósofo grego, é considerado pioneiro na investigação sistemática da lógica. Em seus estudos, estabeleceu regras para relacionar proposições dadas (premissas) a uma conclusão.

Um exemplo é o argumento válido, a seguir, atribuído a Aristóteles:

Todos os homens são mortais.

Sócrates é homem.

Logo, Sócrates é mortal.

A validade de um argumento depende exclusivamente da relação existente entre as premissas e a conclusão. Portanto, a validade de um argumento resulta apenas de sua estrutura lógica (forma) e não do conteúdo do enunciado.

Considerando verdadeiras as premissas:

Todo M é P.
Algum S é M.

Pode-se concluir que

Alternativas

ID
1864369
Banca
CONSULPLAN
Órgão
CBM-TO
Ano
2013
Provas
Disciplina
Raciocínio Lógico
Assuntos

A diretora de uma escola afirmou que “todos os alunos desta escola vão mal em alguma matéria”, visto que incluíra, também, alunos que vão bem em todas as matérias, ela então negou sua afirmação. Das sentenças a seguir, assinale a que expressa de maneira correta a negação da afirmação da diretora.

Alternativas
Comentários
  • A negação da frase “todos os alunos desta escola vão mal em alguma matéria” pode ser representada da seguinte forma: “algum aluno desta escola não vai mal em alguma matéria”. Na negação dessa frase, a palavra “algum” pode ser substituída por “existe” ou por “pelo menos um”; no entanto, ela não pode ser trocada por “pelo menos dois”, pois essa expressão não transmite a mesma ideia que “algum”. Nesse caso, a negação nunca poderá ser representada por “nenhum

  • A negação da frase “todos os alunos desta escola vão mal em alguma matéria” pode ser representada da seguinte forma: “algum aluno desta escola não vai mal em alguma matéria”.


    Na negação dessa frase, a palavra “algum” pode ser substituída por “existe” ou por “pelo menos um”; no entanto, ela não pode ser trocada por “pelo menos dois”, pois essa expressão não transmite a mesma ideia que “algum”.


    Nesse caso, a negação nunca poderá ser representada por “nenhum”.
     

    Luis Telles

     

    Gabarito: B

  • bizÙ ,,,PEA+NâO

  • bizÙ ,,,PEA+NâO

  • Depois de se estudar a matéria, o assunto se torna estranhamente muito fácil. FOCO!


ID
1871380
Banca
FGV
Órgão
MPE-MS
Ano
2013
Provas
Disciplina
Raciocínio Lógico
Assuntos

Considerando a afirmativa “ Todos os lápis que estão nesta caixa são vermelhos" como falsa, analise as afirmativas a seguir.

I. Todos os lápis que estão nesta caixa não são vermelhos.

II. Algum lápis que está nesta caixa não é vermelho.

III. Nenhum lápis que está nesta caixa é vermelho.

Assinale:

Alternativas
Comentários
  • Gabarito B. Explicação Arthur Lima

    Prosseguindo, imagine agora a frase:

    “Algum coelho é verde”

    Como encontramos a negação dela? Basta imaginar que alguém nos disse isso e buscar o mínimo que precisamos fazer para desmentir esta pessoa. Neste caso, como a pessoa disse que algum coelho é verde (e não todos os coelhos), não adianta simplesmente encontrarmos um contra-exemplo (um coelho que não seja verde). Precisamos verificar TODOS os coelhos do mundo e confirmar que, realmente, nenhum deles é verde. Por isso, a negação desta frase é:

    “Nenhum coelho é verde”

    ou

    “Todo coelho NÃO é verde”

    ou

    “Não existe coelho verde”

    Para terminar, vamos encontrar a negação de:

    “Algum coelho não é verde”

    Se alguém nos disser isso, precisamos fazer o que para desmentir esta pessoa? Como está sendo afirmado que algum coelho não é verde, precisamos avaliar todos os coelhos e demonstrar que todos eles realmente são da cor verde. Não basta encontrarmos um ou dois coelhos que sejam verdes, ou mesmo um milhão de coelhos verdes, pois se houver por aí algum coelho que seja de outra cor (branco, por exemplo), a frase da pessoa estará correta. Por isso, a negação é dada por:

    “Todo coelho é verde”

    Ficou claro? Espero que sim. Deixo abaixo uma tabelinha com um resumo bem legal para você – embora eu NÃO aconselhe você a decorar, mas sim a ENTENDER a lógica da nossa análise

    https://www.estrategiaconcursos.com.br/blog/raciocinio-logico-negacao-de-todo-e-nenhum-4/

  • Quantificadores

    TODO - universal afirmativa

    ALGUM - pelo menos um, existe um - particular afirmativa

    NENHUM - universal negativa

    Negação do TODO                                                              Negação do ALGUM

    troca todo por algum e acrescenta o não antes do verbo        troca algum por nenhum

    Ex: Todo aluno é calmo - Algum aluno não é calmo.

  • LETRA B

     

    macete:

     

                          NEGAÇÃO

    TODO  --------------------------ALGUM NÃO 

     

    NENHUM -----------------------ALGUM 

     

    ALGUM------------------------- NENHUM

     

    ALGUM NÃO-------------------TODO

     

    perceba que é só inverter

  • macete: TANA

     

                NEGAÇÃO

    TODO --------------------------ALGUM NÃO 

     

    ALGUM -----------------------NENHUM

     

    NENHUM------------------------- ALGUM

     

    ALGUM NÃO-------------------TODO

  • B

  • Simples futuros servidores públicos federais.

    A palavra Todo e equivalente a palavra Algum .

    ".

    estuda Guerreiro ♥️

    Fé no pai que sua aprovação sai

  • NEGAÇÃO

     

                     TODO  ------------------->  ALGUM ... NÃO / PELO MENOS UM ... NÃO ... / EXISTE ... NÃO...

               NENHUM / NÃO EXISTE      ----------------------->  ALGUM 

                     ALGUM                ------------------------->   NENHUM / NÃO EXISTE

    ALGUM ... NÃO / PELO MENOS UM ... NÃO ... / EXISTE ... NÃO... --------->  TODO

  • Só gravar que não se nega TODOS com NENHUM ou eles mesmo e não se nega NENHUM COM TODOS OU ELE MESMO

  • GABARITO: LETRA B

    NEGAÇÕES DAS PROPOSIÇÕES DO TIPO TODO, NENHUM, ALGUM E ALGUM NÃO.

    A Negação de Todo A é B é=Algum A não é B

    A Negação de Algum A não é B =Todo A é B

    A Negação de Nenhum A é B é=Algum A é B.

    A Negação de Algum A é B = Nenhum A é B 

    FONTE: QC

  • GABARITO B

    NEGAÇÃO DE QUANTIFICADORES

    TODO--------------PEA + NEGA

    PEA-----------------NENHUM+MANTÉM

    NENHUM----------PEA + MANTÉM

    *1 PEA = PELO MENOS UM, EXISTE, ALGUM

    *2 TODO =NENHUM + NÃO

  • TODO ALGUM NÃO

    NENHUM | ALGUM

    ALGUM NENHUM

    ALGUM NÃO | TODO

    MANTÉM!

  • TODO = Pelo menos um.

    #Seguimos#Tentando


ID
1872280
Banca
UPENET/IAUPE
Órgão
Facepe
Ano
2015
Provas
Disciplina
Raciocínio Lógico
Assuntos

Se as afirmações Alguma figura geométrica é circular" e “Nenhum polígono é circular" são verdadeiras, necessariamente é também verdade que

Alternativas
Comentários
  • Ao meu ver, é a mesma ideia da questão Q603526.

     

    Temos três bolinhas.

    1. representa as figuras circulares

    2.representa as figuras geométricas.

    3. representa os polígonos.

     

    A bolinha das figuras circulares está unida com a das figuras geométricas porque o enunciado diz que "alguma figura geométrica é circular"

    A bolinha dos polígonos está unida somente com a bolinha das figuras geométricas porque o enunciado diz que “Nenhum polígono é circular" mas isso não quer dizer que ele não possa ser uma figura geométrica (é uma possibilidade).

     

    Analisando as alternativas:

     

     a) alguma figura geométrica não é polígono: correto porque (admitindo a possibilidade de que algum polígono possa ser uma figura geométrica), 99% das figuras geométricas podem ser polígonos mas 1% têm que ser figuras geométricas por uma questão existencial.

     b) algum polígono é figura geométrica: pode ser que sim, pode ser que não.

     c) alguma figura geométrica é polígono: pode ser que sim, pode ser que não

     d) nenhum polígono é figura geométrica: pode ser que sim, pode ser que não

     e) nenhuma figura geométrica é polígono: pode ser que sim, pode ser que não

     

    Mais uma vez, desculpa pelo jeito tosco de explicar. Espero ter ajudado.

  • Serenna alves, teu jeito tosco dá muito certo!

    Só vou raciocinar tosco a partir de agora.

     

  • Galera, a ideia é a seguinte: se  Alguma figura geométrica é circular e Nenhum polígono é circular, as figuras geométricas que estiverem dentro do conjunto das circulares não poderão ser polígonos.

  •  a)

    esse "alguma figura geométrica" é circular o que nao é poligono consoante "Nenhum polígono é circular"

  • SILOGISMO

    TODO E TODO = TODO

    TODO E ALGUM = ALGUM

    TODO E NENHUM = NENHUM

    ALGUM E NENHUM = ALGUM

  • https://www.youtube.com/watch?v=jgEFFpXS594

    Professor Josimar Padilha


ID
1874650
Banca
IADES
Órgão
CRC-MG
Ano
2015
Provas
Disciplina
Raciocínio Lógico
Assuntos

Considere que:

Todos os contadores são estudiosos. Alguns contadores são ricos.

Acerca dessas proposições, assinale a alternativa correta.

Alternativas
Comentários
  • Diagramas lógicos:

     Oi Pessoal fiz o diagrama, mas aqui não é possivel inserir imagem.

    A melhor forma de fazer esta questão e montar dois circulos, um com os contadores estudiosos e o outro com os contadores ricos, este último deve pegar um pequena parte do circulo dos estudiosos porque no enunciado cita somente alguns. Assim vc conseguirá analisar e responder a questão.

    a) Alguns estudiosos são ricos. Correto. Alguns contadores estudiosos são ricos. 

    B) Todos os contadores são ricos. Errado. Existem contadores que não são ricos.

    C) Não existe estudioso pobre.  Errado. Nem todos os estudiosos são ricos, então pode haver pobre.

    D) Todos os ricos são contadores. Errado. Somente alguns ricos são contadores. 

    E) Todos os estudiosos são ricos. Errado. Somente alguns esdutiosos são ricos.

     

  • Sabendo a "força" dos quantificadores, só existe uma possibilidade de alternativa certa.

    Por ordem de "força", temos: Universal positiva, universal negativa, particular positiva e particular negativa. 

    A conclusão de um argumento nunca terá um quantificador mais "forte" do que os que foram utilizados em suas premissas. Sendo assim, quando se utiliza o particular positivo no enunciado, só existe uma alternativa que pode ser a correta.

    As alternativas B, D utilizam-se do universal positivo. A alternativa utiliza-se do universal negativo.

    Portanto, gabarito é A.

  • Gabarito: Letra A.

    Os estudiosos que são ricos são alguns pertencentes ao conjunto dos contadores. Já que todo contador está dentro do conjunto dos estudiosos.

  • Minha mente simplesmente entra em parafuso só ao ler, sério. Um bloqueio nesse tipo de questão. Me deem ideia de como aprendo isso. pls

  • Caro amigo Gleivan oliviera, se te ajudar eu uso o seguinte metodo:

    TODOS = DENTRO (circula da premisa 1, dentro da premisa 2.

    ALGUM (mesma coisa que EXISTE ou PELO MENOS UM) = CRUZA. ( Os circulos da premisa 1 e 2 se cruzam, formando uma intersecção.

    NENHUM = FORA. (Os circulos das premisas são "desenhados" de forma separada).

  • joga no diagrama q da certo

ID
1884271
Banca
FUNCAB
Órgão
EMSERH
Ano
2016
Provas
Disciplina
Raciocínio Lógico
Assuntos

Uma escola de dança oferece aulas de zumba, samba, sapateado, forró e frevo. Todas as professoras de zumba são, também, professoras de samba, mas nenhuma professora de samba é professora de sapateado. Todas as professoras de forró são, também, professoras de frevo, e algumas professoras de frevo são, também, professoras de sapateado. Sabe-se que nenhuma professora de frevo é professora de samba, e como as aulas de samba, forró e sapateado não têm nenhuma professora em comum, então:

Alternativas
Comentários
  • ´Se desenhar o diagrama fica super tranquilo. Vou tentar explicar:

    A bolinha de zumba está dentro da de samba.

    Separadamente, a bolinha de forró está dentro da bolinha de frevo. 

    Já a bolinha de sapateado só engloba o frevo porque o texto diz que samba, sapateado e forró não têm nenhuma professora em comum.

     

    Depois é só analisar as alternativas.

    Espero ter ajudado.

  • O difícil é desenhar o diagrama corretamente...rs

    Obrigada pela ajuda, Serenna!

  • Desenho: http://imgur.com/a/ShRPS

  • Obrigado pela dica, Serenna. Resolvi facilmente através de diagramas. 

  • R- A

  • Essas questões parecem muito dificeis, mas se você aprender diagramas você resolve rápido


ID
1903525
Banca
FUNCAB
Órgão
SEGEP-MA
Ano
2016
Provas
Disciplina
Raciocínio Lógico
Assuntos

Sabe-se que um executivo é honesto se, e somente se, pratica exercícios físicos. João é um executivo e é sedentário. Pode-se, então, concluir que:

Alternativas
Comentários
  • GABARITO C 

     

     

    Sabe-se que um executivo é honesto se, e somente se, pratica exercícios físicos

    p q 

     

    Que poderá ser reescrito dessa forma: 

    Se um executivo é honesto, então ele pratica exercícios físicos E se um executivo pratica exercícios físicos, então ele é honesto 

    (p --> q) ^ (q --> p) 

     

     

    Com base no exposto acima vamos as alternativas: 

     

    (a) todo executivo é desonesto. ERRADO. Os que praticam exercícios são honestos! 

    (b) todo executivo pratica exercícios físicos. ERRADO. Os executivos que são honestos 

    (c) João não é um executivo honesto. CERTO. João é sedentário, logo ele não pratica exercícios o que nos leva a concluir que ele NÃO é honesto

    (d) todo executivo é honesto. ERRADO. Os executivos que são sedentários, assim como o João, não são honestos.

    (e) nenhum executivo pratica exercícios físicos. ERRADO. Os honestos praticam exercícios físicos 

  • GAB: "C"

    Nessa utilizei o método da loucura mesmo. 

  • Tratei como duas premissas, considerando P2 como verdadeira por ser uma conjunção.

     

    P1: Sabe-se que um executivo é honesto se, e somente se, pratica exercícios físicos. (bicondicional) 

                                               F                          <->                        F    =    V

    P2: João é um executivo e é sedentário (conjunção)

                V                      v              V   =     V

     

    Concluindo que João é um executivo, por ser um executivo pode não ser honesto.

  • O se e somente será será verdadeiro quando V V (ambas verdadeiras) ou F F (ambas falsas), logo ele diz que  "um executivo é honesto se, e somente se, pratica exercícios físicos. "João é um executivo e é sedentário. 

    Se João é sedentário, significa que ele não pratica exercícios físicos, portanto ele não é um executivo honesto. 

     

     

     

  • C

     

    Bicondicional que não satisfez a condição... dá F.

    V + V = V 

    F + F = V

  • Vejam o vídeo que gravei com a resolução desta questão:


    https://youtu.be/9Ongop35leE

     

    Professor Ivan Chagas
    www.gurudamatematica.com.br

  • Obg, Grande Ivan Chagas !  Deus te ilumineeee !!

  • http://sketchtoy.com/68105540

  • A é condição suficiente e necessária para B

    é condição suficiente e necessária para A

    ou seja, praticar exercicios físicos  é condição suficiente e necessária para ser honesto.

    Níguem que seja sedentário é Honesto

    Sendo João sedentário, ele não é Honesto

  • - um executivo é honesto (F), SE E SOMENTE SE, pratica exercícios físicos (F) {No bicondicional pra ser  (v) basta que tudo seja igual}

    - João é um executivo (v) E é sedentário (v) (ou seja, NÃO pratica exercícios físicos). {Na conjunção pra ser (v) tudo tem que ser (v)} --- faz primeiro essa 

     

    Entao...

    Joao NÃO é um executivo honesto.

     

  • Só resolver a questão de trás pra frente...

    "João é um executivo e é sedentário."   e;

    "um executivo é honesto se, e somente se, pratica exercícios físicos."

    Conclusão, para ser um executivo honesto, tem que praticar exercícios físicos. Logo, se João é um executivo sedentário (não pratica exercícios físicos), ele não é honesto.

    Gabarito: C

  • GABARITO:  C

    _____________________________

    Tabela verdade: BICONDICIONAL (↔)

    A proposição será verdadeira (V) se o valor 

    verdade das componentes, antecedente e c

    onsequente, for igual.

     

    A---C / A ↔ C

    V---V       V

    V---F       F

    F---V       F

    F---F       V

    _____________________________

  • Gab-c. Primeiro fiz usando o diagrama do se, e somente se. Depois  fiz da seguinte forma:

    João é um executivo honesto se, e somente se praticar exercício = V

                                                                João não pratica exercício=

                         F                                                         F                       = V

    Conclui-se que  para que a sentença seja verdadeira é necessário que  João não  seja um executivo e não pratique execícios (que é o mesmo que ser sedentário)

     

     

  • a questão diz um executivo somente é Honesto se praticar exercicios fisicos

    se joão é sedentário ( não pratica exercicios fisicos) logo ele é um executivo NÃO honesto

  • Fazendo a tabela verdade fica fácil de identificar que em "se somente se", aquele que não pratica exercícios físicos não é honesto. F <-> F = V sendo que V <-> F = F


ID
1910293
Banca
FUNCAB
Órgão
EMSERH
Ano
2016
Provas
Disciplina
Raciocínio Lógico
Assuntos

Partindo das premissas:

I. Todo médico é formado em medicina.

II. Todo médico é atencioso.

III. Ribamar é atencioso.

IV. Francisca é funcionária do hospital.

Pode-se concluir que:

Alternativas
Comentários
  • Achei essa questão um pouco confusa. Vou colocar como pensei e se alguém tiver um raciocínio melhor, agradeço se comentar.

     

    Bom, pensei assim:

    temos uma bolinha de médico que está dentro da bolinha de medicina. Temos a bolinha de atencioso que engloba totalmente a bolinha de medicina e de médico. Temos a bolinha de ricardo que perpassa as três mas somente nos dá certeza que Ricardo é atencioso, sendo as características de médico e formado em medicina, apenas possibilidades (pode ser que ele seja e pode ser que ele não seja. A bolinha da Francisca foi só pra confundir e acho que ela poderia englobar médico, formado em medicina e atencioso mas essas também são apenas possibilidades (pode ser que seja e pode ser que não seja).

     

    Dessa forma, analisei as alternativas desse modo:

     

    a) Francisca pode ser atenciosa ou pode não ser.  A única certeza que temos é que ela trabalha no hospital.

    b) Nem pensei em inserir o Ribamar na bolinha do hospital de Francisca. De toda forma, seria apenas uma possibilidade ele trabalhar no hospital. Não podemos afirmar.

    c) Todo médico é atencioso e todo médico é formado em medicina, então quer dizer que existe pelo menos uma pessoa que é atenciosa e formada em medicina, pelo menos um médico. Não sei quantos mas posso afirmar que pelo menos um é. 

    d) Não há qualquer relação de matrimônio na questão

    e) Ribamar pode ser ou não ser formado em medicina, é algo que não se pode afirmar. 

     

    Espero ter ajudado. 

  • Nunca pode-se acrescentar informaçõs, só vale o que está na premissa.

    Se não tem certeza de uma coisa, não marca. Só pode assinalar o que certamente está correto, ou seja o que "pode ser" não vale 

  • Essa tá mais pra exame psicotécnico do Detran :p

  • https://uploaddeimagens.com.br/imagens/gra-jpg--2

     

    Desenho da resposta ^

     

    Gab: Letra C

  • A pessoa pode ser atencioso sem ser formado em medicina. Pelo diagrama não dá pra ter certeza da alternativa C .... e pelo diagrama que foi feito lá no início ..o atencioso pode ficar também dentro de formado em medicina. Ou também englobar todo o formado em medicina . ... Ou seja , tá mais pra letra A como sendo a alternativa correta ..

  • Ribamaaaar

  • Se TODO médico é atencioso e TODO médico é formado em medicina, logo, EXISTEM pessoas atenciosas que são formadas em medicina

    Gabarito = C


ID
1910302
Banca
FUNCAB
Órgão
EMSERH
Ano
2016
Provas
Disciplina
Raciocínio Lógico
Assuntos

Uma escola de dança oferece aulas de zumba, samba, sapateado, forró e frevo. Todas as professoras de zumba são, também, professoras de samba, mas nenhuma professora de samba é professora de sapateado. Todas as professoras de forró são, também, professoras de frevo, e algumas professoras de frevo são, também, professoras de sapateado. Sabe-se que nenhuma professora de frevo é professora de samba, e como as aulas de samba, forró e sapateado não têm nenhuma professora em comum, então:

Alternativas
Comentários
  • Só utilizar os diagramas lógicos. 

  • Desenho: http://imgur.com/a/ShRPS


ID
1914952
Banca
FUNCAB
Órgão
EMSERH
Ano
2016
Provas
Disciplina
Raciocínio Lógico
Assuntos

Uma escola de dança oferece aulas de zumba, samba, sapateado, forró e frevo. Todas as professoras de zumba são, também, professoras de samba, mas nenhuma professora de samba é professora de sapateado. Todas as professoras de forró são, também, professoras de frevo, e algumas professoras de frevo são, também, professoras de sapateado. Sabe-se que nenhuma professora de frevo é professora de samba, e como as aulas de samba, forró e sapateado não têm nenhuma professora em comum, então:

Alternativas
Comentários
  • Gab. A

     

    Por eliminação dá pra fazer:

    Toda Z é Sam

    Nenhuma Sam é Sap

    Toda For é Fre

    Alguma Fre é Sap

    Nenhuma Fre é Sam

     

    Se colocar em diagrama teremos: circulo da zumba dentro do samba, separados dos outros.

    Têm-se Forro dentro do frevo e o circulo do sapateado com interseção apenas em frevo.

    Das alternativas, apenas a A é possível.

  • Desenho: http://imgur.com/a/ShRPS


ID
1929382
Banca
FGV
Órgão
Prefeitura de Paulínia - SP
Ano
2016
Provas
Disciplina
Raciocínio Lógico
Assuntos

Considere verdadeira a afirmação: “Toda criança gosta de correr”.

Considere as afirmativas a seguir.

I. Como Abel não é criança, então não gosta de correr.

II. Como Bruno gosta de correr, então é criança.

III. Como Carlos não gosta de correr, então não é criança.

Assinale:

Alternativas
Comentários
  • GABARITO C 

     

     

    “Toda criança gosta de correr”

     

     

    I. Como Abel não é criança, então não gosta de correr. ERRADO --> Nada impede que Abel seja adulto e goste de correr

    II. Como Bruno gosta de correr, então é criança. ERRADO --> Com a mesma justificativa da proposição I Bruno pode ser adulto e gostar de correr.

    III. Como Carlos não gosta de correr, então não é criança. CERTO -->  Para ser criança é necessário que o indivíduo goste de correr caso contrário não será criança. 

  • "Toda criança gosta de correr"   "Verdade"

    I - Como Abel não é criança, não podemos afirmar que ela não gosta de correr.

    II - Como Bruno não gosta de correr, não podemos afirmar que ele é criança.

    III - Como Carlos não gosta de correr, então ele não é criança. (pode se afirmar) VERDADEIRA

  • Para ficar mais fácil a interpretação do texto, vamos mudá-lo:

    p= goiano

    q= brasileiro

    A regra da implicação lógica diz que ( p é condição suficiente para q). Logo, se a pessoa for goiano, ela será brasileira.

    E diz também que ( q é condição necessária para p). logo, ser brasileiro é necessário para a pessoa ser goiano.

    Então a resolução fica assim:

    I - Abel não é goiano, então Abel não é brasileiro ( errado, pois ele poder ser guaúcho, catariense [..] e ser brasileiro).

    II -  Bruno é brasileiro, então Bruno é goiano ( errado, pois ele poder ser brasileiro e ser guaúcho, catariense [..]).

    III - Carlos não é brasileiro, então Carlos não é goiano (correta, pois ser brasileiro é condição necessária para uma pessoa ser goiana. Q é necessário para P).

     

  • A I e II não se pode ter certeza

  • Considere conjuntos:

    [     (Criança)        ] Gosta de Correr

    O conjunto "Criança" está contido dentro do conjunto "Gosta de Correr", mas pode haver outros dentro do conjunto "Gosta de Correr", por exemplo um adulto(Abel - Afirmação I ou Bruno - Afirmação II). Perceba que toda criança deve estar no conjunto "Criança" que está contido em "Gosta de Correr", logo a afirmação III se refere a uma pessoa que não gosta de correr e portanto não é criança.

     

     

     

  • Se criança, então gosta de correr (p -> q)

    Sabendo que p -> q = ~q -> ~p, se não gosta de correr, não é criança.

  • Toda Criança gosta de Correr

    (P -> Q) onde:

    P: É Criança

    Q: Gosta de Correr

     

    Tabela Verdade:

    P    |  Q   |  P - > Q    | opção 1 (~p -> ~q)  |   opção 2 (q -> p)   |  opção 3 (~q -> ~p)

    V    |  V   |  V              |           V                   |                  V           |      V

    V    |  F   |  F              |           V                   |                  V            |      F

    F    |  V   |  V              |           F                   |                  F             |      V

    F    |  F   |   V             |           V                   |                  V            |       V

     

    Portanto: A alternativa 3 é a única que se pode afirmar que é verdadeira!

    Gabarito: C

  • É mais fácil quando vc coloca como conjunto. 

    Faz um conjunto grande de quem gosta de correr e coloca um conjunto menor de crianças dentro desse conjunto maior que gosta de correr, Depois é só fazer as análises. 

  • vídeo com a resolução no link:

    https://youtu.be/3SDDBlQQAPc

     

    Se gostou, se inscreva no canal para dar aquela força!!!

  • CRIANÇA (C)

    GOSTA DE CORRER (GC)

    SE É CRIANÇA,ENTÃO GOSTA DE CORRER

    A EQUIVALÊNCIA DA CONDICIONAL ,DÁ-SE POR ~GC --> ~C

    SE NÃO GOSTA DE CORRER,ENTÃO NÃO É CRIANÇA.

    LETRA C

    ESPERO TER AJUDADO...

  • dá pra resolver por diagrama

  • pode imaginar que afirmativa se trata de uma condicional e fazer a equivalência que nesse caso seria contrapositiva

  • http://sketchtoy.com/68878796

  • Gabarito: C

    Fiz por conjuntos e deu certo, graças a Deus! 0/

    Conjunto de quiança está dentro do conjunto de correr. Quem não gosta de correr está fora dos dois conjuntos. Quem está no conjunto gosta de correr, independente de ser quiança melequenta ou não.

  • Como toda criança gosta de correr, podemos concluir que:

    - se uma pessoa é criança, então ela certamente gosta de correr;

    - se uma pessoa não gosta de correr, então ela não é criança;

    Veja que NADA pode ser concluído sobre as pessoas que não são crianças. Estas podem gostar ou não de correr, não temos informações que permitam julgá-las. Assim, vejamos as afirmativas:

    I. Como Abel não é criança, então não gosta de correr.

    ERRADO, não temos informações para julgar quem não é criança.

    II. Como Bruno gosta de correr, então é criança.

    ERRADO, podemos ter adultos que gostam de correr também. Não podemos concluir que Bruno é criança só porque gosta de correr.

    III. Como Carlos não gosta de correr, então não é criança.

    CORRETO, pois se ele fosse criança deveria gostar de correr.

    Resposta: C

  • Como toda criança gosta de correr, podemos concluir que:

    - se uma pessoa é criança, então ela certamente gosta de correr;

    - se uma pessoa não gosta de correr, então ela não é criança;

    Veja que NADA pode ser concluído sobre as pessoas que não são crianças. Estas podem gostar ou não de correr, não temos informações que permitam julgá-las. Assim, vejamos as afirmativas:

    I. Como Abel não é criança, então não gosta de correr.

    ERRADO, não temos informações para julgar quem não é criança.

    II. Como Bruno gosta de correr, então é criança.

    ERRADO, podemos ter adultos que gostam de correr também. Não podemos concluir que Bruno é criança só porque gosta de correr.

    III. Como Carlos não gosta de correr, então não é criança.

    CORRETO, pois se ele fosse criança deveria gostar de correr.

    Resposta: C

  • Como toda criança gosta de correr, podemos concluir que:

    - se uma pessoa é criança, então ela certamente gosta de correr;

    - se uma pessoa não gosta de correr, então ela não é criança;

    Veja que NADA pode ser concluído sobre as pessoas que não são crianças. Estas podem gostar ou não de correr, não temos informações que permitam julgá-las. Assim, vejamos as afirmativas:

    I. Como Abel não é criança, então não gosta de correr.

    ERRADO, não temos informações para julgar quem não é criança.

    II. Como Bruno gosta de correr, então é criança.

    ERRADO, podemos ter adultos que gostam de correr também. Não podemos concluir que Bruno é criança só porque gosta de correr.

    III. Como Carlos não gosta de correr, então não é criança.

    CORRETO, pois se ele fosse criança deveria gostar de correr.

    Resposta: C

  • Como toda criança gosta de correr, podemos concluir que:

    - se uma pessoa é criança, então ela certamente gosta de correr;

    - se uma pessoa não gosta de correr, então ela não é criança;

    Veja que NADA pode ser concluído sobre as pessoas que não são crianças. Estas podem gostar ou não de correr, não temos informações que permitam julgá-las. Assim, vejamos as afirmativas:

    I. Como Abel não é criança, então não gosta de correr.

    ERRADO, não temos informações para julgar quem não é criança.

    II. Como Bruno gosta de correr, então é criança.

    ERRADO, podemos ter adultos que gostam de correr também. Não podemos concluir que Bruno é criança só porque gosta de correr.

    III. Como Carlos não gosta de correr, então não é criança.

    CORRETO, pois se ele fosse criança deveria gostar de correr.

    Resposta: C

  • Nem sei se é uma forma também de ser feita, mas fiz por diagrama e deu certo.

  • Comentário Ivan Chagas

    https://www.youtube.com/watch?v=_4UMIlWOqMY

  • as explicações so professor são muito resumidas para quem está iniciando... estou aprendendo com os comentários dos alunos..

  • Bizu: não tente achar respostas ou preencher lacunas em uma afirmação vaga, análise apenas aquilo que está escrito na sentença.

  • Galera, não uma coisa que me ajudou muito nessa questão foi desenhar os "balõeszinhos" dos conjuntos. Um círculo menor representando todas as crianças e que certamente gostam de correr, e um grupo maior (com o grupo das crianças dentro) das pessoas que gostam de correr. Olhando o desenho pra mim ficou bem claro que pode ter pessoas que gostam de correr e não são crianças, aí deu pra eliminar as alternativas. Espero ter ajudado!!

    Sucesso a todos na trajetória, vamo que vamo!! (Aqui não precisa seguir a norma culta do português né? kkkk

  • Toda criança gosta de correr <=> Se é criança, então gosta de correr.

    Equivalência da condicional: Se não gosta de correr, então não é criança.

    GABARITO -> [C]

  • Para "Toda criança gosta de correr" existem 2 equivalências.

    1ª (~q --> ~p) ficaria assim: Não gosta de correr então não é criança.

    2ª ( ~p v q) ficaria assim: Não é criança ou gosta de correr.

    Logo, somente a III proposição representa a equivalência. Sendo ~q --> ~p, Carlos não gosta de correr, então ele não é criança.

  • "Considere verdadeira a afirmação: “Toda criança gosta de correr”."

    SE criança ENTÃO gosta de correr.

    A tabela verdade da condicional é:

    V V = V

    V F = F

    F V = V

    F F = V

    Se a primeira parte (criança) for falsa, a segunda parte (gosta de correr) pode ser V ou F que a afirmativa será verdadeira.

    F V = V

    F F = V

    Já se a primeira parte (criança) for verdadeira, a segunda parte (gosta de correr) só pode ser V, caso contrário, a afirmação "Toda criança gosta de correr" seria falsa, sendo que o enunciado já disse que é verdadeira.

    V V = V

    V F = F

    Se a segunda parte (gosta de correr) for verdadeira, tanto faz a primeira (criança).

    V V = V

    F V = V

    Se a segunda parte (gosta de correr) for falsa, então a primeira parte (criança) SÓ pode ser falsa também.

    V F = F

    F F = V

    Resposta C.

  • https://www.youtube.com/watch?v=_4UMIlWOqMY

  • REGRA DA CONTRAPOSITIVA:

    Toda criança gosta de correr”.

    NEGA TUDO E INVERTE:

    SE NÃO GOSTA DE CORRER ENTÃO NÃO É CRIANÇA.

    GABARITO: LETRA C

  • Procurei uma com equivalência e escolhi :)

    deu certo

  • Oi pessoal! Tudo bem com vocês!?

    Caso você goste do meu conteúdo, se inscreve no meu canal, ativa o sininho e indica para os amigos. O link está abaixo. No mesmo, consta a resolução dessa questão da banca FGV.

    https://youtu.be/zIYNUu2t_zE

  • Investe e nega

  • Fiz com círculos e deu certo!


ID
1931587
Banca
FUNCAB
Órgão
EMSERH
Ano
2016
Provas
Disciplina
Raciocínio Lógico
Assuntos

Se todos os maranhenses são nordestinos e todos os nordestinos são brasileiros, então pode-se concluir que:

Alternativas
Comentários
  • Se Maranhense esta dentro do circulo de Nordestino que por sua vez esta dentro de Brasileiro, então...

    Letra C


ID
1940689
Banca
MPE-RS
Órgão
MPE-RS
Ano
2016
Provas
Disciplina
Raciocínio Lógico
Assuntos

Considere as premissas a seguir.


I. Todo funcionário público é solidário.
II. Todo funcionário público é pontual.
III. João é solidário.


A partir dessas premissas, podemos concluir que

Alternativas
Comentários
  • Resposta E

    --------------------------------

    SÓ POR ELIMINAÇÃO MESMO...

    João é funcionário público. NÃO PODEMOS CONCLUIR ISSO

    João é pontual. NÃO PODEMOS CONCLUIR ISSO , POIS NÃO É DITO QUE ELE É FUNCIONARIO PUBLICO

    todos os solidários são pontuais.  NÃO PODEMOS CONCLUIR ISSO

    todos os pontuais são solidários. NÃO PODEMOS CONCLUIR ISSO

    existe solidário.  "RESTA UM"

  • Preciso estudar mais, porque na verdade é uma pegadinha.

     

  • Já que não podemos concluir se é funcionário público ou pontual.

    Existe pelo menos um solidário, JOÃO

  • Abaixo o link com a resolução da questão:

    https://www.youtube.com/watch?v=X51fZygt3Rs

  • Boa Tarde

    Sou novo aqui no QC.

    Gostaria de saber. Por que tem algumas questões nao tem aulas em vídeos ou em texto ?

  • José Dias, não são todas as questões que possuem comentários ou vídeos. Normalmente os professores respondem as questões que são mais indicadas para comentário. Você também pode indicar uma questão para comentário quando tiver dúvidas sobre o seu gabarito ou forma de resolução.  Para isso,  basta clicar em: "+ indicar para comentário" - fica em cima da opção "notificar erro"-, olhe um pouco acima dos comentários e você locarizará...

     

    Bons estudos! =)

  • Perceba que há a única afirmação que podemos realmente afirmar é a assertiva E. 

     

    No desenho - horroroso, que tentei fazer -, a parte hachurada demonstra que poderia haver somente indivíduos solidários, assim como, somente indivíduos pontuais.

     

    http://sketchtoy.com/67568929 

  • Foi por excludente!! Única opção que sobrou.. 

  • PQP essa questão hein!


ID
1957216
Banca
COPEVE-UFAL
Órgão
UFAL
Ano
2014
Provas
Disciplina
Raciocínio Lógico
Assuntos

Se a afirmação "todos os professores de Lógica são milionários" é falsa, então é verdadeira a afirmação

Alternativas
Comentários
  • Negação do "TODO"

     

    PEA + NAO

     

    P - (Pelo menos um)

    E - (Existe um)

    A - (Algum)

     

    "Gabarito E"

  • Veja o vídeo que gravei com a resolução dessa questão:

     

    https://www.youtube.com/watch?v=o7Eptpd7Sqw&feature=youtu.be


    Professor Ivan Chagas

    www.gurudamatematica.com.br

  • kkkkkkkkkkkkkkkkkkkkkkkkkkkkkkkkkkkkkkkkkkkkkkkkkkkk


ID
1960984
Banca
COPEVE-UFAL
Órgão
CASAL
Ano
2014
Provas
Disciplina
Raciocínio Lógico
Assuntos

Dadas as premissas de um silogismo categórico,
1. Alguns homens economizam água;
2. Nenhum ignorante economiza água;

qual a conclusão que torna o argumento válido?

Alternativas
Comentários
  • ALGUM A É

    NENHUM  C É B    OBS ECONOMIZAM ÁGUA ( É COMUM NAS DUAS PREMISSAS) 

    LOGO , ALGUM A NÃO É C 

    O CORRETO SERIA : ALGUNS HOMENS NÃO SÃO IGNORANTES

    AO MEU VER A QUESTÃO ERA PARA TER SIDO ANULADA ,POR FALTA DE ALTERNATIVA .

  • premissa 1)Alguns homens economizam água;

    premissa 2) Nenhum ignorante economiza água;

    --------------------------------------------------------------------

    conclusão: se alguém for ignorante não economiza água

     a conclusão que torna o argumento válido? ou seja, se as premissas forem verdadeiras, a conclusão será tambem verdadeira

    a Alguns homens são ignorantes. (Alguns homens economizam água)

    b) Todo ignorante é homem.  ( pode ser homem ou mulhter)

    c) Nenhum homem economiza água.  (Alguns homens economizam água)

    d) Alguém que economiza água não é homem.  ( pode ser homem ou mulhter)

    e) Alguém que economiza água não é ignorante.   (porque se for ignorante, não economiza de jeito nenhum)

     

  • http://sketchtoy.com/69535870

    a)Alguns homens são ignorantes → conforme o diagrama, não é possível afirmar com certeza;

    b)Todo ignorante é homem → pode ser que alguns sejam, mas também não é possível afirmar;

    c)Nenhum homem economiza água → incorreto. Conforme o enunciado, alguns homens economizam água;

    d)Alguém que economiza água não é homem→ incorreto. Os que economizam água são homens;

    e)Alguém que economiza água não é ignorante→ correto, pois os ignorantes não economizam.


ID
1960996
Banca
COPEVE-UFAL
Órgão
CASAL
Ano
2014
Provas
Disciplina
Raciocínio Lógico
Assuntos

Qual a proposição contraditória à premissa “Algum herói é brasileiro“?

Alternativas
Comentários
  • Letra C.

    Toda A é B Contraditório = Algum A não é B

    Nenhum A é B Contraditório = Algum A é B

    Algum A é B Contraditório = Nenhum A é B

    Algum A não é B Contraditório = Toda A é B

  • é só negar, ou seja, a negativa de algum é nenhum!

  • Letra C

    a questão pede contradição ou seja NEGAR a afirmação de ALGUM é NENHUM


ID
1969609
Banca
COPEVE-UFAL
Órgão
CASAL
Ano
2010
Provas
Disciplina
Raciocínio Lógico
Assuntos

Considere as seguintes premissas: “Quem não bebe fala a verdade.” “Nenhum padre bebe.” Então podemos concluir que

Alternativas
Comentários
  • b) todo padre fala a verdade.

  • Representanto com conjuntos:

    1º círculo - Maior: Fala a verdade

    2º círculo - dentro do maior: Não bebe

    3º círculo - Menor - dentro do círculo não bebe: Padre

    Dessa forma: Todo padre fala a verdade


ID
1969612
Banca
COPEVE-UFAL
Órgão
CASAL
Ano
2010
Provas
Disciplina
Raciocínio Lógico
Assuntos

Somente os baderneiros são presos. Algum professor é preso. Assim, podemos concluir que

Alternativas
Comentários
  • Gabarito:

    c) todos os presos são baderneiros.

  • premissa 1)Somente os baderneiros são presos.

    premissa 2) Algum professor é preso.

    ----------------------------------------------------------------------------

    Conclusão: algum professor é baderneiro e foi preso

    a) somente os professores são presos. (Algum professor é preso.)

    b) somente os presos são baderneiros (somente os baderneiros são presos)

    c) todos os presos são baderneiros. (o critério para ser preso é ser bardeneiro)

    d)todos os professores são baderneiros. (alguns)

    e) nenhum professor é preso(alguns)


ID
1969627
Banca
COPEVE-UFAL
Órgão
CASAL
Ano
2010
Provas
Disciplina
Raciocínio Lógico
Assuntos

Considerando as afirmações: ”Nenhum Presidente é bonito” e “Alguns homens são bonitos”, pode-se corretamente concluir que

Alternativas
Comentários
  • 1) presidente(nenhum é bonito)

    2) bonito(alguns homens)

    Assim:  há homens bonitos, há homens que não são bonitos, há homens que são presidentes, mas não existem presidentes bonitos

    logo, alguns homens que não são bonitos, são presidentes.

    letra A

  • Comentário dessa questão no Youtube.

     

    Questões Comentadas Professor Ivan Chagas (do Canal Guru da Matemática)
    0872 - COPEVE/UFAL - 2010 - CASAL - PROPOSIÇÕES CATEGÓRICAS
    https://www.youtube.com/watch?v=qbefNey61a8


ID
1969630
Banca
COPEVE-UFAL
Órgão
CASAL
Ano
2010
Provas
Disciplina
Raciocínio Lógico
Assuntos

Considerando as seguintes premissas,

Todo marido é ciumento.

Todo marido é trabalhador.

Stefany é ciumento.

Pedro é trabalhador.

podemos concluir que  

Alternativas
Comentários
  • usem diagramas!

    1) são3 circulos um dentro do outro: marido, ciumento e trabalhor

    2) insira stefany no circulo de ciumento

    3) insira pedro no circulo trabalhador

    4) releia as alternativas

    só pode ser a letra D

     

     


ID
1969633
Banca
COPEVE-UFAL
Órgão
CASAL
Ano
2010
Provas
Disciplina
Raciocínio Lógico
Assuntos

Considere as seguintes afirmações: Se “alguns felinos são leões” e “Todos os leões são ferozes”, então, necessariamente,

Alternativas
Comentários
  • a) algum felino é um animal feroz.

  • questão passivel de anulação pois nada se afirma se são "animais". Contendo ainda quesitos sem a expressão "animais".


ID
1977901
Banca
VUNESP
Órgão
MPE-SP
Ano
2016
Provas
Disciplina
Raciocínio Lógico
Assuntos

Suponha serem verdadeiras as afirmações:

Nenhum arrogante é simpático.
Alguns mentirosos são simpáticos.

A partir dessas afirmações, é necessariamente verdadeiro que

Alternativas
Comentários
  • (A)  Algum mentiroso é arrogante? ( não pode afirmar isso)

    (B) Nenhum mentiroso é arrogante? ( não pode afirmar isso)

    (C) se um mentiroso é simpático, então ele é arrogante?  (pode se afirmar que é simpático  mais não que ele é arrogante tornando a questão)

    (D) se um mentiroso não é simpático, então ele é arrogante? (não podemos afirmar que mentiroso é arrogante)

    (E) Algum mentiroso não é arrogante (podemos afirmar que algum mentiroso não é arrogante pois alguns mentirosos são também simpáticos)

  • Se alguém puder, coloca o diagrama ai por gentileza

  • ~E (X)( arr(x) ^ S(x))

    E (x) (Men(x) ^S(x))

    A ^B (v) ^(v) verdadeiro

    Assim, podemos deduzir Algum mentiroso não é arrogante..

     

  • para entendermos essa questão, façamos 3 círculos (tipo os anéis olímpicos) e pintemos conforme o enunciado ... assim, a visualização das alternativas fica mais clara. ..... faça a análise alternativa a alternativa, caso a resposta seja: ¨pode ser¨ ... então, não a considere....

    é difícil explicar a distância ............rsssssssssss  .......................... com um pouco de conhecimento da disciplina as questões se tornam um desafio interessante.

     

  • Ou diagrama de venn ou tabela da verdade, ajuda

  • Dúvida: No caso da alternativa B, no diagrama, os mentirosos ficam totalmente fora do círculo dos arrogantes, então não seria uma afirmação verdadeira também?

  • Eu cai na questão B também pois as seguintes afirmações:

    Nenhum arrogante é simpático. 
    Alguns mentirosos são simpáticos.

    Posso deduzir que "nenhum mentiroso é arrogante."

    Porém, segundo o professor Renato do QC, ele disse que há duas hipóteses e a segunda hipótese, os mentirosos podem estar dentro dos simpáticos e dentro dos arrogantes.

    Não entendi muito bem.

  • O diagrama deve ser feito com a intercessão entre arrogante e mentiroso. E o círculo simpático fica dentro do círculo de mentiroso (sem adentar no arrogante). Se nenhum arrogante é simpatico, ele pode ser mentiroso e muitas outras coisas, menos simpático (pois é o que a proposição traz). 

  • Ué... 

    I. Nenhum arrogante é simpático. BELEZINHA
    II. Alguns mentirosos são simpáticos. OU SEJA, alguns mentirosos são simpáticos, outros EU NÃO SEI.

     

     

    E) algum mentiroso não é arrogante. 

  • Com diagrama resolve rapidinho!

     

    Apelo ao QC!!!

    Colocar uma ferramenta de desenho para as questões de matemática e RL ;)

  • Se fizer as 2 hipóteses como o professor diz não tem erro, o problema é que sempre esqueço a primeira hipótese.....

  • demorei alguns minutos para descobrir o quão simples é essa questao rs

    oras, se alguns mentirosos sao simpaticos, entao algum mentiroso não é arrogante, pq afinal, NENHUM arrogante é simpatico!

  • Fiz diagramas

  • vídeo com a resolução da questão no link:

    https://youtu.be/_yE7QfN9X7s

  • GABARITO

    LETRA E

  • consegui pelo diagrama

  • O professor colocou duas hipoteses...mas dá para fazer com uma só ;). Se os colegas verem o video do professor, e desenhas os "arrogantes", separadamente de os "simpaticos" com intersecção com "mentirosos", a unica opção que não deixa duvida é a "E"...

    Negócio é lembara do bendito diagrama.

  • Olá pessoal,
     
    Vejam o vídeo com a resolução dessa questão no link abaixo
    https://youtu.be/TDHOI_sMyt8
     
    Professor Ivan Chagas
    Gostou? Doe: https://pag.ae/blxHLHy

  • Daria para fazer pela regra do corte?

  • O enunciado não relaciona diretamente os mentirosos aos arrogantes. Eu pediria recurso para essa questão.
  • Segue o link com o diagrama da questão

    http://sketchtoy.com/70468082

  • A- algum mentiroso é arrogante.

    Não podemos afirmar nada sobre isso, a questão não nos trouxe essa informação. 

    B- nenhum mentiroso é arrogante.

    Não podemos concluir isso, pois pode ser que os mentirosos sejam arrogantes, e a questão nada diz sobre isso.

    C- se um mentiroso é simpático, então ele é arrogante.

    Não, pois nenhum simpático é arrogante e vice versa, logo, não tem como afirmar que os mentirosos que são simpáticos, são arrogantes.

    D- se um mentiroso não é simpático, então ele é arrogante.

    Não tem como afirmar isso.

    E- algum mentiroso não é arrogante.

    Sim, pois como a questão deixa margem para a criação de 2 diagramas (o 1º com uma intersecção entre mentirosos e simpáticos, e o 2º com duas intersecções, uma entre mentirosos e simpáticos, e outra entre arrogantes e mentirosos), a alternativa correta precisa satisfazer as duas possibilidades de diagramas. 

    Logo, a única que satisfaz isso é a letra E, em que algum mentiroso não é arrogante.


ID
2021317
Banca
MS CONCURSOS
Órgão
Creci - 1° Região (RJ)
Ano
2016
Provas
Disciplina
Raciocínio Lógico
Assuntos

Considere como verdadeiras as duas premissas seguintes:

I – Nenhum professor é veterinário;
II – Alguns agrônomos são veterinários.

A partir dessas premissas, é correto afirmar que, necessariamente:

Alternativas
Comentários
  • Questão super fácil de ser resolvida.

  • Gabarito: B


ID
2022850
Banca
FCC
Órgão
PM-BA
Ano
2009
Provas
Disciplina
Raciocínio Lógico
Assuntos

Sejam as afirmações:


– “Todo policial é forte.”
– “Existem policiais altos.”

Considerando que as duas afirmações são verdadeiras,então, com certeza, é correto afirmar que: 

Alternativas
Comentários
  • O correto é fazer o diagrama e seguir as garantias que a questão da dá.


ID
2038954
Banca
CONSULPAM
Órgão
Prefeitura de Apuiarés - CE
Ano
2014
Provas
Disciplina
Raciocínio Lógico
Assuntos

Todos os jogadores são brasileiros. Assim sendo:

Alternativas
Comentários
  • a)  O conjunto dos os jogadores contém o conjunto dos brasileiros. Errado. É o contrário o conjunto dos Brasileiros que contém os dos Jogadores

    b) O conjunto dos brasileiros contém o conjunto dos jogadores. Correto. Estaria correto também se estive assim: O conjunto dos Jogadores está contido nos Brasileiros

    c) Todos os brasileiros são jogadores. Errado. Seria o contrário

    d) Algum jogador não é republicano. Alternativa 0800 só tá aqui pra ser eliminada mesmo!

  • Letra B.

     

    Na mesma linha de pensamento da questão Q610010.

  • http://sketchtoy.com/69379863


ID
2047138
Banca
FUNCAB
Órgão
EMSERH
Ano
2016
Provas
Disciplina
Raciocínio Lógico
Assuntos

Considere que as seguintes afirmações são verdadeiras:

“Algum maranhense é pescador.”

“Todo maranhense é trabalhador.”

Assim pode-se afirmar, do ponto de vista lógico, que:

Alternativas
Comentários
  • A questão ao comentar que Todo maranhense é trabalhador, o conjunto de trabalhadores vai cobrir os que são maranhenses (que são só maranhenses) e aqueles que são maranhenses e pescadores (um grupo de pescadores está ligado a maranhenses) . E por isso o gabarito é D.

    Espero ter ajudado e bons estudos.

  • http://sketchtoy.com/68640894

  • RESPOSTA D

    “Todo maranhense é trabalhador.”

    eliminamos as alternativas que tenha a possibilidade de algum maranhense não ser trabalhador A, C, E

    Letra A: não dá pra concluir.

    Letra D: Gabarito.

    #sefaz.al2019 #ufal2019 


ID
2050507
Banca
FUNCAB
Órgão
EMSERH
Ano
2016
Provas
Disciplina
Raciocínio Lógico
Assuntos

Considere que as seguintes afirmações são verdadeiras:
“Algum maranhense é pescador.”
“Todo maranhense é trabalhador.”

Assim pode-se afirmar, do ponto de vista lógico, que:

Alternativas
Comentários
  • Pelo o diagrama resolve....

     

  • Alternativa B

    Algum maranhense trabalhador é pescador. Realmente, é só fazer o diagrama.


ID
2053975
Banca
FUNCAB
Órgão
EMSERH
Ano
2016
Provas
Disciplina
Raciocínio Lógico
Assuntos

Se todos os maranhenses são nordestinos e todos os nordestinos são brasileiros, então pode-se concluir que:

Alternativas
Comentários
  • http://sketchtoy.com/68640999

  • é só fazer 3 grupos, maranhenses, nordestinos e brasileiros.

    faz o grupo dos brasileiros maior, depois coloca o grupo dos nordestinos dentro do grupo brasileiros, e por ultimo coloca o grupo maranhenses dentro do grupo nordestinos, e depois conferir nas alternativas, sempre olhando para o seu desenho o que faz mais sentido


ID
2054131
Banca
UPENET/IAUPE
Órgão
Facepe
Ano
2015
Provas
Disciplina
Raciocínio Lógico
Assuntos

Em um grupo de artistas, todo ator, se não for poeta, ou é músico ou é pintor. Todo músico é talentoso. Ora, se não há poeta nem há pintor que não seja talentoso, é certo que, necessariamente,

Alternativas
Comentários
  • GAB: D

     

  • cerebro deu bug 

  • Gostaria de pedir explicação aos colegasm, pois eu não entendi.

    Exite alguma formula para resolver esse tipo de questões ?

    e-mail joao.victor48@hotmail.com

  • Assitam a aula desse professor (Tácio Maciel) ele explica muito bem como responder esse tipo de questão.

     

    https://www.youtube.com/watch?v=XaBTnly-_ds&list=PLvzDflj11alCZDFCP2KWVUgQWBofYU9wC&index=2

     

  • Vou dizer como matei esse questão: http://sketchtoy.com/68641194

    Depois q se resolve 130 questões desse estilo fica bem mais facil ... não desista. Deus é minha força.

  • Típica questão que se torna simples com o tempo, lógica é o tipo de disciplina que requer exercício, atividade constante, é preciso adaptar-se à linguagem das questões. 

    Em um grupo de artistas, todo ator, se não for poeta, ou é músico ou é pintor. Todo músico é talentoso. Ora, se não há poeta nem há pintor que não seja talentoso, é certo que, necessariamente, 

    Vejamos, Um ator pode ser POETA, MÚSICO OU PINTOR. Todo Músico é talentoso, não há PINTOR ou POETA que não seja Talentoso. então, um ator será sempre talentoso, pois será POETA, MÙSICO ou PINTOR. 

    d) todo ator é talentoso.

  • Resolvi de uma maneira que ficou bem fácil.

    1º criam-se três circulos: um pro POETA, um pro MÚSICO e um pro PINTOR.

    2º põe um circulo pequeno, qué o círculo do TODO ATOR, dentro de cada um dos três círculos que já foram desenhados. Veja que não sabemos em qual dos três círculos está o grupo TODO ATOR, pois se diz "todo ator, se não for poeta, ou é músico ou é pintor", mas está em um deles.

    3º quando ele diz que "Todo músico é talentoso", isso quer dizer que o círculo MÚSICO vai está dentro do círculo TALENTOSO.

    4º quando ele diz: "Ora, se não há poeta nem há pintor que não seja talentoso" ele tá dizendo que todo POETA e todo PINTOR  é TALENTOSO e por isso estão dentro do círculo TALENTOSO, assim como também já está o círculo do MÚSICO.

    5º se você fez os desenhos, e se sabemos que o grupo de TODO ATOR está ou no círculo do PINTOR, ou do MÚSICO, ou do POETA, concluímos que dentro ,obviamente, do círculo TALENTOSO, está o grupo do TODO ATOR. Letra D

  • http://sketchtoy.com/69356691

    Eu visualizei assim, será que estou certa?


ID
2054134
Banca
UPENET/IAUPE
Órgão
Facepe
Ano
2015
Provas
Disciplina
Raciocínio Lógico
Assuntos

Analise as proposições abaixo:

I. Existe urso que não é branco.

II. Algum urso não é branco.

III. Todo urso não é branco.

Existe a negação da proposição “todo urso é branco” apenas em

Alternativas
Comentários
  • Vamos à negação do TODO - Existe um que não é, pelo menos um que não é, algum não é

     Logo o I e II estão certos.

    A negação de Todo urso não é branco é esta Algum urso é branco

     

    GABARITO: C

     

    Nunca desista dos seus sonhos!

    Deus é contigo!

  • Quantificadores lógicos.

    São de Dois tipos: Universal (Nenhum e Todo) e Existencial (Existe, pelo menos um e nem todo).

    Para haver negação tem que trocar o quantificados Universal pelo Existencia, vice-versa. 

    Dessa forma fica tranquilo responder a alternativa C.

     

  • Sacanagem!


ID
2063440
Banca
FUMARC
Órgão
CBTU
Ano
2016
Provas
Disciplina
Raciocínio Lógico
Assuntos

Considerando os argumentos lógicos:

I. Alguns uruguaios são pobres. Alguns pobres são mendigos. Logo, todos os uruguaios são mendigos.
II. Todos os alemães são europeus. Nietzche foi um filósofo alemão. Logo, Nietzche era europeu.
III. Todo mineiro é brasileiro e todo curvelano é mineiro. Então todo curvelano é brasileiro.

É CORRETO afirmar que:

Alternativas
Comentários
  • ALGUNS URUGUAIOS = POBRES -  NÃO SÃO TODOS                                ALGUNS POBRES = MENDIGOS - NÃO SÃO TODOS

    PORTANTO, "TODOS URUGUAIS SÃO MENDIGOS ESTÁ ERRADO.

    EUROPA (ALEMANHA (NIETZCHE))  PORTANTO NIETZCHE EUROPEU ESTÁ CORRETO.

    BRASIL(MINAS(CURVELO)) PORTANTO CURVELANO BRASILEIRO ESTÁ CORRETO.

    LETRA B

  • FIZ COM DIAGRAMAS 

  • A melhor forma de responder essa questão é usando diagramas.

  • RESPOSTA B

    I. Alguns uruguaios são pobres. Alguns pobres são mendigos. Logo, todos os uruguaios são mendigos.

    falso, alguns pobres são uruguaios. (certo)

    II. Todos os alemães são europeus. Nietzche foi um filósofo alemão. Logo, Nietzche era europeu.

    certo

    III. Todo mineiro é brasileiro e todo curvelano é mineiro. Então todo curvelano é brasileiro. 

    certo, curvelano (natural de Curvelo-MG)

    #SEFAZ-AL #UFAL2019


ID
2065489
Banca
UPENET/IAUPE
Órgão
Facepe
Ano
2015
Provas
Disciplina
Raciocínio Lógico
Assuntos

Em um curso de idiomas preparatório para intercâmbio, todos os alunos de alemão são também alunos de italiano, e alguns alunos de italiano são também alunos de espanhol. Todos os alunos de inglês são, também, alunos de francês, mas nenhum aluno de francês é aluno de espanhol. Como nenhum aluno de italiano é aluno de francês, e como nenhum aluno de alemão é aluno de espanhol, então

Alternativas
Comentários
  • Letra E.

    O diagrama "Alemão" não cruza em nenhum ponto com o diagrama "Inglês".

  • Letra E

    Desenho:

    https://uploaddeimagens.com.br/imagens/a-jpg--541

  • Serão feitos 2 conjuntos separados:

    1- Conjunto dos Italianos com o dos Alemães dentro, com intersecção do espanhol

    2- Conjunto do Inglês com o do Francês dentro.


ID
2065495
Banca
UPENET/IAUPE
Órgão
Facepe
Ano
2015
Provas
Disciplina
Raciocínio Lógico
Assuntos

Se "todo pernambucano é brasileiro" é uma proposição VERDADEIRA, a proposição

Alternativas
Comentários
  • Fazendo uso da condicional, para que a proposição seja verdadeira só será admitida:

    P->Q (Verdadeiro)

    V  V

    Pois, caso tivermos p=V e q=F (na condicional) esta proposição será F.

    Assim,

    Como 

    Se, todo pernambucano é brasileiro (p=V), então (como se tem a mesma equivalência) algum pernambucano é brasileiro (q=V).

    Logo, B a resposta.

  • Que explicação mais sem sentido colega!Me desculpe a sinceridade,mas sua explicação não tem lógica alguma.O se.....então é V em sendo VV,FV e FF,não necessariamente só VV,como você falou.O que a banca está querendo afirmar ,meio sem lógica,é que: se o todo É ,uma parte deste todo necessariamente será!A equivalência do se....então é negando as duas proposições,invertendo a ordem e mantendo o sinal de condcional,ou então, negando duas vezes,que dá ~P V Q.Se for para tentar explicar tentemos ter mais cuidade e principalmente estudar.

  • b-

    todo pernambucano é brasileiro

    Como algum esta contido em todos, de todos pernambucanos, algum deles tambem sera brasileiro

  •  

    "todo pernambucano é brasileiro"

    Algum (pelo menos um) pernambucano é brasileiro.

    Resposta: B

  • se fosse a negação seria Algum não é..

    Mas a proposição esta verdadeira, e outra verdadeira igual a inicial é Algum é

  • se todo pernambucano é brasileiro, então é lógico que algum pernambucano é brasileiro, tem que ser verdadeiro. por exclusão letra B de presidente.

  • "Todo A é B" nega-se com "Algum A não é B", mas como a questão falou em ser VERDADEIRA, fica "Todo A é B" ==> "Algum A É B", gabarito Letra(B)


ID
2070802
Banca
INSTITUTO AOCP
Órgão
EBSERH
Ano
2015
Provas
Disciplina
Raciocínio Lógico
Assuntos

A negação de “Todos os candidatos vão passar no concurso” é

Alternativas
Comentários
  • Em expressões generalizadoras e particularizadoras como a da questão, a negação ocorre quando se usa o termo particularizador ('existe pelo menos um', por exemplo) para negar um generalizador (todo, nenhum), acompanhado do não (se for o caso) e vice-versa.

    No caso da questão, temos no comando do enunciado uma frase com expressão generalizadora (todos). Logo, precisaremos utilizar um termo particularizador. Nas alternativas, a precisão da letra c (Existe apenas um candidato que não vai passar no concurso) é equivocada, pois nada garante que somente um candidato não vai passar no concurso. O mais correto é mesmo a letra a, com a frase “Existe candidato que não passará no concurso”.

     

    Portanto, gabarito A.

    Bons estudos!

  • Negação de TODOS = PEA

    Pelo menos

    Existe

    Algum

    Sempre seguido do NÃO antes do verbo quando a frase for afirmativa.

    No caso em questão: “Existe candidato que não passará no concurso”.


ID
2075530
Banca
FGV
Órgão
FIOCRUZ
Ano
2010
Provas
Disciplina
Raciocínio Lógico
Assuntos

Considerando a afirmação: “Todo sapo vermelho é venenoso”, é correto concluir que:

Alternativas
Comentários
  • "Todo sapo vermelho é venenoso" equivale a "Se o sapo é vermelho, então é venenoso".

     

    A) Errada. Falácia da afirmação do consequente.

    P → Q ≠ Q → P

     

    B) Errada. Falácia da negação do antecedente.

    P → Q ≠ ~P → ~Q

     

    C) Certa. Transposição da condicional.

    P → Q ≡ ~Q → ~P

     

    D) Errada. A negação de "Todo sapo vermelho é venenoso" é "Algum sapo vermelho não é venenoso". A letra D é o oposto do que se afirma no enunciado.

     

    E) Errada. Não há informação suficiente para concluir a letra E, pois nada foi afirmado a respeito de sapos de outras cores.

     

    https://rlm101.blogspot.com/

  • Inverte as duas e nega

  • Bizu: Todo A é B pode ser transformado em Se Então.

    Todo sapo vermelho é venenoso = Se é sapo vermelho, então é venenoso

    Agora podemos fazer a equivalência. Para isso é só voltar negando a frase.

    Se é sapo vermelho, então é venenoso = Se não é venenoso, então não é sapo vermelho.


ID
2082583
Banca
CETRO
Órgão
Prefeitura de Manaus - AM
Ano
2012
Provas
Disciplina
Raciocínio Lógico
Assuntos

Quem não pergunta fica com dúvidas. Nenhum adolescente pergunta. Logo,

Alternativas
Comentários
  • http://sketchtoy.com/69372816

    Gab. A - Todo adolescente fica com dúvida.

    Uma dica: sempre façam o diagrama.


ID
2082598
Banca
CETRO
Órgão
Prefeitura de Manaus - AM
Ano
2012
Provas
Disciplina
Raciocínio Lógico
Assuntos

Em uma biblioteca, toda história é interessante. Todo livro, se não tem mapas, tem história, ou tem poemas. Ora, não há mapas e não há poemas que não sejam interessantes. Portanto, tem-se que, necessariamente,

Alternativas
Comentários
  • Gab. C

    Não consegui visualizar esse diagrama.

  • Fiz o diagrama assim:

    http://sketchtoy.com/69456069

    gab. C

    Equívocos, podem me corrigir!

  • todo mapa, poema e livro é interessante. Se um livro tem tais características, então também é interessante

    Fiz um desenho, acho que é isso: http://sketchtoy.com/70108093


ID
2082607
Banca
CETRO
Órgão
Prefeitura de Manaus - AM
Ano
2012
Provas
Disciplina
Raciocínio Lógico
Assuntos

Todo gato é egoísta. Algum gato é branco. Logo,

Alternativas
Comentários
  • Tendi nada.

  • http://sketchtoy.com/69372813

    Gab. A - Algum gato branco é egoísta.

  • Se todo gato é egoísta, e algum gato é branco:

    Quer dizer que algum branco é egoísta, porque algum branco é gato, e todo gato é egoísta. Espero ter ajudado.

    Qualquer erro me avise.


ID
2091220
Banca
INSTITUTO AOCP
Órgão
EBSERH
Ano
2016
Provas
Disciplina
Raciocínio Lógico
Assuntos

A negação de “Todos os alunos vão gabaritar a prova de matemática” é

Alternativas
Comentários
  • Gabarito " E "

    “Existem alunos que NÃO vão gabaritar a prova de matemática”.

     

    Negação do "TODO"

     

    PEA + NAO

     

    P - (Pelo menos um)

    E - (Existe um)

    A - (Algum)

  • Negação do TODO

    Algum não é

    pelo menos um não é

    existe um que não é

     

    Negação do ALGUM

    Nenhum

     

    NEGAÇÃO DO NENHUM

    algum é

    pelo menos um é

     

     

  • acho que a letra c também responde, pois basta 1 contra para deixar de ser todos.

  • A letra C não responde porque não há como afirmar que APENAS um não irá gabaritar. O correto seria Pelo menos um. Portanto a alternativa correta é a letra E.

     

  • A negação do "TODO" nunca será "NENHUM". Como dica sempre tente furar a ideia do "TODO"

    Fica a dica!

     

  • TODO = PEA + NÃO

     

    VOCÊ PASSOU!!!

  • Negação de Proposições categóricas: 

    ~ TODO : PEA + NÃO (pelo menos um; existe um (us); algum) 

    ~ PEA : Ne ToNão (Nenhum / Todo não)

    ~ NENHUM: PEA

     

  • letra "e"

    negação de todo: existe... não

  • Letra E.

    e) Temos que a negação de “Todo A é B” será “algum A não é B”, conforme a dica apresentada.

    Dessa forma, a negação de “Todos os alunos vão gabaritar a prova de matemática” será “Existem alunos que não vão gabaritar a prova de matemática”.

    Questão comentada pelo Prof. Josimar Padilha 

  • ✔️ PARA AJUDAR A FIXAR

    OBSERVAÇÃO IMPORTANTE:

    NÃO SE NEGA UM QUANTIFICADOR UNIVERSAL COM OUTRO QUANTIFICADOR UNIVERSAL

    TODO (UNIVERSAL)

    NENHUM (UNIVERSAL)

    ✍ GABARITO: E


ID
2093857
Banca
UPENET/IAUPE
Órgão
Facepe
Ano
2015
Provas
Disciplina
Raciocínio Lógico
Assuntos

Em um grupo de artistas, todo ator, se não for poeta, ou é músico ou é pintor. Todo músico é talentoso. Ora, se não há poeta nem há pintor que não seja talentoso, é certo que, necessariamente,

Alternativas
Comentários
  • Conforme o gabarito upenet letra D

     

  • Conjuntos: 

    1- ( pintor e talentoso)

    2- ( poeta e talentoso)

    3- (Musico e talentoso)

    4 - Intersecção dos 3: ator e talentoso

  • Para começar, represente em Conjuntos cada uma das três situações:

     

    "Todo ator, se não for poeta, ou é músico ou é pintor"

     

    SE: 1. TODO Ator - Poeta - Talentoso; 

    OU: 2. TODO Ator - Músico - Talentoso; (Lembre-se: "Todo músico é talentoso", isso se aplica as situações 1 e 2 - "Ora, se não há poeta nem há pintor que não seja talentoso")

    OU: 3. TODO Ator - Pintor - Talentoso

     

    Depois compare as situações com as respostas.

    GABARITO: LETRA D

  • Resolvi de uma maneira que ficou bem fácil.

    1º criam-se três circulos: um pro POETA, um pro MÚSICO e um pro PINTOR.

    2º põe um circulo pequeno, qué o círculo do TODO ATOR, dentro de cada um dos três círculos que já foram desenhados. Veja que não sabemos em qual dos três círculos está o grupo TODO ATOR, pois se diz "todo ator, se não for poeta, ou é músico ou é pintor", mas está em um deles.

    3º quando ele diz que "Todo músico é talentoso", isso quer dizer que o círculo MÚSICO vai está dentro do círculo TALENTOSO.

    4º quando ele diz: "Ora, se não há poeta nem há pintor que não seja talentoso" ele tá dizendo que todo POETA e todo PINTOR  é TALENTOSO e por isso estão dentro do círculo TALENTOSO, assim como também já está o círculo do MÚSICO.

    5º se você fez os desenhos, e se sabemos que o grupo de TODO ATOR está ou no círculo do PINTOR, ou do MÚSICO, ou do POETA, concluímos que dentro ,obviamente, do círculo TALENTOSO, está o grupo do TODO ATOR. Letra D

  • http://sketchtoy.com/69356691

    Eu visualizei assim, será que estou certa?


ID
2096035
Banca
CESPE / CEBRASPE
Órgão
EMBASA
Ano
2010
Provas
Disciplina
Raciocínio Lógico
Assuntos

Suponha que, devido a um desastre natural, regiões que ficaram sem acesso a água potável recebam periodicamente a visita de caminhões-pipa, os quais distribuem água entre os moradores dessas localidades. Embora todos os moradores tenham direito a água, são consideradas preferenciais as famílias que tenham idosos, pessoas com deficiência, crianças em fase de amamentação e gestantes, que têm o direito de receber água antes das famílias que não são preferenciais. Considerando o contexto apresentado, julgue o item subsequente. 


A negação da afirmação Todas as famílias da rua B são preferenciais é Nenhuma família da rua B é preferencial.

Alternativas
Comentários
  •  

    A negação de TODO ( universal afirmativo) será ALGUÉM (existêncial) 

    Resumo:

    Quantificadores- Negação: teremos que obedecer 2 passos

    1 PASSO: UNIVERSAL PASSA A SER EXISTENCIAL OU EXISTÊNCIAL PASSA A SER UNIVERSAL

    2 PASSO: AFIRMARTIVO PASSA A SER NEGATIVO OU NEGATIVO PASSA A SER AFIRMATIVO

    PS: Teremos q obedecer os dois passos, caso só exista um passo,  a proposição é inválida.

     

     

  • negação do TODO=ALGUM,PELO MENOS 1,AO MENOS 1,EXISTE

     NENHUM = TODO

     

    GABARITO ERRADO

  • Gab E
    Veja a lógica pelos exemplo e não decore, mas entenda !
    RELAÇÃO DE EQUIVALENCIA:

    TODO A É B = NENHUM A NÃO É B (Todo homem é forte = Nenhum homem não é forte...ou...Eu não tenho nada = eu tenho tudo)
    TODO A NÃO É B = NENHUM A É B

    NEGAÇÃO DOS QUANTIFICADORES:
    TODO A É B = ALGUM A NÃO É B [Ex.: ~(Todo homem é forte) = Algum homem não é forte] 
    ALGUM A É B = NENHUM A É B  [Ex.: ~(Algum homem é forte) = Nenhum homem é forte]

    Feito isso, faça mais exercícios...
    Bons estudos !

  • ERRADO!

    UM EXEMPO DE NEGAÇÃO CORRETA DA PROPOSIÇÃO "Todas as famílias da rua B são preferenciais" SERIA  "Alguma família da rua B não é preferencial."

  • Negação de Todo nunca será Nenhum.

  • Expressões generalizadoras (todo, nenhum...) só podem ser negadas com o uso de expressões particularizadoras (existe pelo menos um, algum não é...).

  • Uso o seguinte método:

    TODOS = DENTRO (circula da premisa 1, dentro da premisa 2.

    ALGUM (mesma coisa que EXISTE ou PELO MENOS UM) = CRUZA. ( Os circulos da premisa 1 e 2 se cruzam, formando uma intersecção.

    NENHUM = FORA. (Os circulos das premisas são "desenhados" de forma separada).

  • A Negação de TODO é ALGUM ... NÃO, de forma alguma poderemos negar o Todo com Nenhum.

  • Negação dos Quantificadores

    Todos = algum, nega o verbo.

    Algum, existe, pelo menos um = nenhum (verbo fica igual)

    Nenhum = Algum

    Foco e Fé!

    A luta continua.

  • ERRADO

  • Eu uso o seguinte método:

    Negação do TODO = PEA + NÃO

    P = Pelo menos um

    E = Existe um

    A = Algum

  • NEGAÇÃO DOS TERMOS (TODO E NENHUM) SAO QUANTITATIVOS UNIVERSAL. PARA NEGAR USA-SE ALGUM, EXISTE, PELO MENOS UM, EXISTE ALGUM.

  • Gab.: E

    -> Não se nega um quantificador universal com outro universal (nenhum, todo, qualquer...). Mas sim com um particular/existencial (algum, pelo menos um, existe/há).

  • NEGAÇÃO:

    Todo é = Algum não é

    Algum é = Nenhum é

  • NEGAÇÃO!

    TODO ------ ALGUM NÃO

    ALGUM NÃO --------- TODO

    ALGUM ------- NENHUM

    NENHUM ------ ALGUM

  • GABARITO: ERRADO

    Quando a questão afirma: “Todas as famílias da rua B são preferenciais”, temos um quantificador universal, pois a questão está “generalizando”. Logo, a negação de um quantificador universal não pode ser outro quantificador universal, conforme ocorre na proposição “Nenhuma família da rua B é preferencial”. Portanto, o item está INCORRETO

  • GAB: ERRADO

    ficaria: Alguma família da rua B não é preferencial. Logo, pediu a negação.

    Negação: Todo A é B = Nenhum A não é B

    Nenhum A é B = Algum A é B

  • Não se nega " TODO " com " TODO " e com " NENHUM "
  • NEGAÇÃO DE TODO:

    • algum...não
    • existe um que não
    • pelo menos um não
  • NEGAÇÃO DO QUANTIFICADOR UNIVERSAL - TODO/QUALQUER -

    • TROCA PELO QUANTIFICADOR EXISTENCIAL - EXISTE/ PELO MENOS UM/ ALGUM.
    • NEGA O VERBO
  • Negar Quantificadores:

    Algum: Nenhum + repetir o resto

    Nenhum: Algum + repetir o resto

    Todo: PEA (Algum/Existe um/Pelo menos um) + negar o resto **

    Errado


ID
2106214
Banca
IF-CE
Órgão
IF-CE
Ano
2014
Provas
Disciplina
Raciocínio Lógico
Assuntos

Sabendo-se que todos os riláceos são polibas, que nenhum poliba é ceteu e que todos os ceteus são altarves, é correto concluir-se que

Alternativas
Comentários
  • Gabarito letra E

     

    Se todos os riláceos são polibas, e que nenhum poliba é ceteu; logo nenhum riláceo é ceteu

  • Questão poêmica, 3 alternativas (c; d; e) respondem a questão...

    c) existem polibas que não são riláceos. [sim, os que não são R (fora do conjunto), serão P]...é a velha explicação: - Todo fortalezense é cearense, mas nem todo cearense é fortalezense...vale para toda relação capital/estado....rs

    d) existem altarves que não são ceteus. [sim, os que não são C (fora do conjunto), serão A] .....explicação acima!

    e) nenhum riláceo é ceteu. 

  • Chaves, F. Eu tbm cheguei a essa conclusão mas notei que os itens C e D podem ser valorados em verdadeiro ou falso, porem o item E sempre será verdadeiro fazendo o diagrama ele será verdadeiro já os itens C e D em caso especiais caem em contradição nunca afirmam com precisão o seus elementos e mais um vez o item E sempre estará correto já que C e D podem ser verdadeiros ou falsos.

     

     

    https://www.youtube.com/watch?v=01Vop7h8gEQ

  • Questão realmente controversa, anulável.

    Havendo uma possibilidade, já era. É o caso das letras C e D também.

  • A única errada é a letra B. O certo seria marcar a "incorreta" e não a "correta" como tem no enunciado. Só acho incrível uma questão dessa não ter sido anulada. A lógica sem lógica.

  • Acertei a questão, mas a meu ver as letras C e D também podem estar certas.

  • A questão no é polêmica, mas precisa que vc preste atenção:

     

    Vejamos:

    riláceos (R)

    polibas (P)

    ceteu (C)

    altarves (A)

     

    Vou tentar explicar aqui, mas seria mais fácil vc fazer os conjuntos ai no caderno:

    1°) R está dentro de P, como também haveria a possibilidade de R ser exatamente igual a P

    2°) C está dentro de A

    3°) C não pode ser igual a A, pois se assim fosse, estaria contrário ao que a questão disse: que nenhum poliba é ceteu

    4°) Pode haver uma pequena intersecção entre A e P ou entre A e RP.

     

    Analisando tudo isso, a única opção que eu tenho certeza é que NEHUM R é C

     

    Espero ter ajudado

     

    Bons estudos.

     

  • Questão de pura lógica, se TODOS os RILÁCEOS SÃO POLIBAS e TODOS os CETEUS SÃO ALTARVES, e NENHUM POLIBA (RILACEOS) é CETEUS

    RESPOSTA: LETRA E

     NENHUM RILACEO é CETEUS
     

  • Tem duas corretas, A letra a e e. 

  • Pode ser que todos os altarvez sejam ceteus, portanto a D está incorreta...gabarito E

  • sacanagem kkkk.

  • Ufa, achava que eu que estava indo mal, mas a questão acho que está formulada errada.

  • Sigo a Magali, construindo os diagramas de Venn há 2 respostas corretas possíveis -> Letras (A) e (E)

  • Não têm duas alternativas corretas.
  • também fiquei em dúvida com a C e a D, porém depois pensei:

    C) ...quando a banca falar que "existem polibas que não são riláceos."

    D) e "existem altarves que não são ceteus."

    PODEM SER TODOS. TODOS OS RILÁCEOS SÃO POLIBAS...

    Portanto o mais correto à se afirmar é que :

    "nenhum riláceo é ceteu." POIS já sabemos que nenhum poliba é ceteu, e que todos os riláceos são polibas.

    att

  • também fiquei em dúvida com a C e a D, porém depois pensei:

    C) ...quando a banca falar que "existem polibas que não são riláceos."

    D) e "existem altarves que não são ceteus."

    PODEM SER TODOS. TODOS OS RILÁCEOS SÃO POLIBAS...

    Portanto o mais correto à se afirmar é que :

    "nenhum riláceo é ceteu." POIS já sabemos que nenhum poliba é ceteu, e que todos os riláceos são polibas.

    att

  • Pelos conjuntos dá pra resolver essa questão.

    Letra E.


ID
2110915
Banca
UFBA
Órgão
UFOB
Ano
2014
Provas
Disciplina
Raciocínio Lógico
Assuntos

A expressão simbólica (∀x ∈ R) (∀n ∈ N) [(x ≥ 0) ∧ (n ≠ 0) ⇒ (∃y ∈ R) (yn = x)], em que R e N denotam os conjuntos dos números reais e dos números naturais, respectivamente, representa a proposição “para todo número real não negativo x e todo número natural não nulo n existe um número real y, tal que yn = x”.

Alternativas
Comentários
  • R= (0, 1, 2, ...)

    N= (1, 2, 3, ...)

    y PERTENCE a R , portanto y pode ser 0

    n é DIFERENTE de 0,

    logo,

    até se y=0, ele será ELEVADO à um N° maior que ZERO (pois n é DIFERENTE de 0) ;

    e 0 ELEVADO a qualquer n° maior que zero é igual à zero, que portanto pode ser x e pertencer ao R.

    ......

    difícil explicar como eu consegui entender a questão , as enfim, tentei ai...


ID
2112928
Banca
FUNCAB
Órgão
EMSERH
Ano
2016
Provas
Disciplina
Raciocínio Lógico
Assuntos

Considere que as seguintes afirmações são verdadeiras:

“Algum maranhense é pescador.”
“Todo maranhense é trabalhador.”

Assim pode-se afirmar, do ponto de vista lógico, que:

Alternativas
Comentários
  • GABARITO B 

     

    O ideal nesse tipo de questão é trabalhar com diagramas! 

     

    - O conjunto dos Maranhenses terá intersecção com o conjunto dos pescadores. 

    - O conjunto dos Maranhenses estará contido no conjunto dos trabalhadores 

     

    (a) ERRADA --> Todo maranhense é trabalhador 

    (b) GABARITO 

    (c) ERRADA --> Todo maranhense pescador é trabalhador

    (d) ERRADA --> Todo Maranhense pescador é trabalhador 

    (e) ERRADA --> Existe maranhense trabalhador que não é pescador 

  • Escorregando na própria nasca de bacana

  • KKKKKKKKKKKKKKKKKKKKKKK exatamente


ID
2118844
Banca
CETRO
Órgão
CREF - 4ª Região (SP)
Ano
2013
Provas
Disciplina
Raciocínio Lógico
Assuntos

Se a afirmação “Existem professores que não são pedagogos” é falsa, então é verdade que

Alternativas
Comentários
  • Se a afirmação "Existem professores que não são pedagogos é falsa" é possível deduzir que não existe professor que não é pedagogo, portanto todos os professores devem ser pedagogos.

  • EXISTEM é sinônimo de ALGUM, logo:

     

    ALGUM A não é B <-> TODO A é B

     

    Boa sorte a todos! 

     

  • GABARITO: B.

     

    negação do ALGUM / EXISTE / PELO MENOS UM 

     

    ➜ troca por todo

    ➜ nega

    ou  

    ➜ troca por nenhum

    ➜ repete o resto


ID
2126650
Banca
SEMASA Itajaí
Órgão
SEMASA Itajaí
Ano
2016
Provas
Disciplina
Raciocínio Lógico
Assuntos

Considerando que a afirmação “Todos os candidatos são catarinenses” é falsa, qual das alternativas abaixo é, do ponto de vista lógico, verdadeira?

Alternativas
Comentários
  • A negação de ''toda/todo/todos/todas'' é ''pelo menos um''. Se PELO MENOS UM candidato desse universo de pessoas não for catarinense justifica a negação da preposição ''todos os candidatos são catarinenses''. Concordam que para que isso fosse verdadeiro seria necessário que TODOS, sem haver exceções, fossem catarinenses? Porém, o enunciado nos disse que isso é falso, ou seja, tem PELO MENOS UM que não é catarinense.

     

    Gabarito D

  • GABARITO: LETRA D;

     

    1°) A sentença “Todos os candidatos são catarinenses” é uma proposição que apresenta um quantificador universal, logo devemos trocá-lo por um quantificador existencial (existe um, pelo menos um, algum, existem, etc.).

     

    Vai ficar assim: Pelo menos um candidato é catarinense.

     

    2°) Agora, vamos negar a sentença “Pelo menos um candidato é catarinense”.

     

    Basta colocar o “não” antes do verbo.

     

    Solução:Pelo menos um candidato é não catarinense”.

     

    Conheçam e inscrevam-se no meu canal no youtube, pois sou professor de Matemática e gravei alguns vídeos com dicas e bizus de Matemática e Raciocínio Lógico.

     

    Link do canal: https://www.youtube.com/channel/UC_FQm8aivYBf2q6ga1rxklw?sub_confirmation=1

     

    Fanpage: https://www.facebook.com/profjuliocesarsalustino

  • Letra D

     

     

    Devemos prestar atenção nas palavras Universal (TODO NENHUM) e as Particulares(ALGUM, PELO MENOS UM, EXISTEM...).

     

    Ex: Toda negação universal não pode ser outra afirmação universal, ou seja, tem que ser palavras particulares

     

    Ex: Todo homens são honesto → negação, algum homem não é honesto

     

    Nenhum homem é bonito → algum homem é bonito

     

    Todo A é B = Algum A não é B

     

    Ex: Todo policial é honesto = algum policial não é honesto.

     

    Algum A é B = Nenhum A é B

     

    Utilizando esta tecnica tempos como respostas a letra D

    Pelo menos um candidato não é catarinense. 

     

  • Letra D

    :)

  • Negação de TODOS       

    *Pelo menos um não é

    *Existe um que não seja

    *Algum não é

     

    Negação de NENHUM

    *Algum é

    *Pelo menos um é

  • Negação de TODO 
                          PEA + NÃO>>  pelo menos um, Existe, algum e nega a segunda.

  • "É VERDADE QUE SEJA FALSO" o cerbro nessa hora pdiu oxigenio


ID
2139442
Banca
SEDUC - CE
Órgão
SEDUC-CE
Ano
2016
Provas
Disciplina
Raciocínio Lógico
Assuntos

Assinale a alternativa que nega a seguinte proposição:

Algum professor que trabalha na escola não é efetivo.

Alternativas
Comentários
  • negação de ALGUM  =  TODO

     

    Algum professor que trabalha na escola não é efetivo. 

     

    TODO .................................................. É efetivo.

     

     

  • Gabarito A

    Negação do todo, nenhum e algum

    Algum não é - Todo é

    Nenhum é - Algum é

    Todo é - Algum não é

     

  • Assinale a alternativa que nega a seguinte proposição:

    Algum professor ...não é efetivo.

    TAN - TODO - ALGUM- NENHUM ,  o algum poderá ser negado tanto com "todo ...não" , como com "nenhum". 

    TODO PROFESSOR .... É EFETIVO (já tinha o não, o retira) 

     a) Todo professor que trabalha na escola é efetivo.  resposta

     

  • • Negação ALGUM...NÃO => TODO ALGUM => NENHUM

ID
2154103
Banca
FAU
Órgão
Prefeitura de Piraquara - PR
Ano
2016
Provas
Disciplina
Raciocínio Lógico
Assuntos

A negação lógica à afirmativa abaixo encontra-se em qual opção?

“Nenhuma calça de João é azul”.

Alternativas
Comentários
  • Gabarito: D

     

    Não precisa que todas as calças sejam azuis para que se negue a proposição "Nenhuma calça de João é azul", basta ter só uma que ela já nega, ou seja, a negação de nenhum é algum/pelo menos um...

  • PEA!

  • Negação de Nenhum= algum

    Então resposta correta letra D.


ID
2154109
Banca
FAU
Órgão
Prefeitura de Piraquara - PR
Ano
2016
Provas
Disciplina
Raciocínio Lógico
Assuntos

Analise as afirmativas:

I - Todo professor usa caneta azul.

II - João é professor.

Agora, considerando (I) e (II) verdadeiras, assinale a opção com a conclusão lógica possível diante das duas afirmativas acima.

Alternativas
Comentários
  • O grupo de professores está contido no grupo de quem usa caneta azul(que por sua vez detém o grupo de professores) e João faz parte do grupo de professores, logo ele usa caneta azul.

    Gabarito B

  • Douglas, estamos diante de um caso de Silogismo, que nada mais é do que um argumento que apresenta duas premissas e uma conclusão.

     

    O silogismo é formado por:

    - Premissa maior: possui o termo maior;

    - Premissa menor: possui o termo menor;

    - Conclusão.

     

    O Termo maior é o predicado da conclusão;

    O Termo menor é o sujeito da conclusão;

    O Termo médio liga as duas premissas (mas não aparece na conclusão).

     

    Assim, temos:

     

    P I - Todo professor usa caneta azul. (premissa maior)

    P II - João é professor. (premissa menor)

     

    Professor: termo médio (liga as duas premissas e não aparece na conclusão);

    Joao: termo menor (sujeito da conclusão);

    Caneta azul: termo maior (predicado da conclusão).

     

    Conclusão: João usa caneta azul. (Veja que “professor”, por ser o termo médio não aparece na conclusão).

     

    Espero ter ajudado. Abraço.

  • Joaoeprofessor

  • Caneta azul, azul caneta.

  • Alternativa D é uma premissa, cuidado